Download as pdf
Download as pdf
You are on page 1of 301
aS Ae aah te - First Edition 1971 Sisth Edition 1987 Reptinied 1990 Seventh Edition 1991 Reprinted 1993, 1998, 2000 ighth Edition 2001 mth Edition — 2011 CoryRiGHT © NEM CHAND & BROS, 2011 No part of this publication may be reprinted any electronic, mechanical or any means, Photocopying and recording, translated in any language with oF reproduced or utilized in any ed in any form now known or hereafter invented, inclag? or in any information storage and retiieval syn 'out the prior written peimission of the Publishet/ Auth Edited by Mrs. Indira Khanna & Mrs. Lalitha Justo ISBN 978-81-85240-63-3, Published by Nem Chand & Bros., Civil Lines, Roorkee 247 667, India e-mail : ncb_rke@rediffmail.com Photoset and Printed at the Roorkee Press, Mahavir Marg, Roorkee 247 667 ‘ila PREFACE TO THE FIRST EDITION The basic concept of highway engineering have been considerably changed in the last ‘two decades. In the past, the subject was essentially based on the empirical practices and hum rales. “The design and construction thereby were done mainly based on the experiences of individual engineers, Like in ather countries, in India also several organisation namely, Cenural Road Research Insitute, Minisify of Transport (Roads Wing), P. W. D. Research Stations and some educational institutions have Contributed'a great deal to the knowledge and newer concepts in this field of engineering. All these contributions have given the subject a scientific oullook. The authors have been associated with the teaching and research in this discipline at the University of Roorkee for the past several years and as result ofthis experience this book embodies the latest know-how on the subject. It has been the obyective of the authors to prepare this book introducing the subject of highway engineering to the student in a systematic manner covering the latest knowledge. ‘The treatment of the subject is fully in metric system. Besides being useful forthe Civil Engineering students in general, it is hoped that the book should also fulfill the need of ‘he students of those institutions where this subject is being taught at an advanced level for the final year degree class as an optional paper or for the preparation of a complete Highway Project. In preparing this book, the authors have liberally drawn information from @ number of publications, particularly, of Indian Roads Congress. The authors feel that the book should serve the purpose of a standard text book for the undergraduate students of Civil Engineering for almost all the teaching institutions in the country. It is possible that some errors might have been left unnotices print’. The authors would sincerely welcome the constructive eri ‘book for the subsequent editios, while the book was in ism for improving the Janvary 26, 1971 8. Roorkee © PREFACE TO THE NINTH EDITION ‘The book has been revised in general and efforts have been made to improve the presentation, Suggestions and comments on the contents and subject matter received from the sidents, teachers and professionals will be considered by the authors durin further revision of the book. s Ss. March, 2011 ic. CONTENTS CORTES trey evs INTRODUCTION 8 1.1 Importance of Transportation 1) 12 Bilterent odes of ransporation 2 13 Characteristics of Roads Transport 4 14 Importance of Roads in India 4 1.5 Scope of Highway Engineering 6 HIGHWAY DEVELOPMENT AND PLANNING 9-50 2 2.1. Historical Development of Road Construction 9 22 Highway Development in India 1g 4 23 Necessity of Highway Planning 2 24 — Classification of Roads 2 25 Road Patterns 24 ' 26 Planning Surveys 7 2.7 Preparation of Plans 29 28 Interpretation of Planning Surveys 30 29 Preparation of Master Plan & its Phasing 30 2.10 Highway Planning in India 35 HIGHWAY ALIGNMENT AND SURVEYS 51-70 3.1 Highway Alignment st 3 32 _Engincering Surveys for Highway Location 3s 33 Drawings and Report a 34 Highway Project a HIGHWAY GEOMETRIC DESIGN 11-188 Introduction 1 4 Highway Cross Section Elements B Sight Distance 86 Design of Horizontal Alignment 103 Design of Vertical Alignment 139 TRAFFIC ENGINEERING ies $1 Introduction \s 5 52 Traffic Characteristics ai 53. Traffic Operation au 5.4 Design of Intersections 3H 5.5. Design of Parking Facility x8 $6 Highway Lighting 2 5.7 Traffic Planning & Administration 287 cus Paving Mixes mt € Cement Concrete DESIGN OF HIGHWAY PAVEMENTS HIGHWAY CONSTRUCTION 8.1 General Construction 8 Earthwork Construction of Earth Roads 84 Construction of Gravel Roads 83 Construction Water Bond Macadam Roads 86 Construction of Bituminous Pavements 8.7 Construction of Cement Concrete Pavements 88 Construction of Joints in Cement Concrete Pavements 8.9 Joint Filler and Sealer 8.10 Reinforced Concrete Pavements 8.11 Prestressed Concrete Pavements SOIL STABILIZED ROADS 9.1 Introduction 9 52 Mechanical soi ‘Stabilization 9.3 Soil-cement Stabilization 9.4 Soil lime Stabilizat 9.5 Stabilization of Soil Using.Bituminous Materials 9.6 — Special Problems in Soil Stabilization Work HIGHWAY MAINTENANCE. 10.1 Introduction 10 102 Pavement Faitures 10.3 Maintenance ot Highways 10.4 Pavement Evaluation 10.5 Stengthening of Existing Pavements HIGHWAY DRAINAGE 11.1 Introduction 11 112 imponance or Highway Drainage 11.3 Surface Drainage 114 Sub-surface Drainage 11.5 Drainage of Slopes & Erosion Control 11.6 Road Construction in Waterclooved Areac 481 488 - 517 488 489, 498, 505 507 $18 - 534 518 518 519 528 532 $32 MONWAY puceieenis MILL ROADS. wea? 12 4 122. Alignment of $i Rex 123° Geometries of Hill Road 124 Design & Consteuction of Hill Ravaas i 12.5 Drainage in Hill Roads g 12.6 Maimenance Problems in Hill Roads ROAD SIDE DEVELOPMENT S89 13.1. Environment Factors in Planning and Development of Hghocys ss 13 (32. RoststeDevlomen and bere 13.3 Planning Plastation of Trees ss 13.4 Species and their Selection at 135 Care of Trees HIGHWAY ECONOMICS AND FINANCE. 565-53 Introduction “5 14 12s iigimay User Benetis Highway Costs Economic Analysis, ‘Highway Finance INDEX ul ) Chapter 1 Introduction 1.1 IMPORTANCE OF TRANSPORTATION L.L.1 Role of Transportation ‘Transportation contributes to the economic, industrial, social and cultural development ‘of any country. Transportation is vital for the economic development of any region since ‘every commodity produced whether it is food, clothing, industrial products or medicine needs transport at all stages from production to distribution. In the production stage, transportation is required for carrying raw materials like seeds, manure, coal, steel etc. In the distribution stage, transportation is required from the production centres viz; farms and factories to the marketing centres and later to the retailers and the consumers for distribution, The inadequate transportation facilites retard the process of socio-economic development of the country. The adequacy of transportation system of a country indicates its economic and social development. 1.1.2 Economie Activity and Transport ‘The economic activities are the processes by means of which the produets are utilized to satisfy human wants, TWo important factors well known in economic activity are: (@ Production or supply and i) Consumption for human wants or demand Man and his products are thus not bound to his local surroundings. The importance of transportation in economic activity isto be found in its effects on both human wants for {goods and satisfaction through production and distribution, While discussie the general ao wor transportation, it may be said thatthe increased productivity and its efficent transportation can lower the cost of the produets. The trans influencing factor on consumer price of commodities 1.1.3. Social Effects of Transportation i always settled along the Progress follows the lines of transportation. Population have ‘ rivey shores, road sides and near railway stations. Inthe presen ‘concept of transportation INTRODUCTION 2 network, this kind of ribbon development is greatly discouraged. Attem made to decentralize the population centres away from the sides re being transportation routes. Thus fown planning pattems are rapidly changin congestion around the populated areas suburban living and industrial emer’ ®¥0ld 8 developing, These suburbs and satellite towns, acting as counter-m, linked up with rapid transit systems. The various social effects of tran further elaborated as follows : (a) Sectionalism and transportation : wagnets shoulg tt a Improved transportation has implication in reducing sectionalism within the country and also outside the Po Under-developed colonies and tribes are improving their living conditions @ n= distances have apparently been reduced with reduction in travel time. More-fex & travels in ther pats of te country and outside the country tend to increase knowhatee™ the people from other sections of the society. The intemational understanding for aoe peace and order also improves with efficient network of transportation, se (©) Concentration of population into urban area: The improved transportation network brings prosperity to the urban population, The prosperity and employmen._ portunities of urban area attract the population from other areas resulting in enfament economic activities. Adequate mass transportation facilities are needed to cater the teternal movements in urban area such as daily movements to and from factories, offices sclools, hospitals and other social needs. Efficient rapid transit facilities are necessary for sub-urban and inter-city long distance travel for business needs, social visits and ‘wurist activities. This also encourages the people to live in places away from their work: centres, thus helping to decrease the growth of slums in urban areas. In general the transportation facilities are essential for the well being of the community. (6) Aspect of safety, law and order : Transport facilites are essential for rushing aids to areas affected by an emergency. To maintain law and order at home, itis required to have an efficient svstem of transport network. To defend the territory of the country against the extemal aggression and to guard the borders with the foreign territories, transport facilities are needed connecting the farthest border area from the head quarters or capitals. At times, this alone may be a sufficient reason to develop a transport network ‘hich may not involve any economic and social benefit directly. Alllthe advantages of transportation may now be summarized : (Transportation is for advancement of the community. (ii) Transportation is essential fo the country, and the economic prosperity and general development of (til) Transportation is essential for strategic movem: Country and to maintatn better law and order. ent in emergency for defence of the DIFFERENT MODES OF TRANSPORTATION 3 ‘can be sold atthe nearest marketing centres for more remunerat ive price resulting in faster economic growth and decreased wastage, With improved facilis for education, health care and other social needs in the villages, the urge for the migration to urban centes ‘deereases, thus helping in balance development of the country as a whole, 1.2 DIFFERENT MODES OF TRANSPORTATION Three basic modes of transport are by land, water and air. Land has given scope for development of road and rail transport, Water and air have developed waterways and airways, respectively. The roads or the highways not only include the modem highway system but also the city streets, feeder roads and village roads, catering for a wide range of road vehicles and the pedestrians. Railways have been developed both for long distance transportation and for urban travel. Waterways include oceans, rivers, canals and lakes for the movement of ships and boats. The aircraft and helicopters use the airways. Apart from these major modes of transportation, other modes include pipe lines, elevators, belt conveyors, cable cars, aerial ropeways and monorails. Pipelines are wsed for the transportation of water, other fluids and even solid particles. “The four major modes of transportation are : (Roadways or highways il) Railways ii), Waterways (iv) Airways ‘The transportation by air is the fastest among the four modes. Air travel also provides ‘more comfort apart from saving in transportation time for the passengers and the goods between the airports. Transportation by water is the slowest among the four modes: but this mode needs minimum energy to haul unit load through unit distance. ‘The transportation by water is possible between the ports on the sea routes or along the river: ‘or canals where inland transportation facilities are available. ‘The transportation along the railway track could be advantageous by railways between the stations both for the passengers and goods, particularly for longer distances. These railway tracks could serve as arteries fr tansporaton by land and the roads could serve ‘as feeder system for transportation to the interior parts and to the intermediate localities between the railway stations. The energy requirement to haul unit load through unit i i xth) ofthat required by road tance by the railway is only a fraction (one fourth to one si f Eomiati? full advantage of this mode should be taken forthe transportation of bulk goods along land where the railway facilities are available = The transportation by road is the only mode si cl Tice oe is mode imum flexibility for : a Speed alco the rough any mode of ad vehicle. Is posible o direction, time and of travel etc. igh any piensa pare ‘door to door service only by road transport. The other three modes id 0 vrtation by roads forthe service to an Waterway and rally hve to depend on renspaation Dy ross S00, om their resrecive terminal, aipons, harbours or Sane | oration and therfore needed no only to me mae 2 ety for oad rave by 2 wel lement them, b 10 SUPP erie of roads throughout the country. INTRODUCTION ination between different modes of transportation is desi ee at wvpealtiy competition between the different modes, ny a Thad varying characteristics ‘The branch of transportation engineering which deals with the planning. qe. contaction and maintenance of aiports and other facilities forthe operation ef 4 oittned Aiport Engineering. The development of terminal facilities for shipe meng their harbouring, repairs etc. are covered under Harbour Engineering. The desi boats, taying of railway tracks and yard, their maintenance and the safety and conrot on movements are dealt in Railway Engineering. The planning, design, construct maintenance of road and roadway facili Covered under Road Engineering or Highway Engineering. 1.3 CHARACTERISTICS OF ROAD TRANSPORT It is an accepted fact that of all the modes the transportation, road transport is ity nearest to the people. The passengers and the goods have to be first transported by road before reaching a railway station or a port or an airport. ‘The road network alone could serve the remotest villages of the vast country like ours. ia ‘The characteristics of road transport are briefly listed here. f (® Roads are used by various types of road vehicles, like passenger cars, buses, tucks, two and three wheeled automobiles, pedal cycles and animal drawn vehicles, But railway tracks are used only by rail locomotives and wagons, waterways are used by only ships and boats. i (ii) Road transport requires a relatively small investment for the government. Mott vehicles are much cheaper than other carriers like rail locomotives and wagons, water and air carriers. Construction and maintenance of roads is also cheaper than that of railway tracks, docks, harbours and airports. i (Gi). Road transpor offers a complete freedom to road users to transfer the vehicle fom fone lane to another and from one road to another according to the need and convenience. This flexibility of changes in location, direction, speed and timings of} travel is not available to other modes of transport. Z (¥) In particular for short distance travel, road transport saves time. Trains SP junctions and main stations for comparatively longer time. (¥) Speed of movement is directly related it i i hi . The rosé dtseases wih nceaing dpeion gests eed nocpen h 8 dispersion in speed. Road transport is subjected to 2 eeree of accidents due tothe Nexbility of movements offered othe cod Derailment of railway locomotives and air crash of air planes are a, ‘ommon. They are in fact more disastrous, 7 Road transport is the only a community alike, (vi) Means of transport that offers itself t0 “ IMPORTANCE OF ROADS IN INDIA’ 401 ignificance of | Planned Road Network It may be said that deficie ’ to the sdbacs tS deficiency in he road development in India has cx . agricul : ltural, commercial and industrial sectors. It is °°” IMPORTANCE OF ROADS IN INDIA s provide roads links between the villages and market cent urban ares aloe dona fet the ecnomie ad ving conor of oe reat ne ‘country as a whole. Overall economic progress can be achieved, only if reasonably adequate transport facilities are made available between the villages and other ditt head quarters and commercial centres. ‘The road networks have also to be supplemented with Express ways to keep pace with the requirement of unimerrupted movement of fast Vehicles along the arterial roads. In general, developing countries have to raise their \tansportation systems toa higher level both in terms of length and quality zo as to meet the demand which is being generated by the development plans. Also road development generates considerable employment potential. It is estimated by Planning Commission and the National Council of Applied Economic Research that Rs. 1,000 crores invested in roads would yield employment for six million persons. thas been shown that a paved surface in reasonably good condition can contribute 10 15 to 40 percent savings in vehicle operation cost, This is very significant from the point of view of energy crisis and conservation of petroleum fuel. Thus itis all the mare important to construct and maintain road pavements in good condition. Revenue from the road transport in India has been much higher than the investment made on road development plans. Out of the estimated revenue of Rs. 10,000 crores from the road sector during the sixth five year plan 1980-85, only Rs. 3439 crores was provided for roads during the plan period. During the seventh plan Rs. 6,000 crores was spent for ross development in the country. There is a great need and considerable scope for bisher investments for development of road transportation in Indi, 1.42. Requirement of Rural Road Development ‘There are 5.76 lakhs villages in India. Of these only about $7 percent of villages with population about 1500, 36.3 percent villages with population 1000 to 1500 and 28 percent villages with population less than 1000 were connected with all-weather roads by 1980-81. Realising the urgency of developing the village roads, these have been treated as a par of minimum needs programme since the fifth five-year plan. It is estimated that Rs. 11,000 crores (as per 1980 cost estimates) will be needed to de all weather motorable road network so that on the average any village is not more yan 1.6 km from a road. During the sixth five year plan (1980-85) out of the provision of Re, 3439 crores for the road development and maintenance, Rs. 1165 crores was set apart for the rural roads under the minimum needs programme, The sites are also sctely engaged in the development of rual reads. For example, a masterplan for rural reads been prepared for Karnataka State at an estimated cost of Rs 949 crores. 1.43 Comparative Study of Road Statistics ‘The road sats give an idea ofthe stage of road development ofa country: The road statistics may be presented onthe bass of area ofthe county othe popstaton, roa spork of tis country has t0 be considerably increased during te derlopreet plans. The poor state of road development in India in te Pes may following reasons : H in the country upto the initiation of 0 .d development of roads in the upto the 9 ie ead Pan the year 1943, Only during the five-year plans since 1951, the development works were speeded UP. 6 INTRODUCTION (Gi) The investment even today’ on the road development programme is mi the revenue from the road transport. A Lowey thay (iii) Poor economic conditions of the vast majority of the population in villa i the owning of private vehicles and discourage the use of transport, °° Prohibiy ‘There has been a noteworthy progress in road development in India durin independent period, especially since the start ofthe five year plans inthe year SPOS: total road length by the year 1951 was only 3,97,600 km, whereas ihe recat: The achieved by 1981 was 15,02,700 km, The density of roads in km per 100 sq, kes te the country by the year 1951 was about 12 which was increased to 21 by ie aTe8 of, 34 by 1972 and 46 by 1981. The road density values of some of the deveen 95h : 4 developing countries by the year 1981 are given in Table 1.1 for comparison, Pe! a8 Table 1.1 Road Density Values in Different Countries by the Year 1981 Road density in km R Country | per 100sq.km area | Country prion ‘Afghanistan 3 [New Zealand 35 [Expr 3 I 67 Nepal S* USA. 68 Australia u Hungary 94 Nigeria 12° Ttaly 97 Pakistan 13° France 146 Malaysia d4* Great Britain 153 Brazil 16 Netherlands 225 Greece 28 Japan 296 "1984 Statistics from World Bank Policy Study. 1,5 SCOPE OF HIGHWAY ENGINEERING The road pavements are generally constructed on small embankments, slightly above the general ground level wherever possil Possible, in order to avoid the difficult drainage and ‘maintenance problems. The term road or roadway thus constructed is therefore termed ‘highway’ and the sci i called ‘Highway Hagar technology dealing with Road Engineering is generally Constructed and maintained. 4 Seuiced sad cian Answers to each of these questions contribute to the In utshelt, it . development, pane eee ee engineering deals with various phase like trafic operation and” its contro, ighway geometric design and location, highay it ‘materi 5 ‘ maintenane, economic considerations, neces sy siement design, constuction a4 jon, : REFERENCES 7 ‘Scope of Highway Engineering, PHASES DETAILS Development, Planning Historical background; Basis for Planning; Master Plan: and Locations Engineering surveys and highway alignment. Highway Design, Road Geometries and their Design; Rigid and Flexible Geometries and Pavements; Design factors and thickness design; Overlay Structures design; Design of drainage system. ‘Traffic Performance Traffic studies analysis; Need for new road links; Traffic and its Control regulation and control; Intersection design and their controls, ‘with signs, signal, istands and markings. Highway Materials and mix design; Highway constuction; Earth ‘work, construction of different types of pavements; Earthen. soi stabilized, water bound macadam, Bituminous surfaces an ‘cement concrete roads; pavement failures; pavement evaluation “Maintenance of pavements and drainage system. Economics, Finance Road user cost and economic analysis of highway projects; ‘and Administration pavement types and maintenance measures; Highway Finance and phasing of exg ss; Administrat Materials, Construction and Maintenance REFERENCES 1. Sinha S. N., ‘Development of Roads in India’. The Indian Concrete Journal, Oct. 1958. 2. ECAFE Report of the Seminar of Low Cost Roads and Soil-Stabilization, ‘New Delhi, 1958 inistry of Information and Broadcasting. ‘Our Roads, Moder India Series No. 11, Publication Division, Government of India. 4. William W. Hay, “An Introduction to Transportation Engineering", Toppan ‘Company Lid., Tokyo. 5. Shivalingaiah, L, “Road Development in Kamataka", Indian Highways, Vol. 9. No. 12, Indian Roads Congress, 1981. 6. Ministry of Shipping ‘and Transport, Roads Wing. “Road Development Plan for India, 1981-2001", Indian Roads Congress, 1984. 7. The World Bank, "Road Deterioration in Developing Countries, Causes and Remedies", A World Bank Policy Study, Washington D.C. ‘ INTRODUCTION PROBLEMS 1, Discuss the role of transportation in the economic and socjy ¥ country Betts 2. What are the diferent modes of transportation ?. Explain cach of them. the specitic fy eto 3. Compare the character Fentueof diferent mode of tanspotti 4 n, 4. Whatare the characteristics of road transport in comparison with o 5. What, in your opinion were the chief causes of ‘Iransportation in India? her system wealected conditions op 6 Explain the role of transportation in rural development in India ? 7. Outline and discuss the scope ofthe highway engineering study, %) Chapter p Highway Development and Planning 2.1 HISTORICAL DEVELOPMENT OF ROAD CONSTRUCTION 2.1.1 Early Development ‘The oldest mode of travel obviously was on the footpaths. Animals were aso used to transport men and materials. Later simple animal drawn vehicles were developed and tis became a common and popular mode of transporation for @ very long period aftr the invention of wheel. This brought up the necessity of providing a hard surfoce for these wheeled vehicles to move on. Such a hard surface is believed to have existed in Mesopotamia in the period about 3500 B.C. The first road on which there is some authentic record is that of Assyrian empire constructed by about 1900 B.C. Only during the period of the Roman empire, roads were constructed in large scale and the earliest construction techniques known are of Roman Roads. The Romans constructed an extensive system of roads radiating in many directions from Rome through the empire mainly for military operations. Hence Romans are considered to be the pioneers in road construction. 2.1.2 Roman Roads Many of the early Roman roads were of elaborate construction. Some ofthese roads are still in existence after over 2000 years. During this period of Roman civilization ‘many roads were built of stone blocks of considerable thickness. The Appian way was built in 312 B.C. extending over $80 km which illustrate the road building technique used by Romans. ‘The main features of the Romans roads are : (i) They were built straight regardless of gradients. Gi). They were built after the soft soil was removed and a hard stratum was reached. i) The total thickness of the construction was as high as 0.75 to 1.2 metres at some be places, even though the magnitude of wheel loads of animal drawn vehicles was very low. 9 10 HIGHWAY DEVELOPMENT AND PLANNING A ypical cross section of Roman road is shown in Fig, 2.1, The conn HISTORICAL DEVELOPMENT OF ROAD CONSTRUCTION u rocedure was as follows : : procedure was Ses metiod of construction was implemented in that cousty in 1778. Can 6 the regime of Napofeon the major develo 7 is “ LARGE STONE SLABS IN LINE 4 PLIME CONCRETE: rajor development of road system in Frarce tok place MORTAR 1D 10 15m THICK 25 TOXOenrmhick cal ctoss section of Tresapuet's road consiruction is piven in Fig. 29 andthe p—— 025m ‘construction steps may be enumerated as below at, SULPING WEARING Sur mses TOTAL Thickness, ToC Sem ce A | Peace +] 075 10 1-2m ssoupeng1cem Sam LARGE FOUNDATION STONES. TW LIME MORTAR 10 70 20¢m_ THICK \sioe oman "Canoe scunoanon stones OW FOU, Hem Prec ‘BROKEN STONES IN LIME MORTAR 2570 40cm THK HARD SOIL STRATUM Fig. 2.2 Typical Cross Section of Tresaguet’s Construction (1775 A.D.) (The subgrade was prepared and a layer of large foundation stones were lid on edge by hand. At the two edges of the pavement large stones were embedded edgewnse to serve as submerged kerb stones. Fig. 2.1 Typical Cross section of Roman Road (i)_A trench of width equal to that of the carriage way was dug along a straight path removing the loose soll from the top. The trench was cut upto a depth until ah stratum was-reached, ya ard Gi) The comers of these heavy foundation stones were hammered and then the interstices filled with smaller stones. Broken stones were packed to a thickness of Gi) One or two layers of large foundation stones were laid in lime mortar a the bottors. hose 8 ca 3 cogs The thickness ofthis bottom layer ranged from 10 to 20 cm. Vertical Kerb stone were placed along the edges of the pavement. (iii) The top wearing course was made of smaller stones and compacted to a thickness of about $ cm at the edges and gradually increased towards the centre, giving a cross (ii) A second layer of lime concrete with large size broken stones mixed in lime mortar slope of | in 45 t0 the surface, to provide surface drainage, id ‘was laid over the bottom course up toa thickness of 25 to 40 em. (iv) The shoulders were also provided cross slope to drain the surface water to the side (iv) “Another layer of lime concrete was laid over this with smaller broken stones mixed. sal in lime mortar to a thickness of 25 to 40 if fs mi lickness of 25 to 40 em, or even more if necessary. sd Metis if Gnetractiow John Metcalf (1717-1810) was engaged on road construction works in Egland during the period when Tresaguet was working in France. He apparently followed the recommendations of Robert Phillips whose paper was presented in Royal Society, Metcalf was responsible for the construction of about 290 km of road in the northern region of England. As Metcalf was blind, much of his work was not recorded. (¥) The wearing course consisting of dressed large stone blocks set in lime mortar was!” provided at the top. The thickness of these blocks also varied from 10 to 15 em. Obviously the above construction should have been much stronger than’ what vas’ required for the animal drawn carts in those days. ‘The enormous cost of construction cannot be justified at all, i this technique is compared with the modern trend of pavement design based on more scientific approaches. 2.1.5 Telford Construction Thomas Telford (1757-1834) began his work in early 19th century. He was the founder of the Institution of Civil Engineers at London. He also believed in using heavy foundation stones above the soil subgrade in order to keep the road foundation fra. He sisted on providing a definite cross slope for top surface of the pavement by varying the thickness of foundation stones, A typical cross section of Telford's construction by the year 1803 is shown in Fig. 23. 21.3 Tresaguet Construction ‘After the fall of the Roman empire, their technique of road construction did not in other countries. Until the eighteenth century there is no evidence of any new. oad construction method, except the older concept o| thick construction of beads as the Roman did, mor a ‘The construction steps are given below : (i) A level subgrade was prepared to designed width af about 9 meters. Large i i laid with hand with their i foundation stones of thickness 17 to 22 cm were ; - largest face down so as to be laid in a stable position. The stones of lesser thickness } HIGHWAY DEVELOPMENT AND PLANNING HISTORICAL DEVELOPMENT OF ROAD CONSTRUCTION Ping WEARING SURFACE Wes) Cem TIC BROKEN STONE. a Sem THICK = fr BROKEN STONE passin 3T5ery SEVE, Wem tice t Re, AMONTAC SOBER? TOLiOSTION STONES CF -aNGuLan Beech SURFACE course wn STONES PASSING Ler} SIEVE, Sem THC. [BROKEN STONE IN LIME WoRTAR WARING S7FATTOZem) STINE Temeeee oem THick [BROKEN SIONES Pasting Sem SIEVE em Te | ; COMPACIED sumoRApE Fig. 2.3 Typical Cross Section of Telford's Construction (1803 AD) WITH CROSS SLOFE 10136; (17 em) were placed towards the edges and stones of increasi thickness were tig Fig. 24 Typical Cross Section of Macadam’s Construction (1827 A.D.) towards the centre. At the centre the largest stones of approximate thickn, | ‘were used such that these foundation stones of varying thickness Provide the oot Oy tis eel achat eng Soundation sons ae erat slope designed by Telford. | ay to be mm layer of construction. He realised thatthe subgrade being the lowest portion of the pavement should be prepared properly and ‘kept drained so as to carry the load transmitted through the pavement. Compacted layer of smaller size broken stones placed at the botiom, according to Macadam, could replace, with advantage, the heavy foundation stones. (ii) The interstices between foundation stones were fil led with smaller chipping and properly beaten down, See. () The central portion of about 5.5 metre width was covered with two layers of angular-broken stones to compacted thickness of 10 and $ em. ‘These layers were intially rammed and later allowed to be compacted under the traffic and get consolidated by the rains, ii) Though the total thickness of construction, was less than previous methods, this technique could serve the purpose in a better way, due to better load dispersion characteristics of compacted broken stone aggregates of smaller sizes. (iv) The size of broken stones for the top layer was decided based on the stability under animal drawn vehicles, The pavement surface was also prepared with a eross slope of | in 36 for drainage of surface water. (¥) A certain width of the pavement towards the ed; broken stones, 15 em thick, sometimes in lit stones so as to provide lateral stability. iges was constructed by compacted ime mortar instead of using the kerb ‘Macadams method is the first method based on scientific thinking. It was realised that the stresses due to wheel load of traffic gets decreased atthe lover layers of the pavement and therefore it is not necessary to provide large and strong boulder stones as foundation or soling course at the lowest layer of the pavement. This method became very popular far and wide. Various subsequent improved methods were based on Macadam's construction and some of the methods stil in use are known after his name, such as water bound macadam, penetration macadam and bituminous macadam constructions (i) A binding layer of wearing course 4 cm thick was constructed on the it sravel. The finished surface had a cross slope of about 1 in 45, ee Telford proposed to provide cross usually laid below the foundation lev Water to percolate from subgrade. Grains at intervals of about 90 meters. They were el as the interstices were large enough to allow the ‘0p to the bottom of the construction and thus soften the level ‘The construction steps are : 2.1.6 Macadam Construction (i). Subgrade is compacted and prepared with a cross slope of | in 36 upto a desired ‘width (about 9 metres). (i) Broken stones of a strong variety, all passing through 5 cm size sieve were ‘compacted to a uniform thickness of 10 cm, (iii) The second layer of strong broken stones of size 3.75 cm was compacted to thickness of 10 cm, i kness of iv) The top layer consisted of stones of size less than 2 em compacted to a thickness 0 Oy Thea coed fished so tal be io slope of paventent surface was also 4 in 36. ‘The Macadam and Telford methods of coastrection differ considerably though both the methods were put forward in the early nineteenth century. HIGHWAY DEVELOPMENT AND PLANNING i “The two methods have been compared here : ‘Macadam method lope The subgrade was given a cross 5 Sie 38 to felite subgrade drainage. _____HskH | (i) The bottom layer of pavement or the ‘The subgrade was hence subgrade proper. Heavy foundation st ny Kept horizontal > drainage 1 OES Of vane subsbase course consid of broken | sizes, about 17 em towards | stones of tess than 5 cm size to | and 22 om towards the sot CW | uniform thickness equal to 10 cm | hand packed and Prepared to seme only sub-base course, es Base and surface courses consisted of broken stones of smaller sizes to ‘compacted thickness of 10 and 5 em respectively and the top surface was iven a cross slope of | in 36. (iv) The total thickness of pavement construction was kept uniform from edge to centre to a minimum value of only 25 em. a Two layers of broken compacted over the foun before laying the weari cm thick with a cross sto ‘The total thickness of consinici varied from about 35 cm at to about 4l cm at the centre, Gi) stones yey dation stones 8 course, 4 PE OF Li 2.1.7 Further Developments Macadam's method of construction gained recognition as a scientific method construction and hence was adopted by various countries with slight modifications, One Of the most popular methods which is even now prevalent in many countries is the wate howd macadam (WBM) construction, known after Macadam’s technique. In this rahod {fe broken stones ofthe base course and surface cours, if any, are bound by the sone lust in presence of moisture and hence the name. WBM roads are in use in Indie both a finished pavement surface for minos lopted in our country include the bitum ion macadam. been given in the Chapter, ‘Highway Construction’ WBM roads several dust palliatives inet varying degrees of success, could not last long, lading heavy oils and bituminous materials were tied on | the edge ‘The details of the construction methods heve HIGHWAY DEVELOPMENT IN INDIA ‘The use of cement concrete for 1 bituminous mixes. The cement con loads expected on the roads even i long service lite. extensively used in our country at present. Utilisation of locally available and the struction. In this respect there is good Hon and other low cost pavement materials There are $2268 techniques of sil stabilization which have been explained ina later chapter ofthe book. ‘The choice ofthe method of stabilization depends on several factors such ws the soit Pe, availability of stabilizers, climatic conditions, the component of pavement which is being constructed and the traffic. 2.2 HIGHWAY DEVELOPMENT IN INDIA 2.2.1 Roads in Ancient India ‘The excavations of Mohenjo-daro and Harappa have revealed the existence of roads in {India as early as 25 to 35 centuries B.C. Old records reveal that in early periods the roads were considered indispensable for administrative and military purposes. The ancient ") Scriptures refer to the existence of roads during the Aryan period inthe fourth century BC. Kautilya the fist prime minister of Emperor Chandra Gupta Maurya, laid down the rules in the literary piece titled ‘Arthasasira. Rules have been mentioned bout |} regulating the depth of roads for various purposes and for different kinds of traffic. “Mention has been made regarding the punishment for obstructing roads. In the beginning +} of fifth century A.D. emperor Ashoka had improved the roads and the facilities for the travelers. | 2.2.2 Roads in Mughal Period During the Pathan and Mughal periods, the roads of India were greatly improved. Some of the highways either built or maintained by Mughals received great appreciation from the foreign visitors who visited india during that periods. Roads were built running from North-West to the Easter areas through the Gangetic plains, linking also the coastal and central parts. 2.2.3 Roads in Nineteenth Century At the beginning of British rule, the conditions of roads deteriorated. ‘The economic and political shifts caused damage to a great extent in the maintenance of the inl transportation, The fall of Mughal empire led therefore to the seant attention to communication. Prior to the introduction of railways, a number of trunk erie metalled and bridges were provided. this was mainly. done fon the remains: ose _ which existed, under the supervision of the British Military Engineers. In fact these ‘connected important military and business centres. 16 HIGHWAY DEVELOPMENT AND PLANNING Military maintenance was not quite adequate and in 1865 was Governor-General formed the Public Works Department form that exists today. The construction of the Grand Trunk Ro, new department. Immediately with the development of Government was shifted from road development except for the railway was gaining the privileges Lord Dathousie, when he in more or less the same tad was undertaken by this railways, attention of the Providing feeder roads and 2.2.4 Jayakar Committee and the Recommendations ‘After te first World War, motor Vehicles using the roads increased a better road network which can carry both bullock car traffic and mar, bites existing roads when not capable to withstand the mixed tafe conditions. A resslunes yas psd by both Chambers of the Indian Legislature 1927 forthe appomme soe Cormmitee to examine and report on the question of road development in India, ia fesporse 10 the resolution, Indian Road Development committee was appointed by tke government with M4.R. Jayakar as Chairman, in 1927, The Jayakar Committee submitted its report by the year 1928. ‘The most important recommendations made by the committee are : 7 (The road development in the country should be considered as a national interest as this has become beyond the eapacity of provincial governments and local bodies, (0) An extra tax should be levied on petrol from the road users to develop a road development fund called Central Road Fund, (iid A semi-official technical body should be formed to poo! technical know how from vais Darts of the country and to act as an advisory body on various aspects of roads, (0) A research organisation should be instituted to carry out research and developmen relopment work and to.be available for consultations, » Most of the recommendations of the Jayakar Committee were accepted by the Goverment, and the major items were implemented subsequently. ‘The Central Road pain was formed by the year 1929, the semb-oficial technical body called the Indian Rods Shonen was formed in 1934 and the Central Road Research Institute was started Central Road Fund Based on the authority of a resolutio Road Fund (C.R.F.) was formed on 1 ‘ - , road experi id research on road and bridge projects of ‘periments an alloted by the Central Govemnient t0 the various ete acul' a rainare tin OF fevenue collected, The accounts of the Gon 1 Read Funda capentined by the Accountant General of Central Revenues iat te cnaret oy BE Spree 8 exercised by the Roads Wing of Ministry of Tent be sal Ree of collection of the levy towards the Ci Sis Sf ables of te © CRF has been revised in order to augment the HIGHWAY DEVELOPMENT IN INDIA Indian Roads Congress n ‘At the instance of central government a semi-fficial i Roads Congress (IRC) was formed in 1934, ‘This, it ae renee coe one recommendations made by the Jayakar Committee. ‘The indian Reads Congress was constituted to provide a forum for regular ‘Pooling of experience and ideas on all matters affecting the planning, construction and maintenance of roads in India, to recommend standard specifications and to provide a platform for the expression ‘of professional opinion on matters relating to road engineering including such questions 2s those of organisation and administration. The IRC has played important role in the formulation of the three 20-year road development plans in India. Now the Indian Roads Congress has become an active body of national importance controlling specifications, standardisation and recommendations on materials, design and construction of roads and bridges. The IRC publishes journals, research publications, standards specifications guidelines and ‘other special publications on various aspect of Highway Engineering, ‘The technical activities of the IRC are mainly carried out by the Highway research Board and several committees and subcommittees consisting of experts in each subject. ‘The IRC works in close collaboration with Roads Wing of the Ministry of Surface Transport, Government of India. Economic depression during the thirties of this century delayed the road development programmes. During this period the share from the Central Road Fund was almost the only source for highway financing. During the second world war intensive efforts were made to develop the road net work essential required for strategic considerations, ‘These projects were substantially supported by defence services funds. ‘Affer the second world war, there was a revolution in respect of automobiles using the roads in our country and a large number of military vehicles started plying on the roads. ‘Thus the road development could not cope up with rapid increase in road vehicles end so the existing roads started deteriorating fast. The need for proper highway plannit g was urgently felt by this time by the authorities. Motor Vehicle Act In 1939 the Motor Vehicles Aet was brought into effect by Government of India to regulate the road traffic in the orm of tra: awsordinanes and regulations. Te tee phases primarily covered are control of the driver, vehicle ownership and vehicle ‘operation on roads and in traffic stream. The Motor Vehicle Act has been appended with ‘several ordinances subsequently. The Motor Vehicles Act has been revised in the year 1988 2.2.5 Nagpur Road Conference je Engi the states and provinces was convened in ference of the Chief Engineers of all the stat ; iis by the Government of india t agp nine of te ii Ronis Cogs i vt lan for the count . nase the fit od evelopment pln forthe county sole, ss = ging ond ern Fo Pan hai lan, popularly known as * dal development pla Poe a eet yar development pose fr tepid 14262 ae iv carr the end of this plan the target road length aimed a ras . ‘square km area of the country, 18 HIGHWAY DEVELOPMENT AND PLANNING ring the first and second five-year plan periods (1951-56 5 eat nragranais systematic and hence the Nagh Dy the development programme also was systematic an e Nagpur plan targeeg 8d Lito, road length wa: .d about two years ahead, in 1961 meeting of the Chief Engineers of the central and state Governments w, consider the future road development programme, A committee was appoin conve the Second Twentyear Road Development plan starting from the year Toe sto ‘0 prepare Central Road Research Institute Inthe year 1950 the Central Road Research Institute (CRRI) was st y h tar for research in various aspect of highway engineering. It may be indicaed tet New Delhi recommendations of Jayakar Committee report was to set up a central o; One of the: research and dissemination of information eanisation foe The CRRI is one of the national laboratories of the Council of Scier : tii P Research; the institute is mainly engaged in applied research and offers tects usta N i a technical adv 'oslate governments and the industries on various problems concerning rode National Highway Act tn 1956 the National Highway Act was passed, The main features of the act ae (i) the responsibility of development and mai (NH) lintenance of th ic ig be provisionally taken by the central ‘government, ss bl e (ii) the central government to be empower red to declare any other hi omit any of the existing national highvoys one a on MgbWaY 25 NH orto ways from the list. 2.2.6 Second Twenty Year Road Development Plan 1961-81 The second twenty a seine Talal ond development plan forthe period 1961-81 was initiated by forward to the Coma) nooo at the meeting ofthe Chief Engineers and the same was Bombay Road Plan The eeemment._ This road development plan is also known as Bombay Hon lan gave due consideration to the development that are taking ment tha! have to take place in our country in various felds during the ending 1980-8) was en "a. An outlay of Rs. 5, level, The constr S382 for this second tw .200 crores for the period During the third five l ieunk eee pele eee 1961-66, the annual plans 1966-69 and the pace The al ea en ond evelopment in India continued at a steady '¢ density of road sara re length being das km per 100 4 = ia, te he? an immediate need to prepare ae Nt REM Development Plan1961-81.. THIS ourtry by then However, dus ey thd long term roa develouen pa forthe 1974-78, annual Plans 1978.80 man ing Policies duri : , he third Jong term road devel eth vive ys 35th pean o SS development plan got delay. ° 1980-85, the preparation of He The Third Tweng wenty Ye repared by a comnite ‘ongress and also at the Iso at th ‘Council of the Indian Roads oF the Country in the year 1984. HIGHWAY DEVELOPMENT IN INDIA 19 Highway Research Board ‘The Highway Research Board of the Indian Roads Congress was set up in 1973 with a view to give proper direction and guidance to road research activites in India. The board is expected to act as a national body for co-ordination and promotion of highway research. The Highway Research Board (HRB) has recommended suitable financial allocation of research by central and state governments and has chosen high priority research schemes for being taken up first. ‘The objective of IRC Highway Research Board are : () Toascertain the nature and extent of research required. (ii) To correlate research information from various organisations in India and broad with a view to exchange publications and information on roads. (ii). "To co-ordinate and conduct correlation services. (iv) To collect and disseminate results on research (¥) To channelise consultative services ‘There are three technical committees of the HRB for (i) identification, monitoring and research application (ji) road research evaluation and dissemination and (iii) bridge research, evaluation and dissemination, 2.2.7 National Transport Policy Committee ‘The Government of India appointed the National Transport Policy Committee (NTPC) in the year 1978 to prepare a comprehensive national transport policy for the country for the next decade or so, keeping in view the objectives and priorities set out in the five year plans. The NTPC report was made available in the year 1980 and many of the major recommendations of this report have been accepted by the Govemument of India. Some of the important recommendations of the NTPC report relate to the liberalisation of the transport sector, inclusion of transport in the priority sector, optimal in-er-modal mix between railway and road transport based on resource-cost consideration and energy conservation. A number of suggestions were made on the road development, these include the need to take into account the requirements of roads in rural, hilly and tribal areas in the next perspective road development plans, strengthening of National Highway system, increase in funds for the maintenance of roads and to connect all the villages with all-weather low-cost roads within next twenty years. Separate recommendations were also made for various factors connected with the development and growth of road transport by the year 2001. 2.2.8 Third Twenty Year Road Development Plan 1981-2001 ‘The Third Twenty Year Road Development plan 1981-2001 was prepared by the Road Wing of the Ministry of Shipping and Transport with the active co-operation from a number of organisations and experts in the field of Highway Engineering and ‘Transportation, This document was released during the 45th Annual Session and the Golden Jubilee celebrations of the Indian Road Congress in February 1985 at Lucknow. ‘Therefore this road development, plan for 1981-2001 is also called ‘Lucknow Road Plan’. ‘This plan has been prepared keeping in view the growth pattern envisaged in various ts Which were ji ; © Riven vo tangent Sets ionmenal Tato incceasing the total road length ¢ vt | oe sthe year 1981 1027.02.00 km yr i om 46 km per 1009, kn ine Sty theyear 200). As the developntent of National ied 6 566 | - oY vo twenty year road development pl ti get length of NH tobe competed ty Pe swear ine 7 [1977-78 329 Review of Highway Development in India after Independence (ageoest [sox [asx ed independence on 1Sth August 1947, the total road fe 2.3 NECESSITY OF HIGHWAY PLANNING cosrmy was 3.88226 kam withthe density of road length working out to about, (0053, bm area, out of this the Length of surfaced roads was only about 36, te fre rae of ead development i th country has improved considera sd length in the country increased from 3,97,600 km € km by the year 1980481 in the year 1950 In the present era planning is considered as a pre-requisite before mnempting acy development programme. This is particularly true for any engineering work, = planning is the basic requirement for any new project or an expansion programme Thus highway planning is also a basic need for highway development. Particularly planning is of great importance when the funds available are limited whereas the total requirement fs mach higher. This is actually the problem in all developing countries like India as the bes ion of available funds has to be made in a systematic and planned way ‘The objects of highway planning are briefly given below : ‘There has also been a tremendous pressure on road transportati rt ; ie aa in umber of ad vies and considerable increase in feight oa pri ‘amied by roads during the above thirty year period. The total renewal of pavement layer: ae L. number of motor ‘ i onstage ancora am ates mama ° Tahaan ice ee af ic on roads increased from about 5.5 to over 104 bill ‘and the vehicle operation costs are to be given due consideration. mca ds (Gi) To arrive at the road system and the lengths of different categories of roads which rool pt nha coe Son ‘mutant oman (ii) To fix up date wise priorities for development of each road link based on utility as crores during ° theabove tiny year period, the main criterion for phasing the road development programme. ‘The growth of total road iv) * k (iv) To plan for future requirements and improvements of roads in view of anticipated & ahr he pera face) fd length andthe length of surfaced os | developments SEremu from road transport due to taxatyoe enone given in Table 2.1. The 1 (¥) Towork out financing system. Seclpmet and maintenance expendi a total expenditure on roads incl eisai istengSREtIe Iomay be seen that only goose te centre and the sates a 24 CLASSIFICATION OF ROADS 8 spent on development and eaten @ fraction of the revenue from road tran 2A Types of Roads ‘enance of roads, 4 é ae Than development of National High ‘The different types of roads are classified into two categories, depending on whether Ise edge int tec il Bas not been atthe dese they can be used during different seasons ofthe year: deve ty Road Plan in Toad length by the end of Nagpur Road Pl (i) All-weather roads and lopment 198) : Plans, the length of Nit 0d tHe targets ofthe two twenty yest fora hieh propor e* 24 fll short of both the plan targe's- AS He Consiuctionof ene OF defame it the country, provision has Ho OE tional length of NE ten existing NH system and also for BY for the growing traffic needs upto We (ii) Fair-weather roads. All weather roads are those which are negotiable during. all ‘weather, except at major river crossing: where interruption to traffic is permissible 1" 8 (ii) unsurfaced roads which are not Surfacing, “The ends ih ae, Provided with bituminous or cement 4 with bituminous surfacing are also called oct ped § ch roads. 2.4.2 Methods of Classification of Roads ‘The roads are generally classified on the following basis : (a) Traffic volume (b) Load transported or tonnage (c) Location and function ‘The classification based on traffic volume or tonn: eran 2 ic age have been arbitrarily fixed different agencies and there may not be a common agreement regarding the Tine foreach of classification group. Based on the traffic volume, the roads are classified as heavy, medium and light traffic roads. ‘These terms are relative and so the limits under each class Should be clearly defined and expressed as vehicles per day etc. Likewise the Slassification based on load or tonnage is also relative and the roads may be classified 2s ‘lass I, Il ete. or class A, B etc. and the limits may be expressed as tonnes per day. ‘The classification based on location and function should therefore be a more acceptable classification for a country as they may be defined-clearly. The Nagpur Road Plan classified the roads in India based on location ‘and function into following five categories and described in section 2.4.3. (i) National Highways (NH) (ii) State Highways (SH) i) Major District Roads (MDR) (iv) Other District Roads (ODR) and (») Village Roads (VR) 2.4.3 Classification of Roads by Nagpur Road Plan a (i) National Highways (NH) are main highways running through the ey a OE ighways an sof large § reauth of India, connecting major. ports, orci highways, capt re, ia fystrial and tourist centres including rosds required for strategic movements 7 st . tefande of India: CLASSIFICATION OF ROADS. a twas agreed that a fst step Natlonal Tails should be constweied the Centre and that ner these should be converted into roads to sit the traffic conditions, It as Aheciied that national highways should be ‘the frame on which the entire road spec nication shoul be based and that these highways may Mt necessarily be of same Specification, but they must give an ‘uninterrupted road communication through out India Pe ould connect the entire road network. ‘AIL the national Righways are assigned the respective numbers The highway connecting Delli-Ambala-Amritsar 8 denoted as NH-1. where bifurcation of this, Fighway beyond -Jalandar to Srinagar and Urt is denoted NH-I-A. The highway eng, Maduri and Rameswaram is NH-49 and Bombay-ARra road is NH-3. A map Showing National Highways is given in Plate [ Gi) Stave Highways (SH) are atrial ronds of a state, connect with the national pighoays of adjacem sae, distriet head quaers and import sis ‘within the state and aie asthe main arteries for traffic to and from district roads- “These highways are considered as main arteries of commerce by roads within 2 state or ‘a similar geographical unit. In some places they may even oP heavier traffic than some eae ronal highways bu this will nt alter their designation or fencton. ‘The NH and Gir have the same design speed and geometric design specifications. (ii). Major District Roads (MDR) are important roads within © district serving areas of production and markets and connecting those with each othe) of with the main fighways of @ dist. The MDR has lower speed and geometric sian specifications than NHVSH. (iv), Other District Roads (ODR) are roads serving rural areas of production and providing the with outlet to market cenzes, taluk head quarters biock development rad quarters or other main roads. These are of lower design specifications than MDR. (§) Tillage Roads (VR) are roads connecting villages or groups of villages with each ‘other to the nearest road of a higher category. It-was specified that these village roads should be in essence farm tracks, but it was desited that the prevalent practice of leaving such tracks to develop and maintain by themselves should be replaced by a plan for a designed and regulated system. ‘A general note was assigned by the Nagpur Road Conference regarding the economics ‘ofroad construction that all roads of whatever type oF class, should be so constructed that fhaintenance and capital costs over a period of 20 years will be minimum. | The responsibility of construction and maintenance of national highways was decided to be ‘withthe central government; it was stated that "Centre should select the national highways ‘and trails, accord priorities and pay forall construction and maintenance”. 244 Modified Classi 1981-2001 ‘The roads in the country are now classified into three classes, for the purpose of transport planning, functional identification, earmarking administrative jurisdietions and assigning prierties on a road net work viz; (9) Primary system (ii) Secondary system and (iii) Tertiary system or rural roads, jon of Road System by Third Road Development Plan, HIGHWAY DEVELOPMENT AND PLA Prima a (a) Expressway and (National Highways (NH) system consists OF Wo category Expressways are a se WS are a separate class of hy ; Standards and are meant as through ra anna With supeto arate te 10 be provided with divided "8, EY high fhe ROAD PATTERNS 5 ‘These have been shown in Fig. 2.5 a,b,c, d,e& £, am nou BS 3 » 7 | SE Separations at cross roads and fencin ignaae® va i thicles. Expressways may be owned! by oe highways shal peed lepending on whether the route is 4 National High = vent ars == The Secondary system consists of two categories os Hlth, (a) State Highways (SH) and i a (©) Major District Reads (MDRy The Tertiary system are Tural roads and is : (a) Other District Road (ODR} Met cont ot Sa (6) Village Roads (VR) : The urban roads, other than expressways, are classified as: @ Arterial roads (ii) Sub-arterial roads (iii) Collector streets and (¥) Local streets imaril ‘on a contis Areerials and sub-artefals are streets primarily for tous tations om routey. but the sub-arterials have a lower level of matic mois he ei ‘ Collector streets provide access to arterial streets h Spat y from and to local streets which provide access to abuting Prope ris iS 2.5 ROAD PATTERN! i ows : ‘The various road patterns may be classified as fo (a) Rectangular or block patter (b) Radial or star and block pattem {c) Radial or sar and circular pattern Rectingular or block pattern (b) Radial or star and block pattern “S = O aos Co (©) Radial or star and circular pattern fact ‘finding surveys. The planning based on the factual data may be considered sound. ie inet for planning and these studies ‘The factual studies point to an intelligent approach relied should be carried out if the highway programme is to be protected from inconsistent shor sighted policies. 28 HIGHWAY DEVELOPMENT AND PLANNING Te planing sures consist of the flowing stiles; he deals shown in ig 4 (a) Economic studies , PREPARATION OF PLANS » (b) Financial studies (i) Sources of income and estimated revenue from taxation on road wanspont (c) Traffic of road use studies (ii) Living standards (@) Engineering studies (iil) Resources at local level, toll taxes, vehicle repstation and fines (iv) Future trends in financial aspects. PLANNING SURVEYS Economic Population; its distribution and cla (©) Traffic or Road Use Studies , nan , studies population growth Listing of ogricain stot, JT All the details of the existing traffic, their volume and pattern of flow should be known, wericultural and indus | before any improvement could be planned. Traffic surveys should be carried out in the development, their future trends-classifica of these details; other activities viz. Income wnt banking, post office etc. e whole area and on selected routes and locations in order to collect the following particulars : : i raffic vol in vehicles day, il % i Financial DataCollested Source of income, Revenue ftom taxation on Oe ee Aba cl eines Studies lust Confirm To transport. Living standard, future trends-detals 9 ‘Adequacy, — vehi, ce ‘Asoo ‘chicle registration, Court fees and Local taxes ete, Origin and destination studies (iii) Traffic flow patterns Teapiee Availability, Traffic volume, traffic flow pattems: O and D studies. pera Accessibility Mass transportation facilities, Accidents - their (iv) Mass transportation facilities ui tre i s Food ase of vehicular traffic, passenger trips aif (0) Accident their cost analysis and causes Engineering iad Sesion “and “at hes (vi) Future trend and growth in traffic volume and goods traffic; trend in traffic panem + alignment studies, Cl ion q studies “Types of roads in use: Maintenance pr FS Leet Fv) Growth of passenger trips andthe trend inthe choice of modes. (@) Engineering Studies {All details of the topography, soil and other problems such as drainage, construccion ‘and maintenance problems should be investigated before a scientific plan or programme is, suggested. The studies include : (@ Topographic surveys ii) Soll surveys Location and classification of existing roads (iv) Estimation of possible developments in all aspects due to the proposed highway development. (v) Road life studies (vi) Traffic-studies-Origin and Destination studies (vii) Special problems in drainage, construction and maintenance of roads. ‘Thus all the above studies for collecting the factual data for highway planning are known as fact finding surveys. The details collected are tabulated and plotted on the ‘maps of the area under planning, topography studies; Road life studies. Fig. 2.7 Details of Planning Surveys (@) Economic Studies classified in groups. (ii) Trend, ‘of population ‘growth, Agricultural in r "ald industrial products and their listing in classified groups, area Wis | (iv) Industrial and agricultural development and (W) Existing fa (vi) Per capita income, (b) Financial Studies ‘The financial studies are te ‘essential to 5 3 income and the manner in wh; study th i b be collected eleacn' * Mbich funds for the ® various financial aspects like SoUress . Project may be mobilized. The detail future trends, 2.7 PREPARATION OF PLANS alignment and other details ofthe road development programme, daring the fact finding surveys should be presented in the form rawings are prepared showing the various details of the area as Before finalising the the information collected of plans. Usually four 4 listed below. 30 HIGHWAY DEVELOPMENT AND PLANNING, PLANT General area plan showing almost all existing detai tating road network and drainage structure, river, cop’! EREPARAIIOB DE HISATRTEAN AND 18 ERASING, u towns and villages with the population; commereint lake Lf some target of road length has been fixed for the hs ome target ofr xed for the county on the basis of aren or agricultural activities are also shown in this map, population and production or both, the same may be taken as « guide for deciding the foal length ofthe road sytem in each lematve propos. In in the gt rod Tengths were decided by the Nagpur road plan formulae for the period 193-63 and by the second 20-year road development plan for the period 1961-81, as mentioned sa Ar. 228 ‘and 2.2.6 and also as described in Art. 2.10, These plan formulae for finding the road length are based on population and areas divide into different categories, depending on development achieved. “The next step is to compare the various alternate proposals of road systems in hand and to select the one which may be considered as best under the plan period This i a {Quite difficult problem as the decision has to be a balanced one. In ariving at the best road system out of the alternate proposals, it is desirable to make use of the concept of “saturation system based on U. S. system of highway planning. ‘After deciding the optimum road length for a plan period the final step is the phasing of the road development plan by fixing up the priorities for the construction of different road links, Saturation system In this system the optimum road length is calculated for area, based on the concept of obtaining maximum utility per wait length of road. Hence ths system is ealled saturation “yistem or maximum utility system. ‘The factors which are taken for obtaining the wility per unit length of road are : (0) Population served by the road network (b) Productivity served by the net work i) Agricultural products (Gi) Industrial products “The following steps may be followed to find the road net work having maximum utility per unit length by the saturation system. ‘Step (i) Population units. Since the area under consideration may consist of villases and towns with different populations, itis required to group these into some convenient population ranges and to assign some reasonable values of utility units to each range of populations served. For example, villages having population range between 1001 and +5000 may be grouped together and be assigned one utility unit pe village. Similarly the ‘various villages and towns may be grouped in different population ranges and be assigned suitable utility units as given below : PLANT This plan includes the distribution of population groups j the categories made in the appropriate plan. #°¢0rdangg PLAN IIL — This plan shows the locations of places with their productivity. ‘ive quanti PLANIV This plan shows the existing road network with r ‘i traffic lines ebuaned from Origin and Destination studies of eon? received from different sources may also be shown in this oe : 2.8 INTERPRETATION OF PLANNING SURVEYS The various details collected from the planning surveys and presented in the ‘eal plans should be interpreted in a scientific way before arrivis development programme. where svi Od The data collected could be inte ae preted and used for the following imponag | (i) To arrive at the road net-work, out of the sever meee ral alternate possible systems, (ii) To fix up priority of the construction it i Projects, so as to phase the el plan ofan area in different period of time such as five year plans sioeapal (iii) To assess the actual road use by i y studying the traffic flow 7 __ therfore sow areas of congestion which ned immediate a (iv) Based on the traffic type and inten Pavement and cross drainage structure data and the past ‘experience, (vy) Ce i ) Compson of he cress may be obtained on the basis of thir economic at Th refore suggest the arcas of immediate need for 1H ity and the performance of existing » A new structure may be designed using. (vi) On statis is in Statistical basis, the data obtained infact finding surveys may be analysed fort future trends in development weal y pment of an area i i i hich in tum generate higher wate eenace rent in productivity and pop future planning, ‘This information may be useful in ity unit = 025 Population less than $00, ut 501 to 1000, utility unit = 0.50 1001 t0 2000, utility unit = 1.00 2001 to $000, utility unit = 2,00 te. yas and villages sith From plan I of population prepared earlier, the number of towns and ith pepsin ranges served by each oad system is found and then converted in sy ts served by each road. Thus the total number of unis based on population ‘obtained for each road system proposed. proposed. In each Is and improvement of isting roads Bt cee Sen a Possible changes in future are kept in vieg Pe) a4 all other details, both existiNe 2 HIGHWAY DEVELOPMENT AND PLANNING Step (i) Productivity units. The total agricultural and industri eae ee uae shoukl be worked out. The productivity seed appropriate values of utility units per unit weight. For example one Spricultural products may be considered equivalent (0 one unit, Simard (mn proiucts may also be assigned some suitable utility units per unit weight, Higa lady values than te je may be Sy faw materials like ores etc. may be assigned lower products, From plan Ill showing the produets in the area, the total prodiey Served by each road system may be estimated. ty Step (ii) Utility. The total utility units of each road system is found : population units and productivity units. The total units are divided by qe 9{%8 Tength of each system to obtain the utility rate per unit length, ‘ Each road system having different layout and length would show different ! ; Pian - 0 utility per unit length. The proposal which gives maximum utility per unit k Plan - P a tS oun chosen as the final layout with optimum road length, based on maximum uty Road Lanai rae ree eas oe saturation system, Prnguctivity unis = 200 Productivity units = 270 This method is useful not only to choose the best layout from the altemate but also to phase the road development plan The only limitation ofthe sen possible variation in the relative weightages assigned to population and productiy Ig SA possible to give a relatively higher weightage either to the population or to acetsane | a of products. A sound judgment based on professional skill and experience shouldbe | 7 helpful in providing balanced weightages for arriving atthe optimum road lengh best road system with maximum utility per unit length of road. 8 Phasing of Road Programme Xi The road net-work to be constructed and improved in the plan period is decided whi finalising the master plan of the road development project. The plan period may be ofa | ens long term, like that of the 20-year road plan or of shorter period like five year plans. whatever be the plan period, it is necessary to phase the road development prog from financial considerations. In other words, it is necessary to fix up the priorities the construction of each link of the road net work development programme 10 which link should be taken up first and which oné the next and so on. The phasing also be done for each annual budget year by fixing up the priorities. Here again the priority for each road link may be fixed scientifically based ‘maximum utility. ‘The utility per unit length of road based on population and produ for each road is worked out. Then each link of the net work is listed in the ord Priority based on utility per unit length of the road. i Road Length = S50 km Population units» 221 Productivity units = 315 Productivity units = Existing roads in the area Proposed roads in the Fig, 2.8 Example 2.1 Work out the utility per unit length for each of the systems and indicate which of the plans yield the maximum utility based on saturation system, Assume utility units as given below : Example 2.1 : (a) Population unit ‘An imaginary area with existing roads is shown in Fig. 2.8. There are four alters 1001-2000 0.25 Proposals P, Q, R and S with different road length by adding extra road links to thee 50 rons ii 1B low 2001-5000 ts * the population and products served are given belo¥ rae asi soot-10000 | 1.00 al agua sal length km sa sagpr il sepulation nage industrial proses oie a [agosto StL > 0, vot (b) Productivity uit =o 20 90 7 $ 1000 tonnes 10 Ho a a aia Solution The problem is solved and values obtained are tabulated j i Table thor the plan proposal Q (with total road length of 400 km) has a Wray leath of rand (based on population and productiviy), equ 8 wg optimum road length in this area is 400 km, 19 U.125 there Table 2.2 Solution of Example 2,1 (Meth of arving at optima rod length based on saturation Wo. of towns & vilages served with bi) Road | Road popilation Total units al | kath wepeat| km | WOF | 200%] S001. 7> 70000) sepa] pea 2o00_| 5000_| 10000 latin | tivity # | 300 | 160% 025| 60x05 [30x1] 6x25] 125 | 209 a 400 | 200x 0.25| 90x05 |60x1| «25 | 175 | 279 a 500_| 240 x 0.25 | 1100.5 | 70x 1] 10%25| 210 | 315 $s 550_| 248 0.25 | 1120.5 | 73x1[12x25] 221 | 335 Example 2.2 ‘Three new roads A, B and C are to be completed in a district duri period. Work out the order of priority for phasing the me utility principle, from the data given below, Adopt uti with population range 2000 to 5000, or for products/100t of industrial products. Assume any ot! ing a five year ty unit oF 1.0 for serving a vil ram by xin catering for 1000t of agrcuhurat HIGHWAY PLANNING IN INDIA 3s 2.10 HIGHWAY PLANNING IN INDIA. ‘The first attempt for proper planning of the highway develo oma ong ter basis was made atthe Nagpur Conference in 1943, ar eed a 22.6. After the completion of the Nagpur Road Plan targets, the Second Twenty Year Plan was drawn for the period 1961-1981. ‘The Third Twemty Year Road Development Plan for the period 1981-2001 was approved only by the year 1984, 2.10.1 Nagpur Road Plan or First 20-Year Road Plan ‘The Conference of Chief Engineer held at Nagpur in 1943 finalize the first twenty year road development plan for India called Nagpur Plan for the period 1943-53. The road net-work in the country was classified into five categories viz. () National Highways (ii) State Highways (ii) Major District Roads iv) Other District Roads and (v) Village Roads as, explained in Ant. 2.4.3. Recommendations were made for the geometric standards of roads, bridge specifications and highway organizations, Two plan formulae were finalised at the ‘Nagpur Conference for deciding two categories of rotd length forthe country as a whole as well as for individual areas (like district). This was the first atempt for highway planning in Indi “The two plan formulae assumed the StaP and Grid pattern of road net work. Hence the two formulae are also called Star and Grid Formulae. ‘ ee “The total length of the first category or metalled roads for National and State Road | Hneth eo vilage served population Productivy 1000 ts Highways and Major Distrit Roads in km is given by the formula: 10-5000 | > cm ‘i A | 1s [10 8 =e Agriewal India NH + SH+MDR (km)= A, Rerovest|+D-8 an bjt is 2 U in 0.0 Cc [18s |20 10 2 20 08 where A = Agricultural area, km? Solution B = Non-gricultural area, km? tr N= Number of towns and villages with population range 2001-000 ‘able (Phasing of road development plan) T= Number of owns and villages with population over $000. Length, a D = Development allowance of 15 percent of road length calculated to be Road | “nh, Total utility units served by the road ua provided for agricultural and Industrial development during the next 20 years unit le ee : a 15 1Ox0.5+8x1+3x24 15x14 12x10 = 46] 46/15= 3.07 R = Existing length of railway track, kim. mi - 12 16x0.543x1+1%24 11x 1+0=24) 24/12=20 “The total length of second category roads for Other District Road and Village Roads in 18 20x 0.54 10x 1+2%2+20x1+ 08 x 10=52| 52/18 = 2.89) km is given by the formula : Therefore order of priority is A, C and B Assume the following utility units as per the given guide lines: Per village served with Population<2000 = 0.5 Per village served with Population 2000-5000 = 1.0 Per village served with Population > $000 = 2.0 Asricultural products served per 1000t = 1.0 Industrial products served per 1000 = 10.0 ODR+VR (km) = [0.32 V+0.8Q + 1.6P +3.25]+D @2) = Number of villages with population 500 or less [Number of villages with population range 501-1000 + with population range 1001-2000, where = Number of villages “Number of villages with population rang 2001-5000 Development allowance of 15 % for next 20 years v Q Pp s D 1 SHWAY DEVELOPMENT AND PLANNING ui 36 Jac, it may be seen that in addition ta the to HIGHWAY PLANNING IN INDIA, v ¢ above two fon the atone specific road length were alloca nd mo agricultural a € “herent population ranges. For example a town ee fi fon agricultural a” a ae cueen 2001 and $000 is allocated a road length of 1.6 km of fr Freee Edam af second category road whereas a village with population aaa SOE sat 039 kn of sevond category oad, Tength of 178 kan a fe Naa oy sob ponte pr tof agua area, This means that grid off ed too af ped at 16 Ams sch hat am area of 16 16 kn? provided with a ra Ni Ses Age of rood length, te 32 km rood length is available in an acu orem Sse oe LO km per 8g Amarca and therefore the term 4/8 is sed nthe Ea 5 aker Blame Roads "a Total main roads Salient Features of Nagpur Road Plan 2 palsy) s fi ‘ E__[ Other District Roads (i) The espensibiiy of constuction and maintenance of national high a bomuge eae assigned to the central government 5 Milnes Rade Tova (ii) Tewas a 20-year plan intended for the period 1943-63 aiming to provide for two lekh km of surfaced roads and remaining unsurfaced roads, so that they target is reached, the total road length of 5, 32, 700 km with a density of {km of road length per 100 sq, km area would be available in the country by: 1963. (iii) ‘The formulae were based on star and gird pattern of road network. But the ex imegular pattern of roads and obligatory points not fitting in the geometric ‘were to be given due consideration, Example 23 ‘The following data were collected for planning the road development progran.me of backward district. (i) Total area = 9600 km? (Gi) Agricultura and developed area = 3200 kan? (ii) Existing railway track length = 10S km. (0) Existing length of metalled road = 322 km. (9) Existing ength of unmetalled road = 450 km, (vi), Number of towns or villages in different population ranges are as below (iv) The first category roads are meant to provide main grids bringing the farthest points in developed and agricultural area within 8 km of metalled road. The size ofthe’ | gird of this category of road in agricultural area would be 16 km so that he | ‘maximum distance from the centre is 8 km and the average distance of the village from mealled road would be less than 3.2 km. In non-agsicultural area the sre Population $3000 | 3007-5000 | 1001-7000 | S0T-1000 [= 305} the gird is of 64 km sides, the farthest distance from the centre to the metal jumber of villages |g 40 30 280 | $90 roads being 32 km. The length of road of this category is governed by the ‘and towns —— Particularly the agricultural area and towns or villages with population 2001 Calculate the additional lengths of metalled and unmetalled roads for the rad wyste= ‘based on Nagpur Road Plan formulae for this district. Solution (i). The total length of metalled roads by Nagpur Plan formula is obtained for equation 2.1 and is equal to: A,B ALB sponsst+p-k tay hanes (*) The second category roads are meant to provide intemal road system linking villages with first category roads. The road length of second category is work ‘onthe basis of villages of different population ranges, of population less than ‘as estimated as 15 percent and this allowance was to be ‘eleulating the road length for both the categories of roads, He ere, ‘A. = 3200 km?: B = 9600 -3200 6400 km? N= 40, T= 8;D=15 percent; R= 105 kro 3200, 6400 1 6. 4o.¢ $uB] + 1306 total road length 10 Meat rnd eng = [228 SP 6xs048=8] 38 HIGHWAY DEVELOPMENT AND PLANNING = [400 +200 + 64 + 64] + 18% of RL — 105 HIGHWAY PLANNING IN INDIA ” Sx 728 g, mi = 7as+ S378 — 105 = 732.24 iii) National Highways + State Highways + Major District Roads (km) A,B OC ‘Additional metalled road neoled [ABS SUBK 624M + 11 2N «9,604 640 +248} + ibe = 732.2-322=410.2k1 732.2 -3 ant Fi) National Highways + State Highways + Major District Roads + Other District Roads (i) Tota length of untae roads by Nagpur plan formula may be obtained tog EY” equation 2.2 and is equal to Tom 3A 3B oc M edstfeuseults [222 necctinarsnaes igo: aoe Here, V = 590, Q=280, P= 130,S=40, D= 15% Unmetalled road length = 0.32% $90 + 0.8 x 280+ 1.6 130+ 3.2% 40}+ 15% road length = [188.8 + 224 +208 + 128] + 15% of RL 5 TABBxIS _ = 1488 + 5g = 861 km Additional unmetalled roads required = 861-450= 411 km 2.10.2 Second Twenty Year Road Plan (1961-81) ‘The Nagpur road plan was intended for the period 1943-63, but the target road lengthy was nearly completed earlier in 1961 (as shown in Table 2.4), mainly because of the phased development that took place in the country during the first two 5-year p! ‘Hence the next long term plan for the twenty year period commencing from 1961, a5 initiated by the IRC and was finalised by the sub committee and this was approved by the Chief Engineers. The Second Twenty: Year Road Development Plan 1961-81 is a0, called Bombay Road Plan. a ‘The second road plan envisaged overall road length of 10,57,330 km by the year 198). The cost of the plan has been worked out to Rs. 5,200 crores based on 1958 pricé Ie for a period of 20 years from 1961. Five different formulae were framed to calculate the lengths of National High State Highways, Major District Roads, Other District Roads and Village Roads. ‘These five formulae are given below : i) National Highways (km) Be: ns + (32K +8M]+D. )) National Highways + State Highways (km) A -|A,B lc [$+ B-S] +tacsanesnians veryep ii) The maximum distance from any 26) 'v) National Highways + State Highways + Major District Roads + Other District Roads + Village Roads ic. all roads (kr) [+B] sane anne rian oer 89+ son 165 oeiT+02v]+D ar = Developed and agricultural areas; km? = Semi-developed area, km? = Undeveloped area, km? = Number of towns with population over 1,00,000 = Number of towns with population range 1,00,000-0,000 = Number of towns with population range 50,000-20,000 ‘Number of towns with population range 20,000-10,000 = Number of towns with population range 10,000-5,000 = Number of towns with population range 5,000-2,000 = Number of towns with population range 2,000-1,000 = Number of towns with population range 1,000-500 = Number of towns with range below 500 = Development allowance of 5 percent of road length calculated for further development and other unforeseen factors. v A wo . Hons —— Fane | 3240 HIGHWAY PLANNING BH INDIA a NH a is 6, No» 25.P 80 das x 10S 287.28 km ia eas 240 1800 Bane su | Em OHO sees a Dba ime th 23501 brah . 00 48x34 24% 7 1212+ 16x 3 2 NH SH 5 20] sora, -s : 240 + 240+ 280» 1303 12% 777.68 x LOS = 816.83 kn 2 [HOO + 233 + 180 + 240 240 + 280 + 12K} SH = $16.83 - 257.25 = $59.28 km = 1671 OS op 754.skm Example 25 = 1784.5 ~ $09.25 © 124528 kin ‘Calculate the total lengths of NH, SH, MDR, ODR and VR needed in a district ji) NH + SH + MDR (km) = second 20-year road development plan or Bombay Road Plan. The data collected ‘fe the district are given below : 1 (i) Total area = 18,400 km? (ii) Developed and agricultural area = 8000 km? (iii) Undeveloped area = 4800 km? (iv) Population centres are as given below : ABO, sak pM AN B6P 64Q 9 24) a 16°24 Jere. Q = 150, R= 300 SH + SH + MDR 000 | 5600 | 1800 | sya 5424n 104 M2 2S 906 + B= a 6 3s 644150 +244 360] Population range | Number of towns = < 500 200 = [1000 + 350 + 200 + 240 + 240 + 280 + 768 + 960 + Bet] 500 - 1000 350 7000 = 2000 750 = 3902 = 108 5147 km 2000 - 5000_ 360 a 5000 = 10.000. 150 MDR = $147~ 17848 3392.5 km 70,000 - 20.000 80 apes 3A BC . 20.000 = $0,000 7 UNIL* SH + MDR + ODR (km) = FE + 35 + 7g 148K + 24M 1128 50,000 -1,00,000 10 Te Teena Se: > 1,00,000 5 +9.6P + 12.8Q+4R+ 08S G32T+D blige ere = 750,17 +380 38000 35600 4800 . A B.C ats sit MOK + oDk = [2400 , 28608 00 agus 26-109 112428+ © Nnom - A484 o hom) = 3+ ay + gg + BOK + 8M] +D 16 2 16 ‘ fits K © ‘S00 ids 4 606 ba? 94680 + 12.8x15044360-+082750+032+380] + Sof RE B= = = 18,400 — [8,000 + 4,800} = 5,600 km? = [1500+ 525 + 300 + 240 +240 + 280 +768 + 1920+ 1449 + 600 + 132] 18 K = 4)M=10:D= 5% - 7925 x 195 = 9321.2 NH (km) = 109 * 871. 800 Set tese a0] +5% of RL 36 ODR = 8321.2-$147.0=3174.2 km v) NH + SH + MDR + ODR + VR (km) = {125~70-+ 50+ 160+ 80) 105 = 4gs x 405 = 50925 }00 100 [t BE ances eti2nssaretaag- so tas-06t 029) a 4 Gi) NH+ SH km) = ig 5,8 0 24 aTtHK 2 9 11.aN .6P] +0 4800 5 gyn $e 2481041128254 9.6804 2 HIGHWAY PLANNING IN INDIA 45 Mi) Expressways should be constructed along major traffic corridors to provide fast travel, (vi) All the towns and villages with population over 1500 should be connected by Major ‘i ye S90360 +1750 0614350025299) : usstsors 230+ 2 : oo 70+ 400° na 4 TERS 28 Tg +224 ga) District Roads and the villages with population 1000 to 1500 by Other District " 4 Roads. ‘There should be a road within a distance of 30 km in plains and 5.0 km in 36x 105 = 10642.8 km Z hilly terain connecting all villages or groups of villages with population less than = 10138 799 ty vy = 200 VR = 10,642.8 - 8321. ‘The various road length required areas follows ‘NH = 509.25 km; SH = 1245.25 km; MDR = 3392.50km; ODR 3174.2 km; VR 2321.6 km Vii) Roads should also be built in less industrialized areas to attract the growth of industries. at various levels, Long term master plans for road development should be prepared ies taluk, district, state and national levels. The road, net work should be scientifically decided to provide maximum utility. ‘The existing roads should be improved by rectifying the defects in the rosd ‘geometries; widening of the pavements, improving the riding quality of the pavement surface and strengthening of the pavement structure to save vehicle operation cost and thus to conserve energy. environmental quality and road safety. Here ‘There should be improvement jermination of Road Lengths by Third Road Plan Formulae 2103 Third Twenty Year Road Development Plan 1981-2001 Policies and Objectives : n As mentioned in An. 2.2.8, the Third Twenty Year Road Development Plan, | 2001 (also known as Lucknow Road Plan) was finalised and the plan docum published by the year 1984. The major policies and objectives of this road plan. below: providing 100 + 100 = 200 km of road length per 100 x 100 = 10000 sq. km area i.e. one km per 50 sq. km area. Therefore the total length of NH in the country or ina state could be obtained by dividing the total area of the country by 50 (i) The future road development should be based on the revised classification ‘ssem consisting of primary, secondary, and tertiary road systems as ment Secondary System :Length of SH ‘The roads consisting of NH and SH should pass through every town or urban area; are 3364 such towns in the country as defined by 1981 census. Therefore the area of the square grids would be equal to total area divided by the number of towns 32,87,782/3364 = 977.3 sq. km, with sides J977.5 = 31.26 km. Therefore the length (NH + SH) will be 2 * 31.26 = 62.5 km for each such square gird. Thus the total of NH + SH) for 3364 towns in the country = 62.5 x 3364 = 2,10,250 km. The total length ENH for the country as determined earlier is 66,000 km. Thus the total length of SH in country = 2,10,250 ~ 66,000 = 144,250 km, or say 1,45,000 km. (ii) The road net work should be develo, ped so as to preserve the rural Sac and develop small towns with all the essential fei ith population over $00 (based com all weather roads by the end ofthis seitary, ee (i) Te ore rad seaiy in the country should be increased to 82 km pet 100 for bill areas false corresponding values of planned road densities foralitud above 210g gon 2100 metres above MSL. and 15 km pet Stes bt rk should be expanded to form square pds Te gall length of SH required for any State may be determined from the following Pat ofthe county is more than 50 km away from a NH: oo () The lengths of SH and MDR required i (a) By total area, SH, km = Area of the State, sq. km/25 on both Humbe fe atited in a state or region should be det region lumber of towns sy; (b) By total no, of towns and area in the State, SH, km = 62.5 x no. of towns in the th ‘ in the Population above 5,000 in IF State ~ area of the State, sq. km/S0. NGUWaY DEVELOP) ENT AND PLANNIN: T 'LANNING, ) Length of MDR “a7 REFERENCES Total Length of MDR in the country The tor i al te ge SMH OF MOR required i fi ation om iN State is determined fom one of ther i) Length of NH, m= 13400/50 = 168 kam (@) By total area, MDR, km iy Langth of SH z ~ Area ofthe State, sq. kav12,5, tate, MDR, km = 90 x no, of ‘oads : Length of ODR and VR, (b) By no. of towns in the § {a By area, SH, km = 1340025 = $36 km WL Tertiary Hi, km = 62.5 x 12 - 13400/50 = 462 km (b) By area and no. of towns, St ‘System or Rt 2s ot h of SH (higher ofthe two criteria) ~ 536 km The total length of ODR an. towns in the State ‘Adopt lengt! development et d_VR in the country b e in the District Plan shall be 21,89,0( ty by the year 2001 as per the H(i) Length of MDR, in roads ic.. NH, SH, MOR ODie soe KM 50 thatthe overall length aes setae | 2 13400/12.5 = 1072 kin eh (a) By area, MDR, kim = 13400/12. The toll Iength of all ee VE ll be 27 ak sth OF all catego (@) By area, (©) By 90. of towns, MDR, kin = 90 > 12 = 1080 km Providelenghof MOR (bigheraftieowo crea) = 10808 (Gv) Total length of all etegories of roads may be assumed to provide an ove y of road length equal to 82 km per 100 sq. km area by the year 2 el 82/100 = 10988 km dept on ute dsetton ofthe lengths of ODR anf NH+SH4MDR+ODR+VR = 13400 x 82/100 = 10988 Length of NH+SH+MDR = 268 + 536+ 1080~ 1884 km ing of ODR + VR = 10988 ~ 1884 = 9104 km ‘Table 2.6 Estimated total road lengths in some states by the year 2001, as per third road development plan targets } ‘Therefore length of Rural roads consi 268 km Total road lengih (NH, SH, MDR, Secondary system consisting of SH = $36 and MDR-= 1080, total lengsh = 1616 km Sue ODR & VR) by the year 2001, km ‘Tertiary system of Rural Roads consisting of ODR and VR = of length = 9104 km ae ana tore LB Taide 10H on 3.__ Bihar 2,12,032 4. Gujarat 114.886 REFERENCES Se Hacyas HB "1. Sinha, 8, Development of Roads in India, The Indian Concrete Joumal, Oct. 1968. Himesh ae 33,840 2. Bahadur, J, Basis of Road Planning, Journal, Indian Roads Congress, Vol. XII, 1988. i ‘Karnataka 144,654 3. Srivastava, P. N., Fact Finding Survey and Highway Planning in Uttar Pradesh, a Kerala 128,963, Journal Indian Roads Congress, Vol. XX-3, 1955-56. s Pradesh 3,41,268 4. Ministry of information & Broadcasting, Our Roads, Modem India Series No. 11, 10._Madhye 2.20871 Publications Division, Govemment of Indi 11, Maharashtra Tea ions Division, Goverment of ie One STs? ; 5. ESCAFE, Report of the Seminar on Low Cost Roads and Soil Stabilization, New 13._ Punjab —J 7,09,392 Delhi 1958, 14, Rajasthan 1,38,115 6. Minister of Transport and Communication, Road Development Plan for India 15. Tamil Nadu 3735160 | (1961-81), Dept, of Transport, Government of India. 16._Uttar Pradesh 1,15,792 7. Bateman J. H., Introduction to Highway Engineering, John Wiley & Sons Inc. i 8. Bruce, A, G., Highway Design and Construction, International Text Book Co, Coltins, H.4, and Hart, C. A., Principle of Road Engineering, Edward Amold & Co. km and there are 12 to¥ms i 10. Riter, LJ, Paquette, R.J., Highway Engineering, The Ronald Press Co. 6 : Example 2. of roads o be provided in distri The area of a certain 198) census. Determine the = district by the year 2001- in India is 13,400 59. K rats of aiferet categorie: |. What are the various methods of classifying roads ? Briefly outline the - What are the various surveys to be carried out before planning a highway system . What are the various plans to be prepared after the planning surveys are carried . Explain how the master {IGHWAY DEVELOPMENT AND PLANNING the National Transport Policy Committee" Planning. Commission, Report of the National Tra Government of Inia, 1980. } Mise’ of Shipping and Transport, Roads Wing, “Road Development Plan for India (1911-2001)" Ineian Roads Congress, 1984. PROBLEMS ‘Briefly discuss the historical development of road construction, ‘What are the salient features of early Roman Roads ? How do these differ from the present day road construction ? ‘Compare the construction methods of Telford and Macadam; bring out the points of differences. Briefly explain the Macadam’s method of road construction, Why is this method considered better and more scientific compared to the previous methods ? Discuss briefly with reasons how further improvements to the water bound, ‘macadam road became necessary. Briefly outline the highway development in India. Explain the necessity and objects of highway planning. classification based on location and function as suggested in the Nagpur Road Plan What are the significant recommendations of Jayakar Committee Report ? Mention how this helped in road development in India ? Explain briefly the modified classification of road system in India as per the Third ‘Twenty Year Road Development Plan, 1981-2001 Briefly outline the main features of various road patterns commonly in use. Explain ‘with sketches the star and grid pattern. fora given area? Explain briefly, out? What are the uses of fact finding surveys ? How are these used and interpreted ? es Plan is prepared and the road development programme is Explain why the saturation system is considered a rational method to decide the final road network and for phasing the road development programme. lustrate the saturation system with an example. P Plans for a backward district. The details with reasons which proposal isthe best assuming, Utility” 8 for the five populati 4 ial products care tion anges and 1.0 and 5 per 1000 units of 0.5, 1.0, 2, 4 and of agricultural and industri 19, 20, 21. PROBLEMS, Total | Number of _ ; 2 Pros) toad orale ered wih Productivity in posal | length, | <2000] 200). 5001 ODOT: > 20000) Rowse ones km 5000 | 10000 | 20000 |Agriculural | industri P| soo { 190 [iso 40 fT 39 T > 156, Qf 600 {200 [350 168 |~38-| 3} 999 3 R 700_| 270 [350 82 36 4 300 35 ‘S| 800 [280 | 410-[ 91} ar 4 400, 42 T | 900 | 290 | 430 | 96 |~aa [a a [Ans. : Proposal S with 1.923 unit'km is the best; 357, T= 1.806, Q= 1.612, P= 1.268) Four new road links A, B,C and D are to be constructed during a five-year plan period. Suggest the order of priority for phasing the road construction based on maximum utility approach. Assume utility units of 0.5, 1.0, 2 and 4 for the four population ranges and 2, 2 and 5 units per 1000 t of agricultural, raw material and industrial products from the following data: No.of villages served with Road | Length population range Productivity served, Tink | km |2000] Agrie- | Raw [Industral to00_| 2000 ultural | Material_| product A_| 75 [30 | 1s [| to | 3 | 8000 |" 3000 | 1000 B | 35 [20 | 8 | 6 | 3 | so00 | 1000 | 1600 cf 40 [is | 6 | s | 5 | 6000 [2000 | 3200 bp | so {4 | 4 | 3 | 2 | 3000 | 7000 | s00 [Ans. I Priority = C (1,888 unitskm), l= B (1.771), IIl= A (1.453), 1V=D(120)] i in bi Four country ? What is the importance of Nagpur road plan in highway planning o Explain the plan formulae and the salient features ofthe plan. Discuss the second twenty year road plan of 1961-81 and is salient features Compare the Nagpur road plan and the second twenty year road plan; discuss the of each. _ From the following data for a district, ealelate the road length required bssed on Nagpur road plan. Total area = 6300 km? = 2800 km? Agricultural area = 280 1-100, 101-2000, 200100, and No. of villages with population ranges < 500. on eS above S001 ee 450, 320, 110,50 and 10 respec. Legh ofa Fig dv (Ropar ange towne wings [Seabees 3001 = 000 38 ‘S001 - 10,000 20 [0,001 = 20,000 20,001 = $0,000 $ "50,001 - 100,001 2 rn rc 454A) Ans. | category roads 50 23, 25. 26. HIGHWAY DEVELOPMENT AND PLANNING. ‘Work out the lengths of NH, SH and MDR required in a district by second 20.yeq road development plan (1961-81) using the following plan formulas and data ey Problem 22 : a NH = A/64+B/80+C/96 +32K +8M+D NH+SH = A/20+B/24+C/32+ 48K +24M + 11.2N+L6P+D, NH+SH+MDR = 3A/16+3B/32+C/16+48K +24M+11.2N+96P+ 12.8Q+4R+0.88 +0327 +p. Area of the district = 10,800 km? Developed and agricultural area = 4100 km? Undeveloped area = 2300 km? j [Ans. : NH = 267.8, SH = 618.6, MDR = 1149.9 km) }. What are the policies and goals of the Third Road Development Plan for: 1981-2001 ? Explain how the road lengths of different categories for a state are determined the year 2001 using the Third Road Development Plan concept. Determine the length of different categories of roads in a state in India by the year 2001, using the Third Road Development formula and the following data : = 80,000 sq. km Total no. of towns as per 981 census = 86 Overall road density aimed at = 82 km per 100 sq. km area [Ans.: Primary/NH = 1600, Secondary : SH = 3200, MDR = 7740, j Tertiary : ODR & VR = 52,485, Total = 65,600 km] Total area of the state Write short notes on : (@) Central road fund (b) Nagpur road plan (©) Fact finding surveys (d) Master plan (e) Saturation system (f) Star and grid pattem (g) Indian Roads Congress (h) Jayakar Committee &) Chapter 3 Highway Alignment and Surveys 3.1 HIGHWAY ALIGNMENT “The position or the layout of the centre line ofthe highway onthe ground is called the alignment. ‘The horizontal alignment includes the straight path, the horizontal deviations ‘and curves. Changes in gradient and vertical curves are covered under vertical alignment of roads. ‘A new road should be aligned very carefully as improper alignment would result x one or more of the following disadvantages (a) increase in construction cost (b) increase in maintenance cost (©) inerease in vehicle operation cost (@ increase in accident rte. of adjoining land a in Sf er sas avc oer consratons whe fein the afgnmes cf °° road need not be over emphasised. 3.4.1 Requirements ‘The basic requirements of a should be : (a) short (b) easy (©) safe, and (@)_ economical 1 anand HIGHWAY ALIGNMENT AND SURVEYS, Short Wis desirable to have a short (or shortest) alignment between two terming stations. A straight alignment would be the shortest, though there may be sey, practical considerations which would cause deviations from the shortest path . Easy : The alignment should be such that itis easy to construct and maintain ther ‘with minimum problems. Also the alignment should be easy forthe operation of vehi with easy gradients and curves. si Safe: The alignment should be safe enough for construction and mainten: the view point of stability of natural hill slopes, embankment and cut sl foundation of embankments, Also it should geometric features. ance from, lopes and” be safe for the traffic operation with sa Economical : The road alignment could be considered economical only ifthe total cost_ tenance cost and vehicle operation cost is lowest alignment. The alignment should be such that it would offer maximum utility by serving ‘maximum population and products. The utility of a road should be judged from its utility value per unit length of road. (For details refer Art. 2.9) 3.1.2 Factors Controlling Alignment For an alignment to be shortest, it should be straight between the two terminal stations. This is not always possible due to various practical difficulties such as intermediate obstructions and topography. A shortest route may have very steep gradients and hence not easy for vehicle operation. Similarly, there may be construction and maintenance problems along a route which may otherwise be short and easy. Roads are often deviated from the shortest route in order to cater for intermediate places of importance or obligatory points. A road which is economical in the initial construction cost, need not necessarily be the ‘most economical in maintenance or in vehicle operation cost. It may also happen that the. shortest and easiest route for vehicle operation may work out to be the costliest of the different alternatives from construction view point. Thus it may be seen that an alignment can seldom fulfill all the requirements simultaneously; hence a judicial choice is made considering all the factors. ‘The various factors which control the highway alignment in general may be listed as : (a) Obligatory points (b) Traffic (©) Geometric design (d) Economics (©) Other considerations In hill roads additional care has to be given for : Stability Drainage Geometric standards of hill roads, and ens aes HIGHWAY ALIGNMENT 5; s (@) Obligatory Poins : There ae : highways, These conta pois maybe aides teat eee He apa of he wo categories (0, Points through which he aiznne iso ase (i) is trough Which the agamen tui np (Obligatory ood acy alignment to often deviate fom he sates aes 18 may cae the or eas ations category may be bridge sit intermedi towne moun pe cageg ne hill pass is available. The suitability of like the topography and site conditions and cost considerations. Figure 31 a shows how the straight alignment AB is devised along the hillside pas, thus avoiding tunnel or heavy cutting, demote ie 5 os Qo tp eanmecrine LINE aOAo (a) Alignment avoiding on itermediste area 0) Alignment to connect intermedioe town Fig. 3:1 Obligatory Points Controlling Arai ht ‘The road bridge serosa iver can be cated only at ple wher ta Se and permanent pth and whee the bridge abutment and ler ome EPS Se ‘The bead aporosches to this bridge should not be cued nea the RAGES oe te yhew crossing should be avoided. Thus in ret 1 ore tite a rede cetera, gee oc alfgetent bebveen stations A and B which pasts 2 yee some ben cet Portion ofthe river on the up-stream side ons. tay While aligning a road been 0 a paces of the important intermediate towns, vl be desirable 10 connect some oer The steht alignment AB HIGHWAY ALIGNMENT AND SURVEYS station C. Its also possible fo connect the station C with a link road as shown int same figure, thus avoiding the deviation of the straight alignment, Mt (ii) Obligatory points through which the road should not pass also may make necessary to deviate from the proposed shortest alignment. The obligatory points wh should be avoided while aligning a road include religious places, very costly st unsuitable land ete. Religious places like temple, mosques, church, grave or tomb hy been protected by the law from being acquited for any purpose. Acquiring cost structures would mean heavy compensation resulting in increased cost. taken across such an area, the construction and maintenance costs are likely to be v high due to special construction techniques and drainage measures to be adopted. A lake, a pond or a valley which falls on the path of a straight , necessitate the alignment to deviate from the straight path and go round along the grade line as shown in Fig. 3.1d. (b) Traffic : The alignment should suit traffic requirements. Origin and Destination study should be carried out in the area and the desire lines be drawn showing the trend of traffic flow. The new road to be aligned should keep in view the desired lines, traffic flow patterns and future trends. (©) Geometric Design : Geometric design factors such as gradient, radius of curve and sight distance also would govern the final alignment of the highway. If straight alignment is aimed at, often it may be necessary to provide very steep gradients. As far as possible while aligning a new road, the gradient should be flat and less than the ruling or design gradient. Thus it may be necessary to change the alignment in view of the design speed, maximum allowable superelevation and coefficient of Iateral friction. It may be necessary to make adjustment in the horizontal alignment of roads keeping in view the minimum radius of curve and the transition curves. The absolute minimum sight distance, which should invariably be available in every section of the road, is the safe stopping distance for the fast moving vehicles. Also there should be enough distance visible ahead for safe overtaking operations of vehicles. moving at design speed on the road. Hence the alignment should be finalised in such @ way that the obstructions to visibility do not cause restrictions to the sight distance requirements. ‘The details of these geometric design factors are given in Chapter 4. (d) Economy: The alignment finalised based on the above factors should also, economical. In working out the economics, the initial cost the cost, of maintenance vehicle operation should be taken into account. The initial cost of construction can b decreased if high embankments and deep cuttings are avoided and the alignment is ch« ina manner to balance the cutting and filling. (¢) Other Considerations : Various other factors which may govern the alignment drainage considerations, hydrological factors, political considerations and monotony. vertical alignment 1s often guided by drainage consi « seepage flow and high flood level are the factors to be kept in view. ENGINEERING SURVEYS FoR ingHway, LOCATIONS ss ‘A foreign territory coming across st alignment around the foreign land, At raight ali considerations. ignment will necessitate deviation of times the alignment is desided only on strategic Special considerations while aligning roads on hilly areas Stability ; While aligning hill roads, special care should be taken 10 align the road along the side of the hill which is stable. A common problem in hill roads is that of and slides. The cutting and filling of earth o construct rozds on hillside causes stepening of existing slopes and affect is stability, Drainage : Numerous hill-side drains should be provided for adequate drainage facility across the road. But the cross drainage structure being costly, attempts should be made to align the road in such a way where the number of cross drainage structures are ‘minimum. Geometric standard of hill roads : Different sts of geometric standards are followed in hill roads with reference to gradient, curves and speed and they consequently influence the sight distance, radius of curve and other related features. The route should enable the ruling gradient to be attained in most of the length, minimising steep gradients, hair pin ‘bands and needless rise and fall. sisting lengeh : The resisting length of road may be calculated from the total work to be done in move the lads along the route aking the horizontal tng, ie ad difference in levels between the two stations andthe sum of infective rise and fal in excess of floating gradient In bret, the resisting length of te alignment should kept Tow as possible. Thus the ineffective rise and executive fll should be kept mi road alignment are discussed in chapter 12. ‘The detailed considerations on bil 3.2 ENGINEERING SURVEYS FOR HIGHWAY LOCATIONS i the engineering surveys jefore a hi ligament is finalised in highway project _leeamon tes» tm fa” Artec Consider all possible ales aligames Kepeg in view te atoms eon highway alignment as discussed in Art. 3.1.2. The survey af the selected alignment. “The stages ofthe engineering surveys a (2) Map study (b)_ Reconnaissance (€) Preliminary surveys (d)_ Final location and detailed surveys. siegest te kel rey of Indic, With 1 aires like rivers, hills valleys ete. ate also ve an idea of several pg be studied later atthe site, The pr nthe following details availabe on the map, aps, itis possible to ls of thes. ent avoiding valleys, ponds oi {g) When a road is to be connected between two stations, one of the top and the of on the foot of the hill, then alternate routes can be suggested keeping in view the permissible gradient; say the ruling gradient. Refer Fig. 3.2. Suppose the scale of! the contour map is known, then from the counter intervals itis possible to decide th length of road required between two consecutive contours, keeping the gradien ‘within allowable limits, In this case, the contour interval is 15 metre and if ruling gradient is fixed as 1 in 20, the road length between two consecutive conto has to be 15 = 20 = 300 meter. With the known scale of the map, the various possible alternate routes may be drawn by drawing ares of the above (300 metre), Jength between the consecutive contour lines. Let A and B be two stations to be connected by a road, see Fig. 3.2. AB is the shortest route (straight line) APQB is a steep route in which the gradient positively exceeds 1 in st route APOB + Steeper gradient APLMINB gradiont APEFGB - Flatter gradient (Alternate route) Fig. 3.2 Alignment with allowable Gradients ale to be reas APEVGB bs ay ate sen from map study 10 drop a certain soae undesirable ground, ensowe, ‘Map st further surveyed in the fied 3.2.2 Reconnaissance The second stage of surveys for highway locstion is the reconnaissan general character of the area for deciding the most feasible field survey party may inspect a fairly broad stretch of land clone routes of the map in the field. Only very simple insrument clinometer, barometer etc. are used by the reconnaissance party to callect additines! details rapidly (not accurately). All relevant details not available in Isp are collected and noted down, Some of the details to be colfected during reconnsissance are gives below : (i) Valleys, ponds, lakes, marshy land, ridge, hills, permanent structures and otter obstructions along the route which are not available in the map. ‘Approximate: values of gradient, length of gradients and radius of curves of alternate alignments. (iii) Number and type of cross drainage structures, maximum flood level and oxmural ground water level along the probable routes. (iv) Soil type along the routes from field identification tests and observation of geological features. ™ Suet of constuction mer at and ston of 0 GTS = i) When passes through hilly or ‘mountainous terrain, ition © eae, ‘ooo Ferny type of rocks, dip of se eee ee = may be observed so as to decide the stable and unstable sides of ced alignment. ' A rapid reconnaissance of the ares, especially when 4iffcult, may be done by an aerial survey. i proposed after From the deals collected during the reeonnisane, he HENNE RET study may be altered or even changed one Sd ‘considerations alternate alignments may be chosen for further prsacal observed atthe site is vast and the terrain is 3.2.3 Preliminary Survey . “The main objectives of the preliminary survey a= < after the reconnissne® nd 10 (]) To survey the various alternate alignments POP aeals of toposrapiy drainage collect all the necessary physical informe and soil. aren props io vr of requirements of a good i compare the O° lem 38 HIGHWAY ALIGNMENT AND SURVE) Git) To estimave quantity of earth work materials and other construction aspects workout the cast af altemate proposals, (iv) To finatise the best alignment trom all considerations, The pecliniinary survey is carried out to collect all the physical information which, necessary in connection with the proposed highway’ alignment, The preliminary sur may be carried out by any one of the following methods : (2) Conventional approach, in which @ survey party carries out surveys using required field equipment, taking measurements, collecting topographical and other and carrying out soil survey (b) Modem rapid approach, by serial survey taking the required aerial photog and by photogrammetric methods and photo-interpretation techniques for obtaining necessary topographic and other maps including details of soil and geology. ‘The procedure of the conventional methods of preliminary survey i steps : iven in follo (i) Primary trayerse + The first step in the preliminary survey is to establish the primary taverse, following the line recommended in the reconnaissance. Fo alternate alignments either secondary traverses or independent primary travers may be necessary. As these traverses are open ‘raverses no adjustment of errors i possible later, so the angles should be very accurately measured by’ the theodolite The length of the centre line should be measured by using very good and accurate) ‘chaining methods or by tacheometry or by modem instruments, Gil) Topographical features : After establishing the centre lines of preliminary survey, the topographical features are recorded. All geographical and other man mad features along the transverse and for a certain width on either side are surveyed plotted. The width to be surveyed is generally decided by the survey party, but absolute minimum width is the land width of the proposed alignment, (iii) Levelling work : Levelling work is also carried out side by side to give the cent line profiles and typical cross sections. The levelling work in the prelimi taken at suitable intervals, generally 100 to 200 metre in plain terra rolling terrain and upto 30 metre in hilly terrain. (iv). Drainage studies and Hydrological data ; Drainage investigations and hydrologi data are collected so as to estimate the type, number and approximate size of cf drainage structures. Also the vertical alignment of the highway, particularly grade line is decided based on the hydrological and drainage data, such as ponded water level, depth of water table, amount of surface runoff, ete. () Soil survey : Soil survey is an essential part of the preliminary survey 35 MY suitability of the proposed location is to be finally decided based on the soll 4% data. ‘The soil survey conducted at this stage also helps in working out detail earth work, slopes, suitability of materials, subsoil and surface requirements and pavement type and the approximate thickness requiremen'® these details are required to make a comparative study of alternate propos ENGINEERING SURVEYS 10K itajyway LOcatIONS At this stage a detailed soil survey ig suitable (pes of hand augers depending ein St. Po toe wae any ee sample up to depth of 1 10 3 mette below the likely fang es cage te round level, whichever is lower. When the road fr expercd ene oe ot te Eustint Erankent the depth of explain sponse Be fost ee an ‘embankinent from the ground level, During the sil exploration thc the height of is struck, the depth from the pround sutice sekersoeey oe water tbe » ‘When the work has to be done rapidly, xenpscal meth of sol exploration are best suited as accuracy is not very important during the preliminary survey. The elcirca! resistivity method is commonly used in road projects. The method is baved on the principle that the earth and rock materials may be idemified bythe difercet values uf the resistance to flow ofa direct current. The soil samples collected during the field work are subjected to identification and classification test in the laboratory. Soif profit is obtained by drawing the longitudinal section along the proposed road alignment upto the depth of exploration. The types of soils encountered along the route upto the depth under consieration ate marked 0 the soil profile either symbolically or by suitable colour coding (vi) Material survey : The survey for naturally occurring materials like stone aggregates, soft aggregates, etc. and identification of suitable quaries should be made. Also availability of manufactured. materials like cement, lime, beick, etc. and thex locations may be ascertained. (il) Traffic survey : Traffic surveys conducted in the region form the bass for deciing the number of tf lanes and readuay wih pavement design and conomi analysis of highway project. Traffic volume counts of the classified vehicles are to Pa ot te exiting roads inh vegion preferably for 24 hus pr dy for seven days. Origin and destination surveys are very useful for deciding the alignment of the roads. This study may be carried out on a suitable sample of Vehicle users or drivers. In addition the required tfc data may aso be colleced so thatthe traffic forecast could be made for 10 1020 year periods. Vii) Determin inal cenire line + After completing the preliminary surveys and aioe i eae tui of aterative algnment te final eet ine of the road isto be decided in the office before the ial actin sree, Ws preliminary survey maps consisting of contour plans ongtudnal role an mS Eetions of the alternate alignments should be prepared and caefaly S008! dese he bet lenme terns eae se tne ts. Aer selecting the fina! alignme _ mint Tear Nn rib boric and ves lens of the designed. preliminary surveys, especially at phic surveys are very much suited wns Sa al phoxographic may whem the distance and area to be covered are vast; The SHNSY following steps: (a) Taking aerial photographs 0 longitudinal and lsteral_ over preparation of mossaics. surveyed with the requited are necessary for the : tes he strips of and 10 slaps, Wesical pee 60 WIGHWAY ALIGNMENT AND SURVE Ss (6) ‘The photographs are examined under stereascopes and contol points ang for establishing the traverses of the altemate proposals, The a located on the maps (6) Using sterco-pair observations, the spot levels and subsequently cont ai gst “ our be obtained. Also from the stereo pairs the topographical details may o down on the maps, (a) Photo- conditions, drainage requirements etc, 3.24 Final Location and Detailed Survey. The alignment finalised at the design office after the prelimi s preliminary survey is to be fi located on the field by establishing the centre fine. Next detailed survey Should bee ‘out for collecting the information necessary for the preparation of pla onstruc details for the highway project, ° arian Location ‘The centre line of the road finalised in the drawings is to be translated on the grour during the location survey, This is done using a transit theodolite and by staking of th centre line. The location of the centre line should follow, as closely as practicable, th alignment finalised after the preliminary surveys. Major and minor control points ar established on the ground and centre pegs are driven, checking the geometric design requirements. However modifications in the final location may be made in the field, found essential, The centre line stakes are driven at suitable intervals, say at 50 met intervals in plain and rolling terrains and at 20 metre in hilly terrain. Detailed survey ‘Temporary bench marks are fixed at intervals of about 250 metre and at all di and under pass structures. Levels along the final centre line should be taken at all sta points, Levelling work is of great importance as the vertical alignment, earth calculations and drainage details are to be worked out from the level notes. The cros section levels are taken upto the desired width, at intervals of 50 to 100 metre in pl terrain, $0 to 75 metre in rolling terrain, $0 metre in built-up areas and 20 metre in hill terrain. The cross sections may be taken at closer intervals at horizontal curves and there is abrupt change in cross slopes. All river crossing, valleys ete. should be su in detail upto considerably distances on either side, All topographical details are noted down and also plotted using conventional ‘Adequate hydrological details are also collected and recorded. A detailed soil survey is carried out to enable drawing of the soit profile. ‘The upto which soil sampling is to be done may be 1.5 (0 3.0 metre below the ground line St finished grade line of the road whichever is lower. However in case of Me embankments, the depth should be upto twice the height of the finished embanknt™ The spacing of auger borings very much depends upon the soil type and its vee CBR value of soils along the alignment may be determined for designing the pave™ The data during the detailed survey should be elaborate and complete for PEPE” detailed plans, design and estimates of the project. DRAWINGS AND Epo 3.3 DRAWINGS AND RE} 3.3.1 Drawings a ORT ‘The Following drawings ar aly prepared in a hiphay (iy Key map i) Index map (ii). Preliminary survey plans (iv). Detailed plan and longitudinal section (9), Detailed cross-section (vi). Land acquisition plans (ii) Drawings of cross drainage and other retaining structures (vii) Drawings of road intersections (ix). Land plans showing quarries ete, Key map should show the proposed and existing roads, and important places to be connected. The size ofthe plan generally should not exceed 22 » 20m, The sale ofthe map is chosen suitably depending upon the length of road. dindex map should show the general topography of the ates. The details are symbolically represented. The index map should also be of suitable scale, the sire beng 32 « 20m. Preliminary survey plans showing details of the various aerate alignments and alt informations collected should be normally drawn to scale of 10.em = 1 km to 25 cm =k, Detaited plans show the ground plan with alignment and the boundaries, contours intervals of 1 to 2 metre in plain country a scale of 12400 and in close county, a eae of 1/1200 may be adopted for detailed plans. The size of the drawing may be A-2 size or 60 x 42 em approximately, ctlons should be dravn to the same horizontal scale ofthe ground a> im detailed plan, Vertical scale may be enlarged 10 times ofthe Tongans wats The longitudinal section should show the details suchas datum fine, existing Vertical profile of the proposed road and position of drainage crowings Detailed cross sections are generally drawn 1 natural scale of Tem = 20 1 2° Cross es ‘should be drawn every 100 m or where there are aie east eel hill roads the cross sections shoul be denon at elon mieten THE drawings should extend at leat up t0 the proposed ehh of Toy Nt rhcopar ‘umber, the reduced distances and the rex of filing m ross section drawings. Longitudinal se fiom the survey dowings Land acquisition plans and schedules are saa poo gather imai far land aequston deta These a show al SM pe cal ated nature of gradients and other detail require may be 1 cm = 40 mor less. —— a HIGHWAY ALIGNMENT AND SURVEYS Derailed design for eross drainage and musonry structures are usually drawn tg of tem = Im, For details of any complicated portion of the structure enlarged vo Sem = 1m or upto half full size may be employed. However the size of q Should not exceed the standard size. Cross sections of streams should be to a seal 9 Jess than Lem = 10m shoulders, islands etc. to scale. Land plans for quarries. Where quarries for construction materials are to be acq for new projects, separate land plans should be prepared. The size of these maps scales may be similar to those suggested under land acquisition. ‘3.3.2 Estimates The project estimates should consist of general abstract of cost and detailed estin for each major head, Ifthe project work is proposed to be executed in stages, the estima should be prepared for each stage separately. 3.3.3 Project Report The project report forms an important part of the project document. It should cont information such as (i) general details of the project and its importance _ (ii). feature of the road including selection of the route, alignment, traffic, etc. (ii) road design and specifications (iv) drainage facilities and cross drainage structures (v)_ materials, labour and equipment (vi) rates (vii) construction programming and (viii) other miscellaneous items like diversion roads, traffic control, road side amenit rest houses, etc. 3.4 HIGHWAY PROJECT 3.4.1 General In a new highway project, the engineer has to plan, design and construct either & work of new roads or a road link. There are also projects requiring re-design a7 & alignment of existing roads of upgrading the geometric design standards. 3 Once a highway is constructed, development takes place along the adjoining lane’ subsequent changes in alignment or improvements in geometric standards become ip difficult. A badly aligned highway is not only a source of potential traffic hazard, bt ‘causes a considerable increase in transportation cost and strain on the drivers passengers. Therefore, proper investigation and planning are most important be project, keeping in view the present day needs as well as the future developments 5 region. Zl HIGHWAY PRoECT 3.4.2. New Highway Project ° ‘The new highway project work may be divided into the followin sages . _ va (i) Selection of route, finalisation ohighay alignment and geomet de (ii) Collection of materials and testing Bs OTe If i 5 mix design of pavement materiaieond ea sata other constuction materials ls and design details of pavement layers. (ii) Construction stages including quality control Route selection The selection of route is made keeping in view the require geologial, opegzapial and tr feaues of he hale apstcd Ans However special care shoud be taken as regards the geometric design standards of the road for possible upgrading of speed standards in future, without being necessary to re= align the road. After the alignment if finalised, the plans and working drawings are prepered. The geometric design requirements of highways have been given in Chapter 4 ‘and the details and requirements of hill roads are given in Chapter 12. Materials and design ‘The soil samples collected from the selected route during the soil surveys ae tested in the laboratory in order to design the pavement thickness required and the design of embankment and cut slopes. The basic construction materials such as selected soi, aggregates etc. are collected from the nearest borrow pits and quarries and stacked alons the roads alignment after subjecting these materials to the specified laboratory test. In ‘order to design the mixes for the pavement component layers and to specify quality control test values during road construction, mix design tests are carried out in the laboratory. “The possibility of using low-cost construction material like soilaggregate mixes, soft aggregates, stabilized soil and pozzolanic concrete mixes, in the sub-base or base course specification and construction contol tests should be sticlly followed. The pavement thickness is designed based on anticipated traffic, stability nd drainage conditions of he subgrade and the type and thickness of pavement layers chasen for the consustion, in India, the CBR method has been ‘by the Indian Roads Congress = designing the thickness of flexible pavements. ‘Recommended procedure for the design of cement cpnerete payment has also been specified by te Indian Roads Congress, (Please ‘see Chapter 7 for details). Construction ‘The construction of the road may be divided into two Sages. ite eee pavement construction. The earth work consis of excaylon 21) Sr tmbankment. uring the excavation for gbwaY Say embankments may Be ‘and construction of drainage network are taken os oh es aye "ander controlled ‘est constructed by rolled-fill method by COMPaCING Oe rents, the stability aaa oar razy using suitable roles. Inthe C25 ONT Te embankment ofthe embankment foundation and slopes ard th POS with time have to be investigated. a HIGHWAY ALIGNMENT AND SURVEYS. The pavement construction is subsequently taken up starting With the preparation « subgrade and the construction of sub-hase, base and surface Courses ofthe pavement Steps in a new project work “The various steps in a new highway projeet may be summarised as given below : (i) Map Study’: with the help of available topographic maps of the area, i) Reconnaissance 8 identification of inspection of the si Survey: a general idea of a topography and other features, ils and survey of construction materials, by an ond (iii) Preliminary Survey : Topographic details and soil survey along. alte alignments, consideration of geometric design and other requirements of alignmer preparation of plans and comparison of alternate routes; economic analysis selection of final alignment. ‘Typical plan, longitudinal section and cross sectg drawing for the new alignment are shown in Fig, 3.3 a & b. (iv) Location of Final Alignment : ‘Transfer of the alignment from the drawings to th ground by driving pegs along the centre line of finally chosen alignment; setting ‘out geometric design elements by location of tangent points, apex points, circul and transition curves, elevation of centre line and superelevation details. (¥) Detailed Survey : Survey of the highway construction work for the preparation longitudinal and cross sections, computations of earth work quantities and oth construction material; and checking details of zeometric design elements. (vi) Materials Survey : Survey of construction materials, their collection and testing. and bridges, and pavement layers. (viii) Earth Work : Excavations for highway cutting and drainage system, construction embankments. (ix) Pavement Construction : Preparation of subgrade, construction of sub-base an surface courses. (x) Construction Controls : Quality control tests during different stages of constructions and check for finished road surface such as unevenness, superelevation and extra widening of pavements at curves. 3.4.3 Re-alignment Project Most of the present highways in India have been upgraded in stages, from the exis they are found deficient in the geometric design elements for the present day autor traffic. There are several stretches of National Highways in the country having. lane of carriage way, narrow bridges and culverts and many locations with horizontal curves and avoidable zig-zags, steep gradients and inadequate sight distane ‘These defects are to be rectified as early as possible atleast in roads of greater impomtty like National and State Highways, It will be worth while to adopt more liberal val geometric design parameters than the ruling minimum values specified, whet® conditions are favourable and the costs involved are not excessive. In HIGHWAY PRoseey & : z E i z for Fig.3.3 (a) Typleal Drawi would be possible to upgrade the highways if mecesary in fue by insteaing Ye ith standards only, but without the necessity of realigning the road, Howe¥et 0 it situations and in difficult terrain, it may not always be economical to impro highway geometric to the recommended design standards. fational Highways should as far as possible be speed, fulfilling the comfort and safety Mneeds. To achieve this objective, itis f roads wherever deficient, to the has been decided as a policy that Nation able to fully cater to the traffic moving at desien requirements, both forthe present and future tafe ne necessary to plan improvements in the geomet 66 HIGHWAY ALIGNMENT AND SURVEYS HIGHWAY Prosey (ii) While improving the vertical overtaking sight distance at samy the stopping sight distance sh ja of the oud The conetons of min an ce ee tt conn involve high cost and soit is desra for shock-free movement of vehicles nay be checked for comfort condi vehicles during night driving 9, hese POwO UEVELS. : exon Ke to provide suitable vertical transition curves A travelling atthe design speed. Valley curves ion and for visibility under the headlights ofthe GROSS SECTION NO 3.AT SkM.300q “™P*Rm Fig. 3.3 (b) Typical Drawing for a Highway Project extent economically practicable along with the other im the pavements, construction of overlay of raising the r (iii) The road stretches which remain submerged under water even for a shor duration ff the gear or those which are in water logged areas should be raised before strengthening or widening pavement section. The formation level should be ruised such that the subgrade is at least 0.6 m above the HFL. Suitable measures should be ‘adopted against water-logging and care should be taken to provide suitable drainage facilites, including the cross drainage works. provements such as re-surfacing road above the flood water level, Necessity of re-alignment ‘The re-alignment of existing roads may be necessary inthe following cases: iv) While reconstructing bridges of length greater than 60 m on sites other than the existing ones, separate surveys should be carried out for the selection of suitable sites. The selection of site for major bridges would be governed by the river ‘raining works, sub-soil conditions for foundation and hydraulic considerations However, in small bridges the road alignment would essentially govern the bridge site selection. (v) The deciding factor which is being considered for providing over-bridzes or unter bridges for a National Highway across a railway level crossing is the produc. of number of gate closures and the intensity of traffic on the highway in tonnes pr day in the design year. When this product exceeds $0,000 or when the lever crossing is within the shunting limits of a railway station, the grade separation is justified. The location is decided keeping in view the highway alignment, the topographic and other site conditions (vi) The necessity to provide altemate routes to bypass through-trafic is assessed from the origin and destination studies. Ifthe bypassable traffic is more than the waffic terminating at the town oF built-up area then the bypass may be justified. (@ Improvement of horizontal ali hor ignment design elements, such as radius, superelevation, transition curve, clearance on inner side of the curve of shifting the | curve to provide adequate sight distance, elimination of undesirable zig-zags, etc. ee Gi) Improvement of vertical alignment design elements like steep gradients, changes in summit curves to increase sight distance, correction of undesirable undulations like humps and dips, ete. Giii) Raising the level of a portion of a road which is subjected to flooding, submergence or water-logging during monsoons. (iv) Re-construction of weak and narrow bridges and culverts and changes in water-way | at locations slightly away from the existing site. (v) Construction of over-bridges or under-bridges at suitable locations across a railway. line in place of level crossing or across another road to provide grade separated. inter-section. Steps in the Re-alignment Project (Reconnaissance ofthe stretch of road to be re-aligned, study ofthe deficiencies and the possible changes in alignment. (i) Survey of existing road recording the topographic features ands oe! Sr features including drainage conditions along a strip cea road. The width of the land 1 be surveyed depends onthe snow Ot anticipated when the road is re-aligned. The field work may plane table and level or by tacheometry. is ero levels Ci) Observations of spot eels lon the centre line fh ands ssi Ines at aabl eras to note the gale st oP, Pe A section levels should be taken at closer interval and at cross drainage works. dias os SET lw i the stretches of land th reson vans ies (iv) Sosy a ese ‘of typical soil profiles aftr testing the 50) pl laboratory (vi) Re-alignment required due to a portion of the road being submerged under water at the reservoir area on account of construction of a new dam. (vii) Construction ofa bypass to avoid the road running through a town or city. (vill) Defence requirements. General Principles of Re-alignment (While improving the horizontal alignment of roads, improvement in sharp curves and zig-zags should be done after considering the whole alignment and not on pie meal basis. The improvement of transition curves would not generally be V8 costly and therefore the defects should be rectified where-ever necessary. The sit distance available generally gets increased when the horizontal alignmen’ © improved; otherwise the set back distance may be increased at horizontal Curves 2 removing or shifting the obstruction from the inner side of the curve. UPI" desired extent. HIGHWAY ALIGNMENT AND SURVEYS 8 f and considerations of feasibility of altemate prop @ cone and spilt) of stretches which are difficult forthe re-alignms realig : (oi) Finalisation ofthe design features of re-aligned road stretches. i i ings showing plan, longitudinal sec sii) Preparation of drawings (Typical drawings show (si) Me section fora realignment project are shown in Fig, 34) a LONGITUDINAL SECTION. joRtzomtaciem «100m VERTICAL som 1d me 3 4 CROSS SECTION Stele em 5. (vi) Fig. 34 Re-alignment Project & iil) Marking out the centre line of re-li te 2 1 oad to the maximum then pare Te road while trying to utilise the ex (x). Earth-work and ic out and construc er Of subgrade ofthe re-alignment road stretches c ew bridges and culverts, PROBLEMS (i) Plan showing exiting rad, proposed signer, contours and al ater feeues of importance. (ii) Longitudinal section showing natural ground elevation, sutace of the existing road and the grade line for the re-construction, Cross section showing the existing roadway and new roadway drawn at 250 imtervals on straights, at the begining and end of tanition carves anda den middle of circular curves. Cross sections are drawn at 5O m interval where the new carriageway falls emirely outside the existing one. A typical set of such drawings is shown in Fig. 3.4. The drawings along with « comprehensive report to justify the re-alignment project should be prepared. REFERENCES Indian Roads Congress, Standard Scales and Standard Sizes of Drawings for Highway projects, Journal, Indian Roads Congress Vol. XI-l, 1956-47. Bruce, A. G., Clarkeson, J., Highway Design and Construction, International Text Book Co., Scranton, Pennsylvania, U.S.A, . ECAFE, Report of the Seminar on Low Cost Roads and Soil Stabilization, New Delhi, 1958, Jones, R. T., Layout of Roads, Journal, Indian Roads Congress, Vol. V, 1940-11 Indian Roads Congress, "Manual for Survey, Investigation and Preparation of Road Projects", IRC Special Publication : 19, 1981 PROBLEMS What are the various requirements of an ideal highway alignment. Discuss briefly. Explain with sketches the various factors controlling the alignment of roads. Explain obligatory points. With sketches, discuss how these control the aligament. Discuss the special care tobe taken while aligning hil roads Briefly explain the engineering surveys needed for locating a new highway. What are the uses of map study in engineering surveys for highway location ? ‘What are the objects of reconnaissance in engineering surveys ? Discuss the scope of aerial survey for the purpase, What are the various objectives of preliminary survey for Losi acne Enumerate the: details to be collected and the various steps in ‘method, . Discuss the general principles in the re-alignment of a HIGHWAY ALIGNMENT AND SURVEYS, Discuss the scape of aerial surveys in preliminary survey for hi What are the steps to be followed ? ishway Explain how the final location and detailed survey of a highway are cartied Give the details of drawings to be prepared in hij i recommended scales and size of the drawings. ay Explain briefly the various stages of work in a new Highway Project, ‘What are the conditions which necessit it i ‘ ge ssitate taking up of a re-alignment project work is carried out. Y) Chapter 4 Highway Geometric Design INTRODUCTION 1 Importance of Geometric Design “The geometric design of a highway deals with the dimensions and layout of visible features of the highway such as alignment, sight distances and intersections. “The geometrics of highway should be designed to provide optimum efficiency in traffic operations with maximum safety at reasonable cost. The designer may be exposed to either planning of new highway net work or improvement of existing hichways to meet the requirements ofthe existing and the anticipated traffic. 11 is possible to design and construct the pavement of a road in stages; but it is very expensive and rather difficult to improve the geometric elements ofa road in stages at 3 Tater date. Therefore itis important to plan and design the geometric features ofthe road during the initial alignment itself taking into consideration the frre growth of eafic flow and possibilty of the road being upgraded toa higher category or toa higher desien speed standard at a later stage Geometric design of highways deals with following elements: (i). Cross section elements (ii). Sight distance considerations Horizontal alignment details f (iv) Vertical alignment details {y) Intersection elements the width of pavernent, formation sent are included. The sight fal and vertical curves and at Under eros section elements, the considerations fer 1 and land, the surface characteristics and eros slope of POT distance or clear distance visible dead of driver =D intersections govern the safe movements of vie} Ss na oe “The change in the road directions are made eee yan porn ewes Super-elevation is provided by raising ne eres ore 1 Oe n traversing a horizontal curve; extra centritgal force developed oak iain er to introduce the ‘centrifugal wid is abo provided On Manton curves ae introduced Between the sy 1 mt ly en eye mt ee alignment ofa highway. Design of road intersecfons with facilities for safe and effcicy traffic movement needs adequate knowledge of traffic engineering. Highway geometries are greatly influenced by the topography, locality and trate cies and the requirements of design speed. The factors which contol ty ometrie design requirements are speed, road user and vehicular characteristics, desig sere trafic capacity and benefit-cost considerations. However, speed is the faci suhich i important governing most of the geometric design elements of roads, as may ‘seen from the subsequent articles of this chapter. HIGHWAY CROSS SECTION ELEMENTS B Fterrain with general cross slope of 10 to 25% is 80 kmph and that nous terrain “ith cross slope 25 to 60% is $0 kanph. As the cred standards affect ever peomeic design element, topography also affects the geometric desig of roads. Further in tilly terrain, it i necessary to allow for steeper gradients and sharper horizontal curves due to the construction problems. (il) Traffic Factors The factors associated with the traffic that affect geometric design of roads ate the 5 vehicular characteristics and human characteristics of road users. Its difficult wo decide the design vehicle or the standard traffic lane under the mixed traffic flow condition prevalent especially on urban roads of developing countries. This is a complex problem. ‘The different vehicle classes such as passenger cars, buses, trucks, motor cycles, et. have different speed and acceleration characteristics, apart from having different dimensions | and weights. However, it is often necessary to consider some standard vehicle as the design vehicle. The important human factors which affect traffic behaviour include the physical, mental and psychological characteristics of drivers and pedestrians. 4.1.2 Design Controls and Criteria ‘The geometric design of highways depends on several design factors. The impor ‘of these factors which control the geometric elements are : (i) Design speed (ii) Topography (iii) Traffic factors (iv) Design hourly volume and capacity (¥) Environmental and other factors. () Design Speed (iv) Design Hourly Volume and Capacity ‘The traffic flow or volume keeps fluctuating with time, from a low value during off- peak hours to the highest value during the peak hour. It will be uneconomical to design the roadway facilities for the peak traffic flow or the highest hourly traffic volume ‘Therefore a reasonable value of traffic volume is decided for the design and this is called the design hourly volume, ‘This value is to be determined from extensive traffic volume studies as discussed in An. 5.2.3. The ratio of volume to capacity affects the level of service of the road. (6) Environmental and Other Factors ‘The environmental factors such as aesthetics, landscaping, air pollution, noise pollution and other local conditions should be given due consideration inthe design on Foad geometries. Some of the aneral high speed highways and expressways are designed for higher ‘speed standards and uninterrupted flow of vehicles by providing grade separated intersections and controlled access. 4.2. HIGHWAY CROSS SECTION ELEMENTS: Design of almost eve speed. For 'y Beometric design element of a road i ’ 4.2.1 Pavement Surface Characteristics section elemento be a temen of the cnvemient surge cat on te dei ie " requirements the ab laered a5 width and clearance requir characteristics, the 61085 transition curve length and te es woah ‘quirements, the sight distance al “The pavement surface depends on the pavement type whichis decided based on the availability of materials and funds, volume and composition of traffic, subgrade, and cTanatie conditions, construction facilities and cost considerations. The important surface ; dius of | i ‘curve lengths-all th lenments such aus OF curve super-elevation, f ar vese depend mainly on the design ised 4s gradient, summit and valley _ ‘Characteristics of the pavement are the friction unevenness, light reflecting ‘characteristics mee aphy peed of the road, é and drainage of surface water. 1¢ topography or the terrai ‘ih Significant Tye in conditions Friction : the ali terrains are 's influence . a i i svement surface is one of the factors semen pin aig, BE nh gee HN Te, isin beremn wei ne reurements stoping ad sce in attards specified nous and ral slope of the country across. ibd it i curve, the lateral friction developed ‘terrain classitic ied for different classes wee? (eFFains as . The the vehicles. When a vehicle negotiates @ horizonts! cur’ Sannin tein With gat For exampie the quest? & Toads are difteeee deen 442. the vehicles. meagan force and thus govers the safe rATog SPECT Og ral cross the desi ads are different d ‘on the sale 1 ay and retardation abilities of vehicles. The slope upto 1084 ie os speed of NH Sp ain force is an important factor in the acceleration an Ph Whereas the speed on rolling WGUNWAY GEOMETRIC DESIGN ™ ortored by the paventent sure UNMer Vag eteretce to the safety. The maxing Ine braking efficiency is high enough ion of brakes, at low speeds. of tietion or the skid resistance sooty aatace coins is rnp’ with Soetfiieat of fiction comes into play only whe partly ares the rotation ofthe WATS oP applica sa occurs whan the slide without revolving oF when the coetticient wheels partially revolve read surface is more than the eircumferentia} when the path travelled along the road sur i oa + aimisot the wheels due to thei otaion. When the brakes ae applied, the Whee soared partially or filly, and ifthe vehiete moves forward, the ongitudinal skidding takes place which may vary from 010 100 percent 7 White a vehicle negotiates a horizontal curve ifthe centrifugal force is greater th the counteracting forces (i. lateral fiction and component of gravity due 10 supe ‘evation) lateral skidding takes place. The lateral skid is considered dangerous as Schiele goes out of control leading to-an accident. The maximum lateral skid coefMici ally equal to or slightly higher than the forward skid coefficient in braking tests: ‘tip occurs when a wheel revolves more than the corresponding longitudinal move ‘along the roads. Slipping usually occurs in the driving wheel of a vehicle when U Vehicle rapidly accelerates ftom stationary position or from slow speed on paver surface which is either slippery and wet or when the road surface is loose with mud, Factors affecting friction or skid resistance ‘The maximum friction offered by pavement surface or the skid resistance depends: upon the following factors (i Type of pavement surface namely, cement conerete bituminous, WBM, earth surface etc. (ii). Macro-texture of the pavement surface or its relative roughness, (iii) Condition of pavement namely, wet or dry, smoothened or rough, oil spilled, mu or dry sand on pavement. 5 tyre (¥) Speed of vehicle (vi) Extent of brake application or brake efficiency (ii) Load and tyre pressure (vii) Temperature of tyre and pavement, and (ix) Type of skid, ifany «ae aps af eran used andthe mix design of pavement surface course affect 'vements, particularly inthe case of old or wom out pavements; wae “The came feduces considerably when the pavement surface is lyre pressure and load, Smooth a {ise decreases slightly with increase in tempera pavement then new res wh ng et Out tes offer higher fition factors on A Pavements new tyres with pod weais See of lage areas of contact. But 0” ‘e higher friction factors than worn out tyFe MIGHWAY CROS SECTION ELEMENTS 1S This is because the lubricating effeet of water on the wet pavenient is reduced as the water rapped between the lyre and pavemient escapes into the treads OF the tyFe. Hence ew tyres are more dependable than smooth ones in adverse surface aid other conditions prone to skidding, such as wet pavements. ‘the minimum anticipated value of eoeicient of friction under worst possible pavement condition is generally taken for design purposes. ‘The friction coefficient decreases with skid speed, which in turn depends on the speed of vehicle and brake efficiency. For the calculation of stopping distance, the longitudinal friction coefficient values of (0.35 to 0.40 have been recommended by the Indian Roads Congress, depending upon speed, See Art, 4.3.2. These values have been suggested keeping in view the minimum coefficient of friction in the Longitudinal direction on wet pavements and afier allowing 2 suitable factor of safety. Further when a longitudinal friction coefficient of 0.40 is allowed for stopping the vehicle, the resultant retardation is 3.93 mvsee* which is not 100 ‘uncomfortable to the passengers, In the ease of horizontal curve design, the Indian Roads Congress has recommended the lateral coefficient of friction of 0.15. This low valuc of transverse skid resistance has been suggested considering the worst possible surface condition such as mud on ‘pavement surface at horizontal curve with super-elevation, during the rains, as it is essential to prevent possible lateral skid, even under adverse pavement conditions. Pavement unevenness Higher operating speeds are possible on even pavement surfaces with less undulations than on uneven and poor surfaces. Pavement surface should hence be maintained with minimum possible unevenness so that the desired speed can he maintained in conformity ‘with other geometric standards. Pavement unevenness also affects vehicle operation cost, ‘comfort and safety. Fuel consumption and wear and tear of tyres and other moving parts jncreases with inerease in pavement unevenness. Loose road surfaces increase the “ractive resistance and hence eauses increase in fuel consumption. Uneven surfaces also increase fatigue and accidents. “The pavement surface condition is commonly measured by using an equipment called Bump Integrator, in terms of unevenness index, which isthe cumulative measure of vertical ‘undulations of the pavement surface recorded per unit horizontal length of the road. For example, unevenness index may be measured in em per km. It has been shown from the teste that itis desirable to keep the unevenness index low, and preferably less than 150 ‘cavkm for good pavement surfaces of high speed highways. A value of 250 cm/m is Satisfactory upto a speed of about 100 kmph Value more than 350 envim is considered very uncomfortable even at speed of 50 kmph. (See AR. 10.4.2 for evaluation of pavement Surface condition). Pavement undulations are also some times measured using 2 straight edge in terms ofthe extent of numberof depression or ruts along and across the pavement ‘An Unevenness Indicator has been designed and patented by the Central Road Research Institute, New Deli. This equipment is useful 0 indicate unevenness values from 3 t0 20 men ‘The unevenness or undulations on pavement surface may be caused by various Fsios. such me (inadequate or improper compaction ofthe fill, subgrade and payee layers {iy unscientific constuction practices including the use of boulder stones, and brick 8 Sling course over loose subgrade sol (ii) use of inferior pavement materials () improper surface and subsurface drainage () use of imran consiruction machinery (vi) due to combination of causes. poor maintenance practices and (vi localized floes Black top pa t! P Pavement surf When the surface ieee * . especially 4.2.2 Cross Slope or ‘Camber Wry Of Water into the bitu causes stripping of bitu ie pavement layer. contact with wate1 iminous pavement layers, deterioration of th imen from the aggregates allow the pavement to get dry soon after the rain: gets considerably decreased under wet condit for vehicle operation at high speeds, the skid resistance of the pavement ion, rendering it slippery and unsafe Usually the camber is provided on the straight roads by raising the center of the carriageway with respect to the edges, forming a crown or highest point on the center line At horizontal curves with super-elevation, the surface drainage is effected by raising the outer edge of pavement with respect to the inner edge while providing the desired super- elevation. The rate of camber or cross slope is usually designated by 1 in m which means that the transverse slope is in ratio I vertical to m horizontal, Camber is also expressed a5. a percentage. If the camber is x%, the cross slope is x in 100. The required camber of a pavement depends on : (ithe type of pavement surface, and Gi) the amount of rainfall is i i i it surface li ‘A flat camber of 1.7 to 2.0% is sufficient on relatively impervious pavement cement concrete or bituminous concrete. In pervious surface like water ond mec " earth road which may allow surface water to get into the subgrade soil, steeper cross sIOPE required. Steeper camber are also provided in areas of heavy rainfall. i i be adopted keepi inimum camber needed to drain off surface water may : dem we oF: of pavement surface and the amount of rainfall in the locality. T00 8 cross slope is not desirable because of the following reason: 3 vehicles causes uncomfortable side thrust and @ i piles, Also the thrust on the wheels along the paver | wear of the tyres as well as road surface. (@) Transverse tilt of steering of automo! more causing unequal n THON ELEMENTS HIGHWAY CROSS § iy Discomfort causing throw of vehicle when grossing He tom” during, overtaking operations. ) Problems of toppling ov iv), Formation oF eross rats due 0 1 of most of the vehicles to travel along the center ine ser of highly laden bullock earts and trucks flow of water. (v) Tendeney ‘Shape of eross slope he camber Is given a parabolic elliptic or sta ne rs ses panbalic o eliptic shape given so thatthe profile is Aa at he mie and seer wards the edges, whieh is prefered by fast moving vehicles as they have 2 eye) vous the erown line during overtaking operation on a two lane highway a ight line shape in the cross section. (8) Strayga line camber: ano.ie anes a Sreaieny Seco ricnennasc leet (6) Combination ofstraxght and parabolic shape Nate: vical seule ae erlatge in th bore setches Fig, 4.1 Shapes of Cross Slope ‘When very flat cross slope is provided as in cement concrete pavements, straight line shape of camber may be provided as shown in Fig. 4.1b. Steel tyred wheels of animal drawn vehicles can cause considerable damage to the pavement surface due to high stresses. ‘The wheel does not have full contact increasing further the contact stress under these steel tyred wheels when the vehicle travels along the center of the pavement with straight camber. Some times a combined camber with parabolic. central portion and straight line camber at the edges as shown in Fig. 4.1 c is preferred. ‘The values of camber recommended by the IRC for different types of road surfaces are given in table 4.1. A range of values are given with a view that in localities with lower ‘rainfall a later chamber and in places with high rainfall, a steeper camber can be adopted. Table 4.1 Recommended values of camber for different types of road surfaces a Range of camber in No. ‘Types of road surface areas of rainfall range Heavy to Light {Cement concrete and high ype bituminous surface Thin bituminous surface 7 2 3. {Water bound macadam, and gravel pavement 4. |Earth’ 1 in $0 (2.0%) to 1 in 60 (1.7 %)| Lin 40 (2.5%) to 1 in 50(2.0%)l 1 in 33.0%) 10 1 in 40 (2.5%) 1 in 254.0%) to 1 in333.0% RIC DESIGN B HIGHWAY GEOME A>" # INGHWAY CROSS SECTION ELEMENTS In the case of tworlane pavernent of ‘The ross slope for shoulders should be 0.5% steeper than the er088 slope of ajo fic would be 1.06 for the pavement, subject to a minimum of 3.0% (and a maximum value of 5.0% fo shoulders). obtained as shown in Fig. 4 2a. Esoarhat mum clearance between Iwo Janes of tral road, as shown in Fig. 4.20. 068m svidih 0.7 m, at widest velicles on the Providing camber in the field For providing the desired amount and shape of camber, templates of camber bo are prepared with the specified camber, These are used to check the lateral profile g finished pavement during construction. Depending on the shape of the camber chosen porta the camber board may be prepared. Forming a straight line camber is very simple. nth case of parabolic camber, the general equation y= y"/a may be adopted, 3 Here a © nW/2 for a pavement of width W and cross slope 1 inn. Hence. y (4) Example 4.1 In a district where the rainfall is heavy, major district road of WBM pavement, 3.8 m__ wide, and a state highway of bituminous concrete pavement, 7.0 m wide are to be constructed, What should be the height of the crown with respect to the edges in these two cases? ee Solution For WBM road Provide a camber rate of | in 33 as the rainfall is heavy. Rise of erown with respect to edge Fig. 4.2. Lateral Placement of Vehicles ‘The number of lanes requited in @ highway depends on the predicted traflic volume and the design trafic volume of each lane. The width of pavement is inereased on horizontal curves as discussed in Art, 4.3.5. In some highways, trafic separators or medians are provided between two sets of trafic lanes intended 10 divide the traffic moving in opposite directions. In such highways the road width depends on the pavement width (or the lane widths and number a lanes) and the width of waffie separators. The width of carriageway for various classes of reads standardised by Indian Roads Congress are given in Table 4.2. Table 4.2 Width of Carriageway = 381 2 0058m 2°33 For bituminous concrete road Provide a cross fall of 1 in 50. Rise of crown with respect to the edges To = (42) be taken as the design speed. If ‘v’ is the design speed in m/sec and t” is the total reaction time of the driver in seconds, then the lag distance will be ‘v.t” metres. Equation 4,1 and 4:2 are the general equations for stopping distance at level. Stopping tance at slopes If the design speed is V kmph, then the lag distance works out to V x = ‘When there is an ascending gradient of say, + n% the component of gravity adds to the braking action and hence the braking distance is decreased, The component of gravity ‘acting parallel to the surface which adds to the braking force is equal to W sin a = Wan «= Wav/t00. .278 V.t meters. The total reaction time of driver depends on a variety of factors and a value of 2. secs. Is considered reasonable for most situations. ‘The IRC has also recommended tt value of reaction time t = 2.5 secs. for the calculation of stopping distance. Equating kinetic energy and work done, Braking distance (ow) - yw The coefficient of friction ‘P depends on several factors such as the type and 100. e of the pavement surface and tyres. Also the value of F decreases with increase in $l 5 IRC recommends the following f-values for design : ts Aes) Ae ‘Speed, kmph 201030] 40 | 50 | 60 | 65 a 0.36 Tcoeflicient of friction. f | 040 |0.3810.37| 0.36 % ‘SIGHT DISTANCE 3 5 se coetTicient of friction as 0.37 and reaction sme of driver as 2.5 second: Ass jsotstion = lag distance + broking distance siopping distance (Ea: 44) : = wt 2et Hence the general equati i equation 4,| es Bradient and may be for stopping di written as © istance may now be E Modified fy SD,m = [sata] 50. y= sokmph orv= 3 = 139 misee t= 25,8 79:8 17037 13.57 28 (f £0.01 3s »; Stopping distance = 13.9*25* 57994037 ‘When the ground is level, n = 0 Equation 4.3 may be re-writ act 'Y Be re-written as follows where the speed is V kmph and the 14m Eq. 4.3 reduces atm. = 34.84 264 the stopping distance may alo be calculated from Eq. 4.2 a8 follows : ‘Alteatively, - 4vensat SD,m = 0278 v1+ —_Y? sp = 0278M 254(F £00) 2 =61.4m Ei = 0278 x50%25+ 55537 necessary to determine the critical roads with gradients and two way traffic flow. (‘Stopping sigh distance when there re two lanes ~ stopping distance = 61.4 m The minimum stopping sight di () Stopping sight distance for two-way traffic with single lane = 2 {stopping distance] in coy a eat Aer hee hen eta a PE : ic roads when there wae lanes. On rds wih reid with or on single le rads when ret Example 43 is permi i : TRICE the wopplig distance to erable bok veloc wate hee oe ae Caleulate the minimum sight distance required to avoid a head-on collision of two cars stop. The SSD should invariably be provided throughout the length of all rads and approaching ftom the opposite directions at 90 and 60 kmph. Assume a reaction time of hence this is also known as absolute minimum sight distance. When the stopping sih seein Can GEO SES UE pra oe epee ne distance for the design speed is not available on any section ofa road, the speed should b a restricted by a warning sign and a suitable speed-limit regulation sign: However Solution should be considered only as a temporary measure and wherever possible, the stretch ‘Stopping distance the road should be re-aligned or the obstruction to visibility removed so as to pro Spring emnce Some of Pe cee (ea 4-1! atleast stopping sight distance forthe design speed. Kiceiey ~ ee w= ee The safe stopping distance values calculated in the similar manner for various 2ef speeds and recommended by IRC are given in Table 4.5. Table 4.5 Stopping sight distance values for different speeds [Design speed, kmph 20 [25] 30] 40] 5060 Safe stopping sight distance for design, m|20 | 25]30]45 [60180 v= 90 kph, v= 20. =25 m/sec Va = 60kmph, v= 2 = 16.67 m/sec AS the brake efficiency is 50%, the il ski Fe he wheels will skid through 50% of the braki ma Sonn {tae trough the remaining distance. Therefor, the value of coefficient of Fel ol aiescipig pile ited ogg, {veloped F may be taken a5 50% of the coefficient of fiction, ie. f= 0.5 x 0.7 way traffic on a two lane road (b) two way tra for design speed of 50 kmph for (2). fic on a single plane road. a 94 HIGHWAY GEOMETRIC DESIGN ‘The stopping distance forthe first car SD, Beoano35 7 '53.6m For second car, SD2 = 1667x254 1667 _ ans / 298x035 | 82m {ht distance to avoid head-on collision of the two approaching cars = SD) +SD2= 153.6 + 822-2358 m, Example 4.4 Calculating the st for a design speed of Solution ‘oping sight distance on a highwa ly at a descendi ie 80 kmph. Assume other data as pe IRC recommenionens Total reaction ti - reas: tion time t may be taken as 2,5 seconds and design coefficient of fr V = 80 kmph; n~—-2% =~ 0.02, G=9.8 m/sec” 80 v2 = =22 3.6 a SSD on road with gradient is given in Eq. 4.3 and 4.4. 2 2 7 nz ies = — $22 «25+ From Bq.4.3, SSD = vit og “22 "25 + 5a 35-00 = 5$.5+76.2= 131.7 msay 132m Alternatively, using Eq. 4.4 ve S50 = 0278 V4 Sr¢Fs00DR 80° asta, = 0278 x 80*25 + 555(935-0.02) Example 4.5 Calculate the va for a highway with a stance and (ii) Intermediate sight ¢ ues of (i) Head light sight ime suitably all the data require design speed of 65 kmph- Solution 65 kmphi 0.36, t= 2.5 secs. v f Assume ve ssp = 0.278 Vit sag7 " = 55.64 76.4 = 132: SIGHT DISTANCE 5 2 65 * x65x25+ “914 0278 x 65 x2.5+ sooo ag m (ip intermediate sight distance = 2SSD=2x91.4~ 1828 m 433 Overtaking Sight Distance (OSD) fall the vehicles travel on a road at the design speed, then theoretically there should be no need for any overtaking, In fact all vehicles do not move at the designed speed and this is particularly true under mixed traffic conditions. In such circumstances, itis necessaty for fast moving vehicles to overtake or pass the slow moving vehicles. I may hot be possible to provide the facility to overtake slow moving vehicles throughout the fength of a road. In such cases facilites for overtaking slow vehicles with adequate safety should be made possible at frequent distance intervals. ‘The minimum distance open to the vision of the driver of a vehicle intending to overtake slow vehicle ahead with safety against the traffic of opposite direction is known fs the minimum overtaking sight distance (OSD) or the safe passing sight distance lable. ‘The overtaking sight distance or OSD is the distance measured along the center of the road which a driver with his eye level 1.2. m above the road surface can sce the top of an ‘object 1.2 m above the road surface. Refer Fig. 4.13. euevet weanicas about Uae Fig. 4.13 Measurement of Overtaking Sight Distance Some of the important factors on which the minimum overtaking sight distance required forthe safe overtaking manoeuvre depends, are (@) speeds of (i) overtaking vehicle (i) overtaken vehicle and (ii) the vehicle « from opposite direction, if any. “ ar (b) distance between the overtaking and overtaken vehicles; the mini ertaking and ov les the minimum spac depends on the speeds. a (©) skill and reaction time of the driver (@) rate of acceleration of overtaking vehicle (©) gradient ofthe road, if any Analysis of Overtaking Sight Distance Figure 4.14 shows the overtakin it i 1g manoeuvre’ of vehicle A traveling at design speed, and another slow vehicle B on a two-lane road with two-way traffic. Third vehicle C ‘comes from the opposite direction. The overtaking manoeuvre may be split up into three ‘erations, thus dividing the overtaking sight distance into three pats, dy dp and dy, , 90. HIGHWAY GEOMETRIC DESIGN (41 is the distance travelled by overtaking vehicle A during the rea SIGHT DISTANCE, n the driver from position Aj to Ap. ction tine tg Now the time T depends on speed of overtaken vehicle and the acceleration of (ii) dz is the distance travelled by the vehicle A. from i ‘overtaking vehicle A, ‘This time T may be calculated by equating the distance dy to h eta y Aa to A3 during the overtaking operation, in time T sec. act (v T+ % aT’), using the general formula for the distance travelled by an uniformly ¥ : lerating body with initial speed vp m/sec and ‘a’ is the acceleration in m/sec” (iii) d3 is the distance travelled by on-coming moles ee a ae. vehicle C from Cy t ing overtaking operation of A, i. T secs, to Ce during dy = (b+25)= (. TH =| q ~ + 48)= | eT . a 7 Cenain assumptions are made in order to calculate the values ofd, dp and dy 2 ° ; ‘i AT In Fig. 4.14, A is the overtaking vehicle originall i i bon a erin a ’ ly traveling at design speed ¥ mi et or V kmph; B is the overtaken or slow moving vehicle moving with taifens i Yo misec or Vp kmph; C isa vehicle coming from opposite ditection at the design y msec or V kmph. In a two-lane road the opportunity to overtake depends on d frequency of vehicles from the direction and the overtaking sight distance avelable ata ‘instant. ‘f Therefore, Te ec, where s = (0.7 Vo +6) Hence, dy = (vy T+2s),m (iil) The distance travelled by vehicle C moving at design speed v m/sec during the overtaking operation of vehicle A i.e. during time T is the distance d) between positions C1 to C2. . Hence, dy vxT ‘Thus the overtaking sight distance Fig. 4.14 Overtaking Manoeuvre can = Bienes (It may be assumed that the vehicle A is forced to reduce its speed to the speed vp of Reece cere ass opportunity for safe overtaking operation. The distance travelled by the vehicle ‘during this reaction time is dy and is between the positions Ay and Az. This dista will be equal to vp x t metre where ‘t’ is the reaction time of the driver in second This reaction time ‘t’ of the driver may be taken as two seconds as an average value, as the aim of the driver is only to find an opportunity to overtake. Thus, the slow vehicle B and moves behind it allowing a space s, till there is an \ In kmph units, equations (4.5) worksout as: | OSD = 0.28 Vyt+028 Vp T+25+028V.T (6) Here Vy = speed of overtaking vehicle, kmph 1 = reaction time of driver =2 secs. V = speed of overtaking vehicle or design speed, kmph : axa.6s _ fads Te = HAE overtakes the vehicle B, and shifts back to it original lane ahead of B in position Ai a a s = spacing of vehicles = (0.2 Vo +6) dy =vpt=2vy,m - ‘A. = acceleration, kmph/sec. spacing ‘s’ of the two vehicles while moving with the speed vp m/sec. minimum spacing between vehicles depends on their speed and is given by empiric formula : In case the speed of overtaken vehicle Vb is not given, the same may be Shia (V ~ 16) kmph where V is the design speed in kmph or vp = (v ~ 4.5) misec and vis design speed in m/sec. . on The acceleration of the overtaking vehicle so be specified. Usually nis departs mo the make of the vehicle, its condition, load and the speed, Bd pi grsher ageesa The may be used for finding the maximum acceleration of vehicles st EN Ope average rate of acceleration during overtaking manoeuvre may the design speed. = (0.7 vp+6),m The minimum distance vetween Bz and A3 may also be assumed equal 10: mentioned above, If the time taken by vehicle A for the overtaking operation position A2 to A3 is T second, the distance covered by the slow vehicle B travels speed of vp m/sec. = 98 HIGHWAY GEOMETRIC DESIGN Table 4.6 Maximum overtaking acceleration at different 5 Speed ‘Maximum overtaking ace Vokmph | wmisee | Askmphisec |g seen 4 25 693 5.00 Lat 30 834 4.80 1.30) 40 U.10 445. 1.24 S50 13.86 4.00 dt 6 18.00 328 092 80. 22.20 2.56 0.72 100 27.80 1.92 0.53 At overtaking sections, the minimum overtaking distance should be (dy + dp when two-way traffic exists. On divide highways and on roads with one way regulation, the overtaking distance need be only (dj + da) as no vehicle is expected the opposite direction. On divided highways with four or more lanes, IRC suggests is not necessary to provide the usual OSD; however the sight distance on any high should be more than the SSD, which is the absolute minimum sight distance. Effect of grade in overtaking sight distance ‘Appreciable grades on the road, both the descending as well as ascending, increase! sight distance required for safe overtaking. In down grades though it is easier for Jovertaking vehicles to accelerate and pass the overtaken vehicle may also accelerate anf cover a greater distance ‘b’ during the overtaking time. el ‘On up grades, the acceleration of the overtaking vehicle will be less and hence passing will be difficult; but the overtaken vehicle like heavily loaded trucks may also decelerate| at steep ascends and compensate to some extent the passing sight distance requirement Therefore the OSD at both ascending and descending grades are taken as equal to that level stretch. However, at grades the overtaking sight distance should be greater than th minimum overtaking distance required at level. The IRC has specified the safe values of overtaking sight distance requ design speeds between 40 and 100 kmph. These values have been suggested based on tf observation that 9 to 14 seconds are required by the overtaking vehicle for the actu) overtaking manoeuvre depending on the design speed. This overtaking time may. increased by about two-third to take into account the distance covered by the vehicle from the opposing direction in the case of two-way traffic road, during the overtaki operation. The OSD values thus obtained for various design speeds are rounded of the IRC and tne recommended values of OSD on two lane highways are givet Table 4.7. Table 4.7 Overtaking sight distance on two-lane highways for various speeds ‘Time component, seconds ‘Safe overtaking sight Speed kh For overaking manoeuvre [For opposing vehicle|Toral| distance (meres) 40. 9.0 6.0. 15 165. 50. 10.0 7.0 17 235. oo 108 72 18 300. 65 Us 75 19 340 80 12.5 85 21 470. 100, 14.0 9.0 23 640 ‘overtaking Zones at every point i be str i sity for vel : But ecw Pg zones which re meat fr oneriahs ue Scat jould be sufficie “sign posts should be inst overtaking zones; this (Gy + da * ds) for two-way 10% ‘be indicated by appropriate sign posts i SIGHT DISTANCE visible ahead s that the length of road vis . struct highways in such a Way cable and there may inl a es Ny Th liom a fe where he safe oveaking distance i C1 pots should be inlet ; possible, en ars jing oF fe eller propibited” before such restricted 20ne% 28 ing*No Pass car at design speed should be given st The icles movie ot venaking are called overtaking Js, These zones it ions. 1t for safe overtaking operations ‘Mavance to indicate the start of the + dz) for one-way roads = should also similarly the ‘end of the overtaking zones Si nstaled ahead at distances specified above, THe um length of overtaking zone ‘should be three time the safe OS nee mt ¥ It is: veeds and 3 (dx + 62 + d3) for two-way rouds. 3 +e mess fvetines te overtaking eM tance; 4415 shows an overtaking zone with specifications for the positions of the sign re ore | ss Caney OSD = Overtaking sigh distance = (duds for one way tfc 2 (d+ 45 +43) for vo vay aio ‘5p; = Sign post over tking zone ahead” SPs = Signpost “end of overtaking one” Fig. 4.15 Overtaking Zones .vement width should be a“ ‘The OSD and pavemett Nt ufficient distance in Figur posts. Criteria for Sight Distance Requirements on Highway ‘The absolute minimum sight distance required throughout the length of the road is the SSD which should invariably. be provided at all places. On horizontal curves the obstruction on the inner side of the curves should be cleared to provide the required set back distance and absolute minimum sight distance. The common obstruction to clear vision on horizontal curves are buildings and other structures, trees, advertisement boards, cut slopes, etc. On vertical summit curves the sight distance requirement may be fulfilled by proper design of the vertical alignment as given in Article 4.5. At uncontrolled intersections sufficient clearances to the sight lines may be given to provide for SSD. Intermediate Sight Distance Sufficient overtaking sight distance should be available on most of the road stretches. On horizontal curves the overtaking sight distance requirements can not always be fullfilled especially on sharp curves, if the safe overtaking sight distance requirements are hhigh, In such cases overtaking should be prohibited by regulatory signs. In case of anay be made assuming both fe (©) mention the minimum length of overaking yum I i (©) draw a neat-sketch of the o ae Solution (@) Overtaking sight distance for two way traffic pie. = dit+a=d Assume design speed as the speed of overtaking vehicle A V = 70kmph 70 v= De 5g 719-4 misce we ei 360° a = 0.99 m/sec per sec. Dy = vot (Adoptt=2 secs) = 11.1 x2=22.2m Acceleration, dy = wT +25 S = (0.7 vy+6)= (0.7 x11.1 +6) =13.8m T= 2 - eet = 747 secs a 099 dp = U1 x 7.474+2%13.8= 110.5 m dy = v.T=19.4x747= 144.910 OSD = ditd2+d3 = 222+ 110.5 + 144.9 = 277.6 m, say 278m (b) Minimum length of overtaking zone = 3 (OsD) = 3 (ay + da + ds) for two-way trafic =3 «27 Desirable length of overtaking zone ~ 5 * (OSD) =5 x 278= 1390 m 2 metres above the road surfoee eH the ge one and show the positions ofthe sign po 8 = 834m SIGHT DISTANCE vo} {o The dels ofthe overaking 20 are shown in Fig 416 A reoey Peet Fig. 4.16 Overtaking Zone (Example 4.6) Example 4.7 Calculate the safe overtaking sight all other data suitably. Solution jstance for a design speed of 96 kmph. Assume OSD = (41 +43) for one-way traffic = (@y + dy + ds) for two-way traffic Vv = 96kmph ‘Assume Vp = V--16= 80 kmph and ‘A 2.5 kmph’sec. (from Table 4.7), t= 2 sees. dy = 0.28 Vpt= 028 x 80x2" 44.8 m = 028 VT +25 s = (02Vp+6)=02x80+6=22m fads _ [isaxre « = PAS = 113 ses. T x aa secs, dp = 0.28% 80% 113 +2" 22297 m dy = 0.28VT=0.28 x 96% 11.3 = 303.7 m OS.D. on one-way traffic road = dy +d) = 341.8 m; say 342m = dy +z + ds = 645.5 mi, say 646 m OS.D. on two-way traffic road 434 Sight Distance at Intersections Its important that on all approaches of intersecting roads, there is a clear view 9708 the comer from a sufficient distance s0 as to avoid collision of vehicles. This i ll be ‘more important at uncontrolled intersections. The sight line is obstructed by roses ther objects at the corners of the intersections. The arcs of sight formed by the lines of vision is called the sight triangle. See Fig. 4.17 The design of sight distance at intersections may be based on three possible conditions We THGTIWAY OFOME TINE DESIOn DESIGN OF HORIZONTAL ALIGNMENT $8 ‘a uncontrolled intersections, sufficient visthility shen be provides sachs tat the 8 Mratance of each road is atleast equal to the SED eanrespeaiding to the design en ott oad, if the sight tangle available is lees than the desivabie minima size dye ws iMvoidable reasons, the vehicles approaching the htetverion waxy te oeenes of controlled by suitable signs ‘At rotaries the sight distance should be at feast equal to the safe stopping Cistance for the design speed of the rotary. At signalized intersections, the aba three requirements ‘are not applicable. |At priority intersections where a minor road crosses # major road, the wathic on the rninor road may be controlled by stop or give-way sign to give priority to the traffic on the Inajor road. The visibility distance available along the minor toad should be sufTicien vo tnable the drivers stop their vehicles. The visibility distance along the major road depends upon the time required for the stopped vehicles approaching from the mize toad to evaluate the gaps between the vehicles on the major road, to accelerate and to cross the major road safety. IRC recommends that a minimum visibility distance of 1$ m along he minor road and a distance of 220, 180, 145 and 110m along the major and corresponding to the design speeds of 100, 80, 65 and 50 kmph respectively may be provided ()_ Sieahting she approaching vehiole to change speed + The si sutficlent to enable either one or both the re schist tangs peed Noi collision, The vehicle approaching from the minor road should slow dowal The toral reaction time required for the driver to decide to change speed may assumed as two seconds and at least one more second will be needed for making change in speed. Hence the two sides AC and BC of the sight triangle along intersection approaches upto the conflict point C should be atleast equal to ti distance covered by a vehicle traveling at design speed in two seconds. But sight distance being too less, should be increased in all possible cases. 4.4 DESIGN OF HORIZONTAL ALIGNMENT 44.1 General Often changes in the direction are necessitated in highway alignment due to obligatory points as discussed in Chapter 3. Various design factors to be considered i the horizontal alignment are design speed, radius of circular curves, type and length of transition curves, superelevation and widening of pavement on curves. ‘The alignment should enable consistent, safe and smooth movement of vebicies ‘operating at design speeds. It is hence necessary to avoid those sharp curves and reverse ‘curves which could not be conveniently negotiated by the vehicles at design speed. Improper design of horizontal alignment of roads would necessitate speed changes resulting in increased vehicle operation cost and higher accident rate. Gi) Enabling approaching vehicle to stop : In this case, the distances for: approaching vehicle should be sufficient to bring either one or both of the vehicle a stop before reaching a point of collision. Hence the two sides AC and BC off sight triangle should each be equal to the safe stopping distance. In almost. uncontrolled intersections one of the two cross roads is a preference highway or through road or a major road. Thus it is the responsibility of the drivers on minor road who would cross or enter this main road, to stop or change speed avoid collision. The traffic of the minor road is generally controlled by appropriate traffic sign. In such a case the sight distance for 2 minor road shoal atleast equal to the SSD for the design speed of that road. The sight d requirement of stopping is higher than that of condition (j) above and hence i ‘as vehicles can stop if necessary. (iii) Enabling stopped vehicle to crass a main road : This case is applicable vehicles entering the intersection from the minor road are controlled by and so these vehicles have to stop and then proceed to cross the main road. Ins ‘a situation, the sight distance available from the stopped position of the minoe should be sufficient to enable the stopped vehicle to start, accelerate and 1ain road, before another vehicle travelling at its design speed on the mam reaches the intersection, The time T required for the stopped vehicle to cross main road would depend upon (a) reaction time of the driver (b) width of th HE 4.4.2 Design Speed ‘The overall design of geometrics of any highway is a funetion of the design speed. ‘The design speed is the main factor on which geometric design elements depends. The sight distances, radius of horizontal curve, superelevation, extra widening of pavement, length of horizontal transition curve and the length of summit and valley curve are all dependent on design speed. The design speed of roads depends upon (j) class of the road and (ji) terrain. The speed standards of a particular class of road thus depends on the classification of the terrain through which it passes. The terrains have been classified as plain, rolling, ‘mountainous and steep, depending on the eross slope of the country as given below : rond (c) acceleration, and (@) length of vehicle. Thus the minimum sieht #50 “Fecal dasalfiaton | Crass Slope ofthe coum, patent fulfil this condition is the distance travelled by’ vehicle on the main rood # oie 0-10 speed during this time ‘T, Rolling — 10-25 25-60. From safety considerations, the sight distance at amcontrolled intersection Mourtsineus Preater than 60 therefore fulfil all the above three conditions. The higher of the three 2 Jaen at unsiunatived intersections at grade, except at rotaries. The IRC a The desi " of rons ore in and mini) standardied by peeds are the guidi terrains in rural areas are given i y the IRC for ditt 1 guiding criteria given in Table 4.8 reat cag, pending Feiseeptel vist entire ee However, le iy ruling design speeds st 'e conditions or economic consi minimum, ; speeds suggested for the Nat nic considerations passing through plain terrain is 100 kmph baie se Highways of our eo eran i 80 krmph, qT ‘able 4.8 Design.Speeds on Rural Highways Road classification ae sect Tmph for various terains - ii ann, Mountai : Ruling [ Min, [Rui untainous | _ Steep Tan Ga ee ee a 40 40, fajor District Roads 80 Other District Roads 65 Se [so Lan aos as Village Roads 30 | 40 | 40 | 35] 25 fobs Speed restrictions have been imj i passe . posed for heavy vehicles (oth like buses, trucks and vehicles pulling trailer en one Votes ate AIS limits are specified for different categories of and on some stretches of rural highway when warrant speeds for different classes of urban roads are: ;chicles by regulatory signs on urban ted due to safety considerations. ‘The recommended design i (i) for arterial roads 80 kmph, (ji) sub-arterial roads 60 kmph, (iii) collector streets 50 kmph and (iv) local streets 30 kmph 4.4.3, Horizontal Curves ‘A horizontal highway curve is 3 © cenal Tine of a road. When a vehicle Weve See horizontally outwards through the centre of gravity © the centrifugal force developed depends on the radius 0 ; speed of the vehicle negotiating cecarve. This centrifugal force is ComneEE, Teansverse frictional resistance ereioped between the tyres and the Pri caables te vehicle ro change the irpection along the curve and to mainiin given by the equation: the vehicle, Centrifugal force Pis wurve in plan to provide change in direc ‘es a horizontal curve, the centrifugal f f the vehicle. the horizontal curves ands p- we gk = centrifugal force, KE Here weight of the vehicle, KB radius of the circular curve, m™ < RF Ew 4 speed of vehicle, muse : 8 m/sec nits under Motor Vehicles Act. Also speed DESIGN OF HORIA 0 PIW is krarwn as te i of the vehicle, Ms ‘ois thus equal 10 ¥ /E% zontal curve has 10 wheels and centrifugal force to the weiel riparia OF AE impact faster, ‘the cenuifugal , ing on a vehicle negotiating & Ho u the outer effects the vehicle outwards about iy, Tendency tosh he vehicle laterally, omward. i i ditions agai + analysis of stability of those (wo Sk siting tr the vehicles negotiating horizontal curves below = jy Overturning fet wal force that tends the vehicle to overtum about He over wheels Fon cle to Ored in Fig, 4.18. The oveuam crrmntal curve without supere}ev 0m js_illustrat . serene to centrifugal force Ps oh this resisted BY (he fesares mace eee eight of the vehicle W and ¢ height gravity ofthe vehicle above Mineel track of the vehicle oO) ming and transverse inst overt ation are giver ithout superelev ‘The centrifu 1 to W.b2, where h is ne oa race and ih of the wheel base or the b is the wi Fig. 4.18 Overturning due to Centrifugal Force on for overtuming will occur when Ph = Wb, oF when hat there is danger of overturning when the centr fugal ratio ‘The equilibrium con prw = bi2h. This means PAW or v/gR attains a values of b/2h. (ii) Transverse skidding effect “The centrifugal force developed has also if the centrifugal resistance due to the friction. transverse direction. Refer Fig. 4.19. The equilibrium condition resistance developed is given by P=Fa+Fa=f(Ra+ Ra) = iB the above relation, fs the coefiient of friction between the tyre and the pavement surfce inthe eansveree direction, Ra and Ro are normal reactions atthe wheels A and B hntha (aq + Rp) is equal tothe weight W ofthe vehicle, as no superelevation has Been (provided in this case, _ Since Pe We centrifugal ratio PW is equal to °P'. In other words when the ‘gal ratio atains a value equal tothe coefficient of lateral friction there fs a danger of lateral skidding. the tendency to push the vehicle outwards in force P developed exceeds the maximum the vehicle will start skidding in the for the transverse skid 106 HIGHWAY GEOMETRIC DESIGN DESIGN OF HORIZONTAL ALIGNMENT 107 WAnee si oeoaNe 7 Tp rests SHADED AREAS show THE PRESSURE UNDER ‘THE NEN AND OUTER WHEELS A AND Fig. 4.19 Skidding Effect due to Centrifugal Force Thus to avoid overturning and lateral skidding on a horizontal curve, the cen ratio should always be less than b/2h and also. ee Fig. 4.20 Superclevated Pavement Section Analysis of superelevation The forces acting on the vehicle while moving on a circular curve of radius R meres at speed of v misee are (4) the centrifugal force P = Wy"/eR acting horizontally outwards through the center of gravity, CG (ii) the Weight W ofthe vehicle acting vertically downwards through the CG (ii) the frietional force developed between the wheels and the pavement counteracions ‘wansversey along the pavement surface towards the center ofthe cure “The centrifugal force is thus opposed by corresponding value of the fiction developed and by a component of the force of gravity due to the superelevation provided Figure 4.21 shows the cross section of a pavement with all the forces acting on the vehicle resolved parallel and perpendicular to the inclined road surface. Considering the equilibrium of the components of forces acting parallel to the plane, (P eos 8) the component af centrifugal force is opposed by (W sin 0) the component of grav frictional forces Fa and Fr, The vehicle negotiating a horizontal curve with no superelevation has to fully dep ‘on the coefficient of friction “f to resist the lateral skidding. The centrifugal force be enough to cause overturning or lateral skidding of the vehicle if ether the speed of th vehicle is high or the radius of the curve is less. In such a case the vehicle would skid not overtum if the value of ‘Fis less than b’2h, On the other hand the vehicle wo Svertum on the outer side before skidding if the value of b/2h is lower than “f'. “Thus relative danger of lateral skidding and overtuming depends on whether f is lower higher than b2h. If the pavement is kept horizontal across the alignment, the pressure on the 0 wheels will be higher due to the centrifugal force acting outwards and hence the reacti Rg at the outer wheel would be higher. The difference in pressure disiribution at inn and outer wheels has been indicated in Fig. 4.19. When the limiting equilbri condition for overturning occurs the pressure at the inner wheels becomes equal to 2 4.4.4 Superelevation In order to counteract the effect of centrifugal force and to reduce the tendency. vehicle to overturn or skid, the outer edge of the pavement is raised with respect to the| ‘edge. thus providing a transverse slope throughout the length of the horizontal curve. 7 transverse inclination to the pavement surface is known as superelevation or cant or ban ‘The superelevation “e" is expressed as the ratio of the height of outer edge with respect 28 horizontal width. From Fig. 4.20 it may be seen that superelevation, For equilibrium condition, Peas O=Wsin @+ Fat Fa NL = — =n ML In practice the inclination @ with the horizontal is very small and the nie of seldom exceeds 0.07. Therefore the value of than @ is practically equal to sin@..- « Henee, ¢~ tan 0 = sin 0 = © which is measured as the ratio of the relat B easure- Fig, 4.21 Analysis of Superelevation of the outer edge, E to width of pavement, B. This is more convenient to ™ é Ife i the superelevation rate and E is the total superelevated height of OU total rise in outer edge of the pavement with respect to the-inner edge = NI % 108 HIGHWAY GEOMETRIC DESIGN DESIGN OF HORIZONTAL ALIGNMENT. If the coefficient of fiction i ne equilibrium superelevation required by: The limiting equilibrium is reached when the full values of the frictional forces 1 developed and the values of Fa and Fy reach their maximum value of fx Rg and f, respectively where *P is the coefficient of lateral friction and Ra and Rp are the no reactions at wheels A and B. slected or assumed equal to zero, qua ie. if f= 0, the 0 counteract the centrifugal force fully will be given at ya vv Therefore, Peosd = Wsin@+f(Ra+Rp) "GR aR = Wein @ + £(W eos 0+ P sin ®) If superelevation is provided sceording to 7 ehedi-tiat, « WanbatWend inner wheels wil be equal; bu this will result in avery high value of supeelevation. As considerable role is played by the lateral frictional resistance in counteracting the Dividing by W cos 8, centrifugal force, it is always taken into account. In places where superelevation is not 4 . 2 i Fite gh densi dt provided due to practical difficulties, ie. where e = 0 and f= = - age ad the frictional force has to fully counteract the centrifugal ratio. In some types of intersections Po tant it is not possible to provide superelevation and in such cases the friction counteracts the Wo i-ftano centrifugal force fully; with no superelevation, the allowable speed of vehicle negotiai ‘a tum should berestricted tothe condition, eae 2 & p= SS erv~ finvte aR TR ‘The value of coefficient of lateral friction, *f is taken as 0.15 for design purpo (See article 4.1.1). ‘The value of tan 0 or transverse slope due to superelevation sel ‘exceeds 0.07 of about 1/15. Hence the value of ftan 0 is about 0.01, Thus the value (1 = f tan 6) in the abs ual 100,99 and may be approximated to 1.0, fe Hee aig SPL ea ET 4 It is possible that at some intersections, a negative superelevation is unavoidable. “Thus the superelevation ‘e’ required on a horiznntal curve depends on the radius ofthe ‘curve R, speed of the vehicle V and the coefficient of lateral friction or the transverse skid resistance f. Therefore, inorder to assess the superelevation e required, the speed is taken a5 equal to the design speed of the road and the minimum value of transverse skid resistance f for design purpose is standardised equal to 0.15. Therefore, But Example 4.8 ‘The radius of a horizontal circular curve is 100 m. ‘The design speed és 50 kmph and the design coefficient of lateral friction is 0.15. (2) calculate the superelevation required if full stra! fron is assumed to develop. (b) calculate the coefficient of fiction needed if no superelevation is provided. (©) calculate the equilibrium superelevation ifthe pressure on inner and outer wheels should be equal. Solution (8) Superelevation is given by the relation ‘Therefore, = rate of superclevation = tan @ Here e f = design value of lateral friction coefficient = 0.15 v= speed of the vehicle, m/sec R= radius ofthe horizontal curve, m = acceleration due to gravity =9.8 m/sec” Ir the speed of the vehicle is represented as V kmph, the Eq. 4.8 may be follows : 5. ye eet = = cecuations7£48) > ak IR eee ee ae i 0 PA f= 0.15; V= SO kph or v= = msec ie. ett aR R= 100m Vv. = speed, kmph R = radius, surfaces a Dts bOs pressure on inner and outer wheels to be equal or for equilibr n counteracting centrifugal force fully, f= 0 and ae DR 0.917 1275100 i.e. equilibrium superelevation rate is | in 5.1, However ihis rate of superetevation b very high, cannot be provided. Maximum superelevation |As per Equation 4.7 and 4.8, the value of superelevation needed increases. Wj increase in speed and with decrease in radius of the curve, for a constant value) coefficient of lateral friction “f. From the practical view point it will be necessary limit the maximum allowable superelevation to avoid very high values of ‘e’. Th particularly necessary when the road has to cater for mixed traffic, consisting of fasta slow traffic. In the case of heavily loaded bullock carts and trucks carrying less dense mater straw of cotton, the centre of gravity of the loaded vehicle will be relatively high will not be safe for such vehicles to move on a road with a high rate of superele Because of the slow speed, the centrifugal force will be negligibly small in the bullock cars. Hence to avoid the danger of toppling of such loaded slow. vehicles. itis essential to limit the value of maximum allowable superelevation. Roads Congress had fixed the maximum limit of superelevation in plain and 1 terrains and is snow bound areas as 7.0 percent taking such mixed traffic consideration. However, on hill roads not bound by snow a maximum superele upto 10 percent has been recommended. On urban road stretches with f intersections. it may be necessary to limit the maximum superelevations to 4.0 pél keeping in view the convenience in construction and that of turning movem vehicles, Minimum Superelevation From drainage considerations it is necessary to have a minimum cross slope 10 7 ‘off the surface water. If the calculated superelevation from Equation 4.8 od ‘equal to or less than the camber of the road surface, then the minimum supere! be provided on horizontal curve may be limited to the camber of the surface. the elimination of the crown a uniform cross slope equal to the camber is maint outer to inner edge of pavement atthe circular curve. In very flat curves with I firrce de or ve the outer bal of the pa with this evative superelevation would te cox velficient on such curves, IKE re curves beyond which narmat camber is required for curves, ape presen cross stope, she conte Fable 4.9 Radii beyond which Superetevation is not required Design speed | Ra Gkmphy [3% | 20 | 50 25 70 30 | 100 30 3 0 $70_|620_| 750 980 30 700__| 950 {11001400 100 tion {1500 | 1800 | 2360 ‘Superelevation Design Design of superelevation for mixed traffic conditions is complex problem. 2: differese vehicles ply on the road with a wide range of speeds. To superelevate the pavement urw the maximum limit so as to counteract the centrifugal force fully. neglecting the ster friction is safer for fact moving vehicles. But for slow moving vehicles this may que inconvenient. On the contrary to provide lower value of superelevation thus relying more ‘on the lateral friction would be unsafe for fast moving vehicles. As a compromise and from practical considerations itis suggested that the superelevation should be provided t2 fully counteract the centrifugal force due to 75 percent of the design speed. (by negiectng lateral friction developed) and limiting the maximum superelevation to 0.07 (except oo hill roads, not bound by snow where the maximum allowable value is 0.1) ‘Steps for superelevation design : sittin sos ithe design of pesetvation npn iy be sian ven ow ‘Step (i) The superelevation for 75 percent of design speed (v misce oc V kmphy is ‘calculated neglecting the friction cosy? . o.7svy ak TR ie 4 9) sR i) If the calculated value of ‘eis less than 7% oF 0.07 the valuc so obtained is provided. Ifthe value of e" as per equation 49 execeds 0.07 then prove the ‘maximum superelevation equal to 0.07 and proceed with steps (i) oF (i HIGHWAY GEOMETRIC DESIGN 12 jiiy Check the coefficient of friction Sep (te al vale of desin speed. developed for the maximum value of ¢ = 0.97 If the value off thus calculated is less than 0.15, the superelevation of 0.07 safe for the design speed. If not, calculate the restricted speed as given in (iv). 4 Step (iv) As an alternative to step (iii), the allowable speed (vs msec. or Va kmph) at the curve is calculated by considering the design coefficient of lateral friction and the maximum superelevation, ie., e+f = 007+0.15 2 2 y, Ms = 02 Ya = Ya gk 127k calculate the safe allowable speed, va = f0.22gR = V2.1S6R misec or Va = ¥2794R kmph 4.11) If the allowable speed, as calculated above is higher than the design speed, then t design is adequate and provides a superelevation of *e" equal to 0.07. If the allowak speed is less than the design speed, the speed is limited to the allowable speed Va kmp calculated above. i Appropriate warning sign and speed limit regulation sign are instalied to restrict regulate the speed at such curves when the safe speed V, is less than the design speed For important highways, it is desirable to design the road without speed restriction curves, as far as possible. Hence if site conditions permit, the curve should be re-alig with a larger radius of curvature so that the design speed could be maintained (§ Art. 44.4 and Table 4.10 for radius of horizontal curve). Example 4.9 A two lane road with design speed 80 kmph has horizontal curve of radius 480 Design the rate of superelevation for mixed traffic. By how much should the outer ed of the pavement be raised with respect to the centre line, if the pavement is rotated ‘respect to the centre line and the width of the pavement at the horizontal curve is 7.5 M- = For mixed traffie conditions the s fre for 58 dean oe sePeesaton shoul fully counteract the ee ‘Hence using Equation 4.9, DESIGN OF HORIZONTAL ALIGNMENT 13 ve 25K 225x480 Since this value is less than 0.07, the superetevation of 0.059 may be adopted ‘The total width of pavement B = 7.5m, ee 0.059 Raising of outer edge with respect to centre = pe Be 27. 2 0.059 = 0.22 m Example 4.10 Design the rate of superelevation for & horizontal highway curve of radius 500 m and speed 100 kmph. Solution For mixed trafic conditions, superelevation is given by Eq, 49. ve 225R V = 100 kmph R = 500m 1007 225x500 = 0.089 As the value is greater than the maximum superelevation of 0.07, the actual superelevation to be provided is restricted to 0.07. Check for coefficient of lateral fiction developed for fll speed using Eq. 4.10. 4 F2VE gfe tet 127k 127500 =. 0.157=0.07=0.087 ‘As the value is less than 0.15, the design is safe with a superelevation of 0.07. ~007 Example 4.11 The design speed of a highway is 80 kmph. ‘There is @ horizontal curve of radius 200 ‘mon a certain locality. Calculate the superelevation needed to maintain this speed. Ifthe ‘maximum superelevation of 0.07 is not to be exceeded, calculate the maximum allowable speed on this horizontal curve as it is not possible to increase the radius. Safe limit of transverse coefficient of fricti 15. Solution considering 75 percent design speed for finding the The problem may be shed by coms re 49, Superelevation or counteract the centri HIGHWAY GEOMETRIC DESIGN us ve ee a 25 2 a= ste oe 225x200 Maximum allowable value of eis to be limited to 0.07 Check forthe value of fietion developed, 0.18 a 007 127200 ; ‘As this value is greater than the maximum allowable safe friction coefficient of 0.18) ‘and also as the radius can not be increased, the speed has to be restricted. ‘ence the maximum allowable speed (Va kmph) on this curve is obtained by assumin the full value of design friction coefficient on 0.15, ‘This is given by the Eq. 4.11. Va = (2794R = 74.75 kmph ence the speed may be restricted to less than 74 or say 70 kmph at this curve. Example 4.12 ‘A major Distret Road with thin buminous pavement surface in Lvs rainfall area horizontal curve of radius 1400 m. Ifthe design speed is 65 kmph, what should be superelevation ? Discuss. Solution Using Eq. 4.9. vi 6st 225R 225% 1400 ‘The superetevation value required is only 0.0134 which is even less than the nor ‘ross slope required to drain off the surface water. The recommended camber for bituminous pavement in low rainfall area (Table 4.1) is 2% oF 0.02. The radius b ‘which no superelevation is required for a speed of 65 kmph and 2% camber is 950 m8 per the IRC (See table 49). AS the radius of the horizontal curve in this case is 1400 there is no necessity of providing superelevation; therefore the normal camber of 2461 be retained at the horizontal curve. = 0.0134 However, check for safety against centrifugal force at design speed along with ‘negative superelevation at the outer half of the pavement due to the normal camber. Net transverse skid resistance = —¢ + 002+ 0.15 = 0.13 a Convitgal ratio » 2 _65? 127K 127%1400 = 0.024 DESIGN OF HORIZONTAL ALIGNMENT us ‘As this value of 0.024 is considerably lower than the net transverse skid resistance of 0.13 available at the curve, this horizontal curve with i a a iege pensar es bee ith normal cambered section is quite ‘Attainment of Superelevation Introducing superelevation on a horizontal curve in the field is en important feature in construction. The road cross section atthe straight portion is eambered with the crown at the centre of the pavement and sloping down towards the edges. But the cross section in the circular curve portion of the road is superelevated with a uniform tilt sloping down from the outer edge of the pavement up to inner edge. These may be seen from sections at A and E of Fig. 4.24, Thus the crowned camber sections at the straight before the star of the transition curve should be changed to a single cross slope equal to the desired superelevation at the beginning of the circular curve, This change may be conveniently attained at a gradual and uniform rate throughout the transition length of the horizontal curve. The full superelevation is attained by the end of transition curve or at the beginning ofthe circular eurve. ‘The attainment of superelevation may be split up into two pars : (@) Elimination of crown of the eambered section (b)_ Rotation of pavement to attain full superelevation Elimination of crown of the cambered section ‘This may be done by two methods. In the first method, the outer half of the cross slope is rotated about the crown at a desired rate such thatthe surface falls on the same plane as the inner half and the elevation of the centre line isnot altered. (Ref. Fig. 4.22). The outer half of the cross slope is brought to level or horizontal (by rotating about the crown line) at the start of the transition curve or at tangent point T.P. See cross section at B in Fig. 4.24. Subsequently the outer half is further rotated 50 as to obtain uniform cross slope equal to the camber, as shown in Fig 4.22 (a) and in cross section C of Fig. 4.24. (a) Oueredge rotated about the crown (2) Crown shied ouneerds (dagonal crown methd) Fig. 4.22 Elimination of Crown of Cambered Section Thus no point on the curve will have a negative superelevation atthe outer half of the Pavement event at the start of the transition curve. This method has @ drawback that the surface drainage will not be proper at the outer hal, duringashor stretch of the road with 4 eross slope less than the eamber between point A and Cin Fig. 4.24 In the second method of eliminating the crown, known as diagonal erown method. the crown is progressively shifted outwards, thus increasing the width of the innes Nat of Cross section progressively. This method isnot usually adopud s POTAS OT half of the pavement has increasing values of negative superelevation on to @ port ‘he outer half, before the crown is eliminated (see Fig. 422) {UKIWAY GEOMETRIC DESIGN shin full superetevation ue shawn of the camber is eliminated, the XOENe nay be greater than the camber in’ many. ci whe bed | will have t wad ouaher till the desired banking is obtained, \s at example, ifthe specified camber in a bitumi Seo ituminous pavem nee we meting toa” te can ct lg Superelevation of 0.07. If the core farther increased til iam i satcrnent #1 the Rorizontal curve 1B the total bankiog of ihe on a a everneat wil respect tothe inner edge ssqualio =e tN Sa There are two methods of ro ; . tating the pavi superelevation after the elimination Oftiecnse en ean (i). By rotating the pavement cross secti ction about the ji . edge and raising the outer: centre line, yen ith ing the outer edge each by half the total amount ray he 1y E/2 with respect to the centre. ‘superelevation, OY By roping ee ‘pavement cross section about the inner edge of the pavement sectio s = well as the outer edge of the pavement such thatthe o edge is raised by the full amount of superelevation, E with respect tothe inner edge. ‘The two methods are shown in Fig. 4.23. Method (u) Rowating about te inner edge Method (1) Rotating about centre line Fig, 4.23 Rotation of Pavement Section to attain Full Superelevation tated about the centre line, the ver sement section is 10 wuter edge is banked and inner eda unchanged; the o in balancing the earth work, The disadvantage oF In the first method as the pav' profile of the centre line:remains Gepressed resulting in an advantage sera isthe drainage problem due to depressing the inner edge below the general The drainage problem is of greater significance in areas with high rain fall subgrade is in cutting or in level terrain, Ifthe ‘subgrade is in embankment oF toad has a significant gradient to facilitate longitudinal drainage, there Wi drainage problem. ‘The second method of rotating about the inner edge is preferably in ve hen the road is not taken on embankment, in orde! lem. But the entire pavement width and outer should idtional earth fill, In this case the raised with respect t0 pavement is also raised, which may be considered as a disadvantage of the penical alignment of the road is altered. perelevation has been shown in detail in Fig. 424 ‘The attainment of su 1 pe Juding the straight, 1d circular curves the horizontal curve inc transition an DESIGN OF HORIZONTAL ALaGNMENT Mi ut Fig. 424 Attainment of Superelevation Fig, 4.24 a. Elimination of the crown of cambered section of cam attainment of uni cee perce tn lo ae Ol person re been illsrmted in Fig. 4246, The cue half of the cambered section is raised to a ermal peste between A and B at the same rate of introduction of superelevatice ‘long the transition curve of length Ly. Thus at the tangent poi song the tars lengt atthe tangent point B there is no negative When the pavement is rotated about the inner edge, the length AB is given by cBN chy 2 2e Jevation, 8 isthe width of pavement and long the transition curee of length Ly 1 to the camber and the dissance Cant & to attain ful! ‘where ¢ and ¢ are the rates of camber and superel 'N is the rate of raising the outer edge of pavement al ‘Atpoint C the pavement attains uniform eross slope e403) C= AB, The pavement is further rotated a the same rate berwnesn superelevation. 5 HIGHWAY GEOMETRIC DESIGN is ined gradually over the ful length arta rhe ful ese shal yale ot the Saring oi ofthe a ar gn pT ot be, provided fOr SOME Ten, tw soar re ransition CON ‘sraight portion before the start of yy. fthe circular curve. ect 2 and outer ed the inner edge, centre fine ster edge by the wie a pode. It may be seen that in the conte ae sows Genial profile of the Pavement ene i averent section "ny rotating about inner edge, the levels ets cn carat above oil Yee of wae by rain the ote edge the payem wi ed “sing eran and | in 60 on mounting cxctng 1150/8 PEs of the Indian Roads Congress. Hence rai 5 pe SOc the supeelevaton Ew ion esn ie and vae aE, Tis he enh of anston cy ioe ao speeevtion E at rae of in 150 will be 150 E, ifthe ps road abou the ane ed 4A Radius of Horizontal Curve Fora cersin speed of vehicle te centrifugal force is dependent on the someon curve, To keep the centrifugal ratio within a low limit, the radius of seritit kept corespondngl high. The centrifugal force which is counterac Sopra and itera ition is given as per Eq. 4.7 and 4.8, by the relation In this equation, the maximum allowable superelevation rate has been peat (07 and the design coefficient of lateral friction ‘fis taken 4.1.2), a Hence, e+f = 097+0.15=022= 4 = eR 127K Ifthe design speed is decided for ahi i is | hh Re canbe fund fom the above relationship. fain Thus the ring mi _ 3 Vargh egies imam aus ofthe curve for ruling design speed V1 Reng Hg Also, ‘ Baty = 127(e+f) DESIGN OF HORIZONTAL ALIGNMENT he Brin = Her oy Inthe above equations, yand V = ruling design speeds, in msec and kmph respectively, y" = minimum design speed, kmph E = rate of superelevation; the maximum value of ¢ is taken 25 0.07 at all the regions except at hill roads without snow where its taken as 0.1. f = design value of transverse skid resistance or coefficient of friction, taken as 0.15 2 ~ acceleration due to gravity = 9.8 msec ‘According to the earlier specifications of the IRC, the ruling minimum radius of the horizontal curve was calculated from a speed value, 16 kamph higher than the design Need ie, (V + 16) kmph. However now the calculations are based onthe ruling and rrinimum design speeds given in Table 4.8, ‘The ruling and absolute minimum values of radi of horizontal curve of various classes of roads in different terrains (as per the latest IRC specifications) are given in Table 4,10. ‘Table 4.10 Minimum radii of horizontal curves for different terrain conditions, m a Tar Sapa TT nin | Raia [Rena] ow bond [Rio | Seow ond tut é Mod ee _Mee ea Ie TTT ang inguin tea a ra ae a ean [sin {| Oe meer et ee see ob ae te Te te Fe peste se poe pe ea woe [aes etre Pst ae ate se tate pete tee tet Tes ‘Note: The values of ruling minimum and absolute minimum radii conespond to the ruling and minimum design speed values given in Table 4 Example 4.13 Calculate the values of ruling minimum and absolute minimum radius of horizontal curva af s National Highovay in plain terain, Assume olin design speed and minimum design speed values as 100 and 80 kmph respectively Solution Roting minimum radius i ealeuated using Eq 412 0413 for ling desin speed of 100 kmph with the maximum values of ¢ = 0.07 and = 0.15. v2, __ 100? Reing = T37(erH 127(0.07+0-15) tated from the rinimum design of V" = 80 ‘= 357.9 m say 360 ‘The absolute minimum radius is eal ‘mph, using Eq. 4.14, v2 80? it paVE jie aay im Rein” Ta7eeh) | 12700074015) 120 HIGHWAY GEOMETRIC DESIGN “therefore provide rlng minimum radius of 360 m and absolute minimum radius of 230m, 44:46 Widening of Pavement on Horizontal Curves x : pecially when they are not of vey large radi iis common widen tne paverent slighty more than the normal width. he objet of proving exe | ger a pavements on Horizontal euves are due tothe following reason i (0) An automobile has a rigid whee bate and oa the front wheels can be tumed ie vehicle takes a tum to negotiate a horizontal curve, the ‘alow he = path as that ofthe front wheels. This phenomenon is called affoad felon Ty at Tow speeds and up t0 the design speed when no lateral sPPINE °F viele take place) the rear wheels follow the inaer path on the curve as comni weet he corresponding front wheels. This means tat if inner front wheel kes 9 al dase cede ofa pavement ata horizontal cure, inner rear whee! willbe off pavement on the imer shoulder. The offtracking depends on the length of the w Peet the vehicle and the turning angle or the radius ofthe horizontal curve nez0ti ‘This is illustrated in Fig. 4.25. (On horizontal curves, es Fig. 4.25 Mechanical Widening on Horizontal Curve (b) At speeds higher than the design speeds when the superelevation and Ia fiction developed are not fully able to counteract the outwards thrust due to 1h centrifugal force, some transverse skidding may occur and the rear wheels may take on the outside of those traced by the front wheels om the horizontal curves. However ti occurs only at excessively high speeds. (c) The path traced by the wheels of a trailer in the case of trailer units, is also to be on either side of the central path of the towing vehicle, depending on the sPe# rigidity of the universal joints and pavement roughness. (@) In order to take curved path with larger radius and to have greater visibili curve, the drivers have tendency not to follow the central path of the lane, but to US © outer side atthe beginning ofa curve. a (©) While two vehicles cross or overtake at horizontal curve there is a psycholol tendency to maintain a greater clearance between the vehicles, than on stra increase safety DESIGN OF HORIZONTAL ALIGNMENT ofthe avi epi the length of wheel based of the vehicle I mde of worm Ne Me < pryetolgieal factor which sa function ofthe speed ofthe vei tod he caine It has been a practice therefore to provi curves when the radius is es than abou S00 me "ith of vee horizontal “Analysis of Extra Widening on Curves ‘The extra widening of pavement on horizontal curves i (@ mechanical and (i) psychological widening. ee Mechanical widening ‘The widening required to account for the off-tracki ing due to the rigidity of wt based i clled mechricel widening (W,) and maybe enka ss gives Dim, Nee Ry = radius of the path traversed by the outer rear wheel, m Ry = radius of the path traverse by the outer front wheel, m Wm => off-tracking or the mechanical widening, m 1 = length of wheel base, m Wm = OC-OA=OB-OA=Rr-R; From ‘AOAB,OA? = 0B?—BA* RPP But Ro-We (Ro- Wr)? = RY =P ie, R222 Wmt Wm? = Re=P P = Wa@R2-Wa) 2 Wa = a. Te ais) 2 aq ermine) Here R is the mean radius of the curve, The mechanical widening calculated above is required for one vehicle negotiating a horizontal curve along one traffic ane. Hence in a toad having ‘n° traffic lanes, as ‘n° vehicles can travel simultaneously, the total ‘mechanical widening required is given by 2 af 416) Psychological widening Extra width of pavement for greater maneuverabili sso provided for psychologice! reasons such as, to provide srorcering at higher speeds, to allow forthe extra space 12 HIGHWAY GEOMETRIC DESIGN hangs of veiicles and the curves. Psychol lane. An empirical formula has been recommended by ‘widening *W,, which is dependent on the. R of the curve. The psychological widening ae ts for the over Sod overtaking vehicles on ements with more than ONE : Ta for finding the addtional psychological design speed V of the vehicle and the radius js given by the formula: requirement v We - 35 UR 4.17) Hence the toial widening We, m required on a horizontal curve is given by : We = Wat Wps a wiv ie, w= M+ IR” 95dR Hencen = number of traffic lanes. {= length of wheel base of lo be taken as 6.1 m or 6.0 m for commerc ngest vehicle, m. ‘The value of / may normall jal vehicles, if not known. ‘= design speed, kmph R= radius of horizontal eurve, m “The extra widih recommended by the Indian Roads Congress for single and two pavements are given in Table 4.11. ‘Table 4.11 Extra width of pavement at horizontal curves 4 TOI 10 300 | Above 300} Radius of curve (m)]| Upto 20] 20 to 40 [41 to 60 [61 vo 100 Exira width (em) a Tworlane isp 13 _| 12 [09 06 Nig Single-lane a9 | 06 | 06 | Nil Nil Nil the pavement widening is calculated by adding half Note: For multi-lane roads, fh lane of the multi-lane road. ‘extra width of two-lane roads to eact Methods of introducing extra widening ‘The widening is introduced gradually, starting from the beginning of the curve or the tangent point (T.P.) and progressively increased at uniform rate, till th value of designed widening *I,' is reached at the end of transition curve wh values of superelevation is also provided, as shown in Fig. 4.26. The full value of width We is continued throughout the length of the circular curve and then e «gradually along the length of transition curve, Usually the widening is equally. ie, Wel2 each on inner and outer sides of the curve, But on sharp curves of hill road5" extra widening I~ may be provided in full on inside of the curve. On horizontal circular curves without transition curves, two-thirds the wid provided at the end of the straight section, i.e, before the start of the circular eu the remaining one-third widening is provided on the circular curve beyond the point as in the case of superelevation, In such cases, the widening is provided inside ofthe curve. Refer Fig. 4.27. to provide greater clearance for crossing logical widening is therefore important Fig. 4.27 Widening of Pavement on Sharp Curve Example 4.14 Calculate the extra widening required for a i irizon ‘ pavement of within 7m on a hi carve of aus 250 m if the longest wheel base of veil enpeted on the odds vom ign speed is 70 kmph. Compare the value obtained with IRC recommendations. Solution Extra widening required We= Win + Wps 2 =o (an) Hence, n= 2 (1wa lanes for pavement with of 70 m) 1= 70 R = 250m V = 70kmph ge She © ax250 * 95250 The IRC recommends extra widen! 300m. (See Table 4.11) = 0.196 + 0.466 = 0.662 m ing of 0.6 m when the radius ofthe curve i 101 0 124 HIGHWAY GEOMETRIC DESIGN Example 415 Find the total width of a pavemer be aligned slong a rolling terrain with a ruling nt on a horizontal curve for a new national highway inimum radius. Assuime necessary Solution “Assume the following data : (i) National highway on rolling terrain, ruling design speed, V ~ 80 kmph (ii) Normal pavement width, W=7.0 iii) Number of lanes n=2 (iv) Wheel base of the truck f= 6m (v) Maximum value of superelevation e = 0.07 and skid resistance f= 0.15 Rule = > * Soy ling = D7 Qe) 127(007+0.15) = 229m, say 230m 80 9.51250 We = + pa 2R 95VR — 2*230 ap, iv 2x6? Extra = 0.157 +0.555=0.712m Total pavement width on curve = W+We=7.0+0.71=7.71 m 4.4.7 Horizontal Transition Curve ‘on curve has a radius which decreases from infinity at the tangent point ‘ular curve. When a transition curve is introduced be transition curve decreases becomes mini fF the transition cx A transiti designed radius of the circ straight and circular curve, the radius of the ‘atthe beginning of the circular curve. The rate of change of radius ‘will depend on the equation of the curve or its shape. Object of Providing Transition Curves ‘Suppose a curve of radius R takes off from straight road, and a vehicle travels om | road: then due to the centrifugal force which suddenly acts on the vehicle just aft tangent point, a sudden lateral jerk is felt on the vehicle. This not only causes discomis to the passengers, but also makes it difficult to steer the vehicle safely. Refer Fig. 4 If a transition curve BC of length L, is introduced between the straight AB and! ‘circular curve CD of radius R, the centrifugal force will also be introduced graduall the radius of the transition curve decreases gradually from infinity. The rate at which force fs introduced can be contolled by adopting suitable shape ofthe transition CU and by designing its length, so that the vehicle can have a smooth entry from the to the circular curve at the design speed. ‘A transition curve which is introduced between the straight and a circular curv help also in gradually introducing the designed superelevation and the ext i necessary. + DESIGN OF HORIZONTAL ALIGNMENT 12 “Thus the functions of transition curves inthe horizonal alignment of highway may be summed up into the following points: introduce gradually the centrifugal force between the tangent fing of the circular curve, avoiding a sudden jerk onthe vehicle ‘own comfort and security. (@) 10 (b) to enable the driver tur the steering gradually for his (to enable gradual introduction ofthe designed superelevstion and extra widening of pavement atthe start ofthe circular curve, {(@) to improve the aesthetic appearance ofthe road, tn a good highway alignment t should be posible to mania the design sets 68 on roitimal curves, ‘The radius i fist designed a discussed in article 4.3.4 and th 9 area gnape ofthe transition curve is selected and its length i designed The ‘deal aan eeetranstion curve should be such thatthe rate of inroducton of, cxotrifugal shape of 2 tran chonge of centrifugal aceleraton should be consistent, This mess force oF the rate of ee ion curve should consistently. decrease from iniity wt agent point B (refer Fig 428) to the radius Rof te ciel curve at po} the end of | Lange po etre teglh Ly In an ideal tanition curve the lng La should be inversely proportional tothe radius Rie, (Ls VR) oF Le R i consent The spiral transition fulfils this requirement. Different Types of Transition Curves ‘The types of ransion curves comnonly adopted in Rezo aignnen (@) Spiral (also called elothotd) (b) Lemniscate Cee 19, All the three curves in Fig, 4.29. A hele ct en i follow almost the same path upto. deflection <0 ost the same pat inal hese uve significant difference even UP he radius deereases as the Fength Fig. 4.29 Different Types of Transition Curves increases. But the rate of change of radius and hence the rate of change of centrifugal acecleration is not constant in the case of lemniscate and cubic parabola, especially at. deflection angies higher than 4°, In spiral curve the radius is inversely proportional to the length and the rate of change of centrifugal acceleration is uniform throughout the length, ofthe curve. Thus the spiral fulfils the condition of an ideal transition curve, ‘The Indian Roads Congress recommends the use of the spiral as transition curve in. horizontal alignment of highways due to the following reasons : (i) The spiral curve satisfies the requirements of an ideal transition. (ii) The geometric property of spiral is such that the calculations and setting curve in the field is simple and easy. The equation of the spiral may be written as : LR = Ly Re=constant mo (4.19 ‘Therefore, L Calculation of Length of Transition Curve The length of transition curve is designed to fulfil three conditions, viz. : (i) rate change of centrifugal acceleration to be developed gradually (ii) rate of introduction the designed superelevation to be at a reasonable rate (iii) minimum length by IR empirical formula. (i) Rate of change of centrifugal acceleration : At the tangent point the centrifug acceleration (v'/R) is zero atthe radius R is infinity, At the end of the transition radius R has the minimum value Ry. Hence the centrifugal acceleration F distributed over a length L, of the transition curve. The centrifugal acceleration should be developed at such a low rate that it will not cause discomfort for t passengers of a vehicle traveling at the design speed (v misec). It is evident larger the length of transition, lower will be the rate at which the centrifiGl acceleration is introduced. Let the tength of tansition curve averse this tran: asin contri at Land hen 1 men ion length at unifanin eleration of VR is i Peer The maximum allowable value of the rate of change of cent i change gal scecleration without producing discomfort or undesirable oscillation, is dependent on the ane varies inversely with the radius. The IRC bas recommended the following equtson for finding the value of C forthe design speed V kmph : . — 80 (5+¥) msec" os Ls (4.23) 46.5CR cR ey L, = length of transition curve, m C = allowable rate of change of centrifugal acceleration, msec’ as given in Eq. 421. R= radius of the cireular curve, m. (i) Rate of tmiroduction of superelevation : In open country if a high value oF supereievation is to be introduced, itis not desirable to raise the outer edge of 3 Pavement at a larger rate than 1 in 150 relative to the grade of the centre line Hence the length of transition curve should be afleast 150 times the total amount by which the outer edge of the pavement is to be raised with respect 10 the centre line. However, the transition curve Hength may be reduced ty allowing an fnereased differential gradient of 1 in 100 in built up areas an | in 60 on Bill JOMETRIC DESIGN HIGHWAY GI 128 fe line, then the total ig pavement setae aout the ier es an0 the ered in calculating veinee edge with respect to inner edge Nas 10 be cor Tength of transition curve required. Let ‘e’ be the rate of superlevation designe! as PEt Eq. 4.9 for a highway raving normal pavement width W. Let “We' Be the extra widening provided at t aapae curve so that the total wid B of pavement = (W © ‘We) and the total raising pavement wid respect co the inner edge = 6B = 6 (W+Wo=E. Ifit is assumed the pavement is rotated about centre Tine afer neutralizing the camber, (maintaining the Pavavignment of the centre Line) then the maximum amour by which the outer eri ded atthe circular curve with espect to the centre = EP. Hence allowing a we NGhange af superclevation of | in N where minimum value of N = 150 to 60 ‘fiscussed above), the length of transition curve Ls is given By = EN y= B=Swewe (424 However ifthe pavements rotated about the inner edge the length of transition cur is given by + Ll, = EN=eN(W+WO (4248) (iy By Empirical Formula : According t the IRC standards, the length of horizon Transition curve Ly should not be less than the value even by the follows ‘equations forthe terrain classifications d (a) For plain and rolling terrain (b) For mountainous and steep terrains; - ante the length of transition curve forthe design should be the highest of the three mentioned above. Therefore, the design steps are given helow : (@) Find the fengih of transition curve based on allowable rate of change centrifugal acceleration (Eq. 4.21 and 4.22 or 4.23). (6) Find the length of transition curve based on rate of change of supere! (Eq, 4.24 014.25). (c) Check for the minimum required value of Leas per Eq. 4.25a or 4.286. (d) Adopt the highest value of Ls given by (a), (b) and (¢) above as the design transition curve. 3 ‘The minimum length of transition curves for various values of radius of design speeds recommended by the IRC for plain and rolling terrains and ‘mountainous and steep terrains are given in Table 4.12. Aix) Maximum amount of superelevation, E which depends on the maximum rate of DESIGN OF HORIZONTAL ALIGNMENT ‘Table 4.12 Minimum transition tength for di it shh 4 ins treatin et Plain and rolling terrain gee a Sees ‘Curve Design speed seus a {kmph) ats | Radius Rem) [FOOT 80 | 65 [ 50. ee pes aaephi | cet s [a0 [was satin [aaa oss COREE EV cy a= Tabs stp = Tt fs [0 Tao =the ior f= T= rao se ea 150 fanbase sao Para va - | - [70740735720 50 |e 7 ax ao fsa sss =f] S0{30 {a0 kt — ay — pea sion fat [a Selassie so fife ef P= pst et usts5[sr pao} -t | woofs tos rte tehafee st} tate Cae aE St tts se tat fare tt} apo Sos Bo ttt faa tte 1200, go|Nr] - | - [=] - 400 1s] - =e 1300 [35] - [=== 1- 1 soo —Iwat= P= T= vso0 | 30/-|-|-|-|- a 2000 _[NkT-[- [-1- |= | Note : NA — Not applicable; NR ~ Transition not required ‘The length of transition curve L; required on a horizontal highway curve therefo depends upon the following factors : = (i) Radius of circular curve, R. (ii). Design speed, V (ii) Allowable rate of change of centrifugal acceleration, C (whi sae ion, C (which is also dependent on superelevation, ¢ and the total width of the pavement, B atthe horizontal curve (¥) Whether the pavement cross section is rotated about the inner edge or the centre Tine, afer the elimination ofthe camber. (i) Allowable rate of introduction of superclevation, which depends on the terran, Jocation and environmental conditions of the Setting out of transition curve When transition curves are to be provided on both ends of a circular curs. 1 following procedure may be adopted. Refer Fig. 4.28. ‘Let PCDQ be the original cireular curve of eaioe RPP” and QQ ae eq ie skit Sof the trasiton curve given Py the formula : 2 eer (426) AR HIGHWAY GEOMETRIC DI 130 shee Lis the length of transition curve and is the radius ofthe creular curve, ag vad £ QD are the two transition curves, cach of length Ls and © D is thes : arve, The length of BP* and P°C are approximately equal to L/2. The points B ang remain as tangent points to the new compound curve BP’ CDQE. In order 10 set out the transition spiral, the design details such as the radius of cingular curve R, length of transition curve La, total deviation angle A, tangent deviti rele ofthe transition 8, central angle of circular are A, tangent distance, apex dist tte, are determined, The curve may be laid cither by off-set method or by po deflection angle method. The details of calculating the off sets/deflection angles Setting out the curve in the field are not given here; they are available in the books 39 Surveying. DESIGN OF HORIZONTAL ALIGNMENT total width of the pavement athe curve, B= 7.5 a Tesal raise of outer edge of pavement with respect the cen entre line = EL eB _ oo EL eB aos 2 2 7 26 9 ate of introduction of superelevation, 1 in N= 1 in 180 | EN b= 5-026. 150=39m (©) Minimum value of Ly as per IRC (Eq. 4.25) 2.1V? _ 2.746 R 20 Adopt the highest value of the three i. i transition curve. % i.e., 51.9 or say $2 m as the design length of Example 4.16 =519m Calculate the length of transition curve and the shift using the following data : Design speed = 65 kmph Radius of circular curve = 220m "Allowable rate of introduction of superelevation (pavement rotated about the centre line) Vin 150 2 shins = be? 24R 24x220 2h Example 4.17 ‘A national Highway passing through rolling terrain in heavy rai A rain fall a hesioiad curve of radius 500 m. Design the length of wansition curve tenia vine a. Pavement width including extra widening = 7.5m Solution (@) Length of transition curve Ls as per allowable rate of centrifugal accelerat “Allowable rate of change of centrifugal acceleration as per Eq. 4.2, 80 _ 80 (I5+V) (75+65) Solution For a National Highway on rolling terrain, the followir aa Nate 1g terrain, wing data may be assumed as per Design speed, V = 80 kmph Normal pavement width, W = 7.0m Allowable rate of change of centrifugal acceleration, (range of value 0.5 o-0.8) 80 80 (5+) 75480 AS the value of C is betweei 0.5 and 0.8 itis accepted for design. Allowable rate of introduction of superelevation = 1 in 180, pavement to be rotated bout the inner edge to effect better drainage in heavy rainfall ares. (@) Length of transition curve by rate of change of centrifugal acceleration 3 3 ) “vF _ 00215 V9, 0.0215%80? «25 TR CR (052x500 (b) Length of transition curve by the rate of introduction of superetevatica: = 0.57, misec? ‘This value is between 0.5 and 0.8 and hence accepted. 3 3 = 00215 V3 _ 0.0218%65? 471 CR 057x220 (b) Length Ls by allowable rate of introduction of superelevation E : Vs 225R 225x220 As this value is greater than the maximum allowable rate of 0.07, limit the value 9 ¢=0.07. Check the safety against transverse skidding for the design speed of 65 kmph oe Superelevation rate e= 0.085, ve 65 IR 127x220 = 015-007 = 0.08 AAs this value of fis less than the allowable val ation O07 is sate forthe design speed cf S nph of 0.15, the superelev: 2 v2, _ 80° _ 29.057 (<0.07.0K) © 2R5R ” 225x500 132 HIGHWAY GEOMETRIC DESIGN 1g wo lanes and wheel base of 6 m) 5 DESIGN OF HORIZONTAL ALIGNMENT. xtra widening at curve (assumin a v 26% 80 yu = 2x8 5 We = oR * o5dR 2*500 95/500 Total widih of pavement = B=7.0+0.45 = 7.45 m Lg = 745 «0.057 x 150 = 63.7 m 133 =0.45m “Therefore half central angle is given by : ane iS SS satiang 1808 2 ay alan 8 gs ‘The distance from the obstruction to att dina ome aro th cite Rex a, There te ttc (©) Check for minimum valve of Ls by Eq. 4.25 a, 272 _ 2.780" m= R=Reos & a = = =346m * (4273) Jn the case of wide roads with two oF mo stance = it re lanes, if dis the ds es A of i od ah a Ie fe Se somes us Sei dcmee's = of the ner side lane ad he s-back dice, gen ‘Adopt the highest of the above three values = 63.7 say, 64m. ‘Therefore, the d length of transition curve is 64m. 4.4.8 Set-back Distance on Horizontal Curves 4 In the design of Horizontal alignment, the sight distance along the inner side OF8 cures should be eonsidered. Where there are sight obstruction like buildings. slop conver on the inner side of the curves, either the obstruction should be removed or ‘lignment should be changed in order to provide adequate sight distance. It may sd aeratbe possible to make some adjustments in the normal highway Cos; secon tp small deficiencies in sight distence. If'it is not possi provide the adequate Fesance on curves on existing roads, regulatory and cautionary SBM should be it saree the traffic suitably. In case of new highways forthe desian speed and ‘Gquements, the actual condition in the alignment should’ Pe checked and nee manner most fitting to provide adequate sight di m = R-(R-d)cos 2 (R-d)¢08 5 aan where (b) Les. If the sight distance required is greater than the length length of curve Le, then the angle subtended at the centre is determined with reference le centre is determined wi to the length of ci we etek cnmce beerkedon averse gen ow See a3. adjustments be made in a Specific study is usually necessary for each site condition. BL NOL. gy 2 Ine-o ‘Ag discussed in Art. 4.3, the absolute minimum sight distance which is: th stopping sight distance should be available at every section of the highway, from st point. Thus itis essential that in horizontal alignment. special care should be taken provide for the stopping sight distance; these values may be adopted as prothe design speed. Overtaking sight distance requirements are given [n Table 4, erahanee distance or set back distance required from the centre line of horizontal eran obstruction on the inner side of the curve to provide adequate sight dis depends upon the following factors : (i) required sight distance, 8 (Gi) radius of horizontal curve, R (iii) length ofthe curve, Le which may be greater or lesser than S. Refer Fig, 431. Let C be the obstruction to vision on the ianer side of Mt highvay curve ofradius R, ABC the line of sight and arc AB be the sight distance () le? S Let the length of curve Le be greater than the sight distance S, The angle by the are length S atthe centre be a. On narrow roads such as single lane sight distance is measured long the centre line of the road and the angle subten ts: centre, a. i equal to S/R radians, ¢ Gl) t Soke sin S (428) m! = R~(R=d) cos = + SS8 Deo 5 SPs ‘The clearance of obstruction upto the set-back distance is important 5 is important whe is slope on the inner side of the horizontal curve, The method SP ecclaing the stack distance is illustrated in Examples 4.19 and 4.20, 44.9 Curve Resistance The automobiles are stered by turing the font wheels, but the rear wheels do not turn, When a vehicle driven by rear wheels moves ona horizontal curve, the direction of rotation of rear and font wheels are diferent, as shown in Fig 4:30 and so thee i ome loss in the tractive force. ‘A and B are the rear driving wheels which give a tractive force T in the direction PO. “The front wheels C and D ate tured so as to steer the vehicle along a horizontal curv. the tangential direction of which is RS. ence the tractive force available in this Uireetion = T cos a which wil be less than the actual active fore, T applied. Obviously sahen the runing rads is sharp, the ruin angle wil be high andthe value of F cos Fa a a ne loss of tractive force due to turing ofa vehicle ong horizenay curve, sich is termed as curve resistance wil be equal t(T—T.€05 0) oF Ti cos a) and will depend on the turning angle &- 14 HIGHWAY GEOMETRIC DESIGN, DESIGN OF HORIZONTAL ALIGNMENT Hence provide superelevation rate = 0,058, (iy Extra widening of pavement, We We = ae oy 2K osdR 3s the pavement width is 10.5 m 6m ae Wheel base ; wee BE 6s * "25325 * 95505 = 0.166 + 0,380 = 0,546, say, 0.55 m (ii) Length of transition curve, Ly {a) By rate of change of centrifugal acceleration : Allowable rate of change of centrifugal acceleration C is given by = a) 80 1S+V 76465 value is between 0.5 and 0.8, accepted for design) Fig. 4.30 Curve Resistance for Tu Turning along the horizontal curve is effected due to the lateral friction develop, between the front wheels and the pavement. At sharp curves, if the speed is high t may be even sliding along the tangential direction PQ. Thus while a vehicle driven by rear wheels tums along a horizontal curve, there is increased resistance and if the s speed as on straight is to be maintained, a higher tractive effort is needed. But in Vehicles with front driving wheels, this problem docs not exist. Most of the hea Comercial vehicles have rear driving wheels and hence on sharp curves the additio urve resistance should also be considered while designing the geometric features = 0.57 misee? (As sharp and jon, the roads have steep gradients. The compensation in gradi needed for such a case has been explained later in Art, 4.5.2 L, = 2e21sv? _ oorisx6s? | CR 0574325 (b) By rate of introduction of superelevation, & : Total superelevation, E = B x e 319m 4.4.10 General Examples on Horizontal Alignment Example 4.18 While aligning a highway in a built up area, it was necessary to provide a horizo circular curve of radius 325 metre. Design the following geometric features otal pavement width including extra widening on curve, B= W+We=105+055= 11,05 m Superelevation rate, © = 0.058 E = 11,05 «0.058 = 0.64 m ‘Assuming that superelevation is provided by rotating about the centre line, the total superelevation to be distributed along the length of transition curve = E/2. The rate of introduction of superelevation may be taken as | in 100, being built up area, Length oftansiton cure Ly = 9 x100=32m (i) Superelevation Gi). Extra widening of pavement (iii). Length of transition curve Data available are Design speed = 65 kmph, Length of wheel base of largest truck = 6 m, Pa width = 10.5 m B Solution (©) By IRC formula, the minimum length a 2 2:9V2 _ 29x63" scm tem oi 325 ‘Adopting highest of the above three values, length of transition curve L ~ 38 m. (i Superelevation rate, € From practical considerations of mixed traffic conditions, superelevation to ear centrifugal force should be designed with 75% of design speed. Here f =325m. 8 Example 4.19 A State Highway passing through ‘equal to the ruling minimum radius. v2 225R 225x325 AS this value is less than 0,07, itis safe for the design speed. 4 rolling terain has horizontal curve of radius = 0.058 136 HIGHWAY GEOMETRIC DESIGN (i). Design all the geometric features | of this curve, assuming suitable data. Specify the minimum set-back distance from the centre fine of the two | Oar on the iner sie ofthe curve up to which the buildings ele. obstruct 3 Constructed so that Intermediate sight distance is available vision should not be te Teeibhout the citcular curve. Assume the length of eircular euve greater than the unt distance. Solution ‘The various geometric elements to be designed are (2), Ruling minimum radius (b) Superelevation (©) Extra widening, (a), Length of transition curve (e)_ SSD, ISD and set-back distance ing minimum radius of curve for ruling design speed of 80 kmph : ve 807 iT 127(0.07+0.15) Rruing = 229 m, say 230m (b) Design value of superelevation ee YO moa SR ~ 225x230 ‘As the value is higher than the maximum superelevation of 0.07, limit the value of 007. The curve should be safe for the full speed of 80 kmph as the ruling minimun radius has been adopted. Hoswever check the transverse skid resistance develope 2 2 pe SY -e= 8 -007=0:149 127R 127230 (Less than 0.15 and hence safe) (c) Assume two lane pavement, i.,n=2 and /=6 m. Extra widening of pavement, a at 2R SAR We = 2x6? 80, 25230 | 9.5230 Provide an extra width of 0.71 m and a total width of pavement B = 7.71 m. = 0.157 +0.555 =0.712m (4) Length of transition curve is designed by calculating the values based on (i) f of change of centrifugal acceleration C (ii) rate of introduction of the amo superelevation E and (ii) minimum length formula; the highest of three values is 2d at the design length Ls. " DESIGN OF HORIZONTAL ALIGNMENT ae 18 PAD) TS+V ” 75480 (as this is within the range 0.5 to 0.8, the value is acceptable for design, @ cs 052 Ly = 20215? _ o0215x80? CR 0520" in Toa eared capone » the raising of the outer edge of the pavement with respe = Bx e= 7.71 x 007 = 0.54 m. in soli ‘sume the pavementto be rotted about the cenve startet tn SD E y= 054x150 Lge = 5 N= OSI 405m (ii) Minimum value of Las per IRC is given by : 2.av? _ 27x80" = 2-20. = 7m ‘Adopting the highest ofthe three values, design length of transition curve = 92 m. (€) Intermediate Sight Distance =2 SSD 2 2/0278 vis 24 = 807 = 2|0.278%80%2,5+— _|=2« 127.6= [ aos] 2 1276=255 m (f) Refer Fig. 4.31. The length of circular curve is assumed greater than the desired si Sinance SD" The minimum lerancs or eck dane nerded m= CD and hall he central anglea’/2 = angle AOD. 431 Clearance on curve or Set-back Distance for Desired Sight Distance (Example 4.19) ‘centre line of the pavement and the centre line of the ‘The distance d between the wrth the with of pavement at the curve (being a two- inside lane may be taken as one-fo Jane pavement) = 7.71/4 = 1.93 m. 138 HIGHWAY GEOMETRIC DESIGN 1308 __180*255_ 400 zy * Te(R=d) 2RQ30-1.93) a Set-back distance m’ = R-(R-d)cos > = 230 — (230 - 1.93) cos 32° = 36.6 m ‘Therefore the minimum set-back distance or clearance required to provide a clea vision for an ISD of 255 m is 36.6 m. Example 4.20 ‘There is a horizontal highway curve of radius 400 m and length 200 m on highway. Computer the set-back distances required ftom the cent Tine on the inner si lof the curve so as to provide for (a) stopping sight distance of 90. m (b) safe overtaking sight distance of 300 m. “The distance benween the centre lines of the road and the inner lane is 1.9m. Solution (a) Refer Fig. 4.32. ‘The stopping sight distance (SSD) of 90 m is less than th circular curve length of 200 m Fig, 4.32 Minimum Set-back when SD is less than Length of Curve (Example 4.20a) _ 18090 _ goog: 2x 400-19) 400 - (400 - 1.9) cos 6° 29° = Required clearance from the centre lie to provide SSD of 90 m is 4.4m. (b) Refer Fig. 4.33. The overtaking sight distance of 300 m is greater than is curve length which is 200 m. Therefore the required set-back distance is CF = G GF) and is given by Eq. 4.28. S= 300 m, Le = 200 m,R = 400 m, d= 1.9m DESIGN OF HORIZONTAL ALIGNMENT 39 puevees ves toa ¥ (ore: nor fo scaue) Fig, 433 Minimum Set-back Distance when SD is Greater than Length of Curve (Example 420) @_1ROL, 180200 2” Be (Rd 2xao0-15) Set-back distance m’ = CF=CG + GF = R-(R=4)c0s & 2) gin 2 2 = 400~ (400 ~ 1.9) c0s 14.39°-+ 300-200) in 14.39° 1 1444+ 12.4=26.8 ‘Minimum set-back distance required from the centre line of the roads on the inner side of the pavement to provide an OSD of 300 m = 27 m. 4.5 DESIGN OF VERTICAL ALIGNMNET 45.1 General While aligning a highway itis the general practice to follow the general topography oF profile of the land, But the natural ground may be level only at some places and btherwise the ground may have slopes of varying magnitudes. Hence the vertical profile fof a road would have level stretches as well as slopes or grades. In order to have smooth Vehicle movements on the roads, the changes in the grade should be smoothened out by the vertical curves, The vertical alignment is the elevation or profile ofthe centre fine of the road. The vertical alignment consists of grades and vertical curves, and it influences the vehicle speed, acecleration, deceleration, stopping distance, sight distance and ‘comfort in vehicle movements at high speeds. 4.5.2 Gradient i i y the lenath of the road with respect to the Gradient is the rate of rise or fall aiong we patel: a eae horizontal, It is expressed asa ratio of | inx(1 ‘vertical unit times the gradient is also expressed as a percentage, nie, 2 100. i, 140 HIGHWAY GEOMETRIC DESIGN When the angle of gradient, a is small (Refer Fig. 4.34a) the gradient which ig or tan a is approximately equal to the eircular measure or o in radians (a). Al) ‘vthin the practical range of gradients on roads may be treated as small, Hence grading ‘which are generally represented as °n’ percent, would mean that this is the vale ofthe tangent of the angle made by the gradient with horizontal, ic, n% = tan c. The ascending gradients are given positive signs and are denoted as + nj, + m2 ete, and, descending gradients are given negative signs and are denoted as ~ m3, — ng ete. The angle which measures the change of direction at the intersection of two grades is calleg the deviation angle N which is equal to the algebraic difference between the two grades, In Fig, 4.34b the deviation angle, Ving N ZDBC = ZBAC + ZBCA = +m-Ca)e mtn where ny is ascending gradient of AB and — n9 the descending gradient of BC. Fig. 4.34 Measure of Gradients While aligning highway, the gradient is decided for designing the vertical. cu Vehicle operation cost is increased. The engineer has to consider all aspects such, construction costs, practical problems in construction atthe site and the vehicle operati cost in such alternative proposals before finalising the gradients. Gradients are divided into the following categorie Ruling gradient Limiting gradient Exceptional gradient Minimum gradient The terms are explained below : Ruling gradient is the maximum gradient within which the designer attempts to desi the vertical profile ofa road. Gradients up tothe ruling gradient are adopted as 2 10 course in design of vertical alignment and accordingly the quantities of cut and fill ae balanced. Hence ruling gradient is also known as design gradient. However fl sradients may be preferred where ever practicable, 5 DESIGN oF VERTICAL AUGNuNET “the selection of ruling gradient fog some time not even possible to. levels to be covered in short length of road. SM becatie wf tangs aiference in ‘A vehicle which travels with a cenain speed om a level effort put in, Would lose speed at grades; he sneny in length of grade. With the maximum pulling poses ay eee wih increase sustain the same speed even on long sections only up ta s eerta, ee oid be te to the maximum power developed by the engine is equal to the pomee crrneg eet tele min e ee e aepn nomae i fore this gradiet, i tie one ‘Thus it is not possible to lay down precise standards of ruin the mixed traffic and for the country as @ whole. fa we ‘The IRC has recommended ruling gradient values of 1 x 0 tera, 1 in 20.00 mountainous train and I in 16.7 on steep tera tS FE temain. Where topography ofa place compels adopting steeper gradients than ruling gradients limiting graders are wsed in view of enormous incase @ nan canted ak entler gradients. However the length of continuous grade line steeper thas roles value should be limited. On rolling terrain and on hill roads, it may. be fregocets necessary to exceed ruling gradient and adopt limiting gradient; but care should be taken to separate such stretches of steep gradients by providing either a level road or 3 road with easier grade, In some extra ordinary situations it may be unavoidable to Provide still steeper gradients atleast for short stretches and in such cases the steeper gradient upto exceptional gradients may be provided. However the exceptional gradient should be strictly limited only for short stretches not exceeding about 100 m at a stretch. ‘The maximum length of ascending gradient which a loaded truck can operate without undue reduction in speed is called critical length of grade for a design. A reduction in speed of about 25 kmph may be considered reasonable limit. The critical length for design depends on several factors such as size, power, load and grade ability dara of the ‘ruck, its initial speed at the beginning of the ascending grade and the desirable limit of the minimum speed at the end of the grade so as to avoid unreasonable interference with the movement of other vehicles. The critical length of ascending gradients should therefore be limited to lower values at steeper gradient ‘The maximum values of gradients recommended by the IRC at different terrains are Siven in Table 4.13. ie phan ; i cases there will s a de fond can be level, ih itl oro ef te ie ris by proving sean, Te ea Se so in longitudinal slope is essential, to drain the water along. ‘the side drains, % ‘Surface of the drains. 42 HIGHWAY GEOMETRIC DE! Table 4.13 Gradients for Roads in Different Terrains " Ruling] Limiting [Experimental] Tera gradient_| gradient pull I Piain oF rolling 3.3 per cent [5 per cent] 6.7 per cent ee (in30) | in20) | (1 in 15) ‘Afountainous terrain, and’ steep terrain) 5 per cent 16 per cent] "7 per cent [paving elevation more than 3,000 m| (1 in20) (Vin 16.7)) (Vin 14:3) labove the mean sea level FRiecp terrain upto 3.000 m height above) 6 per cent ]7 per cent] 8 per cent imean sea level (Lin 16.7) |(1 in 14.3))_ (1 in 12.5) Suppose the road is with zero gradient passing through level land and open side draing are provided with a gradient of say 1 in 300. It may then be necessery 10 ‘deepen the. downstream end of the drain by about 3.3 m for one kilometer length of road. This course is not possible from practical ‘considerations. Hence it is desirable to have a) certsin minimum gradient on roads from drainage point of view, provided topography favours this, The minimum gradient would depend on rain fall run off, topography and other site conditions. [A minimum gradient of about | in S00 may be sufficient to drain water in cone drains or gutter: but on inferior surfaces of drains a slope of I in 200 or 0.5 percent bs needed where as on kutcha open drains (soil drains) steeper slopes upto 1 in 100 or 1.0 percent may be needed. ‘Compensation in Gradient on Horizontal Curves ‘At horizontal curves, due to the turning angle @ of vehicles, the curve resi a developed is equal 10 T (1 — cos a), a5 explained in Ar. 4.4.9. When there i a hori cece i addition to the gradient, there will be increased resistance to fraction due to sradient and curve. It is necessary that in such cases the total resistance due Yo grade a sadiert ould not normally exceed the resistance due to the maximum value of seetient specified. For design purpose, this maximum value may be taken as the rul Sradient and in some special cases as limiting gradient for the terrain. | When # Fathom curve is to be introduced on a road which has already the maxim permissible wradient, then the gradient should be decreased to compensate For the loss of | tractive effort due to the curve. ‘This reduction in gradient at the horizontal curve is called grade compensation. is intended to off-set the extra tractive effort involved at the curve. This, is calcula from the relation : 5 _ 30+R Grade compensation, percent = , subject to a maximum value of 75/R, R is the radius of the circular curve in metre. According to the IRC the grade compensation is not necessary fo than 4.0 percent and therefore when applying grade compens gradients need not be eased beyond 4 percent. Example 4.21 While aligning a hill road with a ruling gradient of 6 percent, a horizontal DESIGN OF VERTICAL ALIONMNET e i solution Ruling gradient = g.gey Grade compensation ~ 222% 300. ‘Maximum limit of grade compensation = 75K = 7566 » | 258 ‘Therefore, compensated gradient = 60-125 = 4754 4.5.3 Vertical Curves ee changes in prada the vera alignment of highway seeway introduce venical curve at the intersections of different grades to smoxten ont ‘artes pooled ts ease of the changes radcas ft be fe mening vce. ‘The vertical curves used in highway may be classified into two categories, (a), Summit curves or crest curves with convexity upwards (b)_ Valley oF ag curves with concavity upwards ‘Summit curves _ Summit curves with convexity upwards are formed in any one of the case Whar in Fig. 4.35. The deviation angle between the two interacting gradices ts equi to che algebraic difference between them. Of all the cases, the deviation angle will be ‘maximum when an ascending gradient meets with a descending gradient Le. = 1 (=z) = (my +73). Fig. 4.38 Summit Curves When a fast moving vehicle travels along a summit cure, the cevrifueal free wit act upwards, against gravity and hence a part of the pressure the tyres and spring of the vehicle suspensions is relieved. So there is no problem of discomfort to passengers oon summit curves particularly because the deviation anges cn roa." GY small and Fike veut i designed to have adequate sight Stance. be nett ‘of the curve would belong enough o ease the shock due fe change i FAS ‘The only problem in designing summit ewes oe nat diane ‘The stopping sight distance or the absolute ri os ate provided at all sections of the rasd system and 9° curves. As f 144 HIGHWAY GEOMETRIC DESIGN possible safe overtaking sight distance oF at least intermediate sight distance, feice the stopping sight distance should also be available on these curves for imp highways, except when restrictions to overtaking have been strictly imposed a, sections concerned. ‘Types of summit curves ‘As the design of summit curves (except low summit or /iumps which do not intr with sight distance) are govemed only by considerations of sight distance. transit curves are not necessary. Circular summit curve is ideal as the sight distance avai throughout the length of eircular curve is constant, From this view point, transition e may be said to be even undesirable to be used on summit curves. This is because the. J radius of curvature and hence the sight distance would vary from point to point along length of curve. ‘The deviation angles in vertical curves of highways are very small so between the same tangent points, a simple parabola is nearly congruent with a ci are: also a parabola is very easy for arithmetical manipulation for computing, ordi The use of Because of these reasons in actual practice a simple parabolic curve is used as sum curve instead of the circular arc. 4 “There may be situations where the problem of sight distance does not arise asin the ea when the road goes over a hump. ‘This may be due to the presence of a culver, the top which is above the general level of the road by less than about a metre thus causing a. but relatively small summit or hump on the road profile. In such a case the problem is discomfort to the passengers and so the most suitable curve would be transition cu fer side of this hump. For proper design of humps, the vertical profile should curves on either side of the hump with a level strip in between. ‘wo trans Length of summit curve Parabolic summit curves are generally adopted, the equation which is given by : 5 N = x7, with value of a= > — yx with value of a= > Here N is deviation angle and L is the length of the curve. Since the summit curves Jong and flat, the length of the curve ‘L’ is taken as equal to its horizontal projection A (Refer Fig, 4.36) as itis practically equal to the actual length of the curve. Length of Summit Curve While designing the length of the parabolic summit curves, it is necessary 10 the stopping sight distance (SSD) and overtaking sight distance (OSD) separtey mentioned earlier, itis essential to provide sight distance at least equal to th distance at all points on a highway so as to avoid accidents due to i distance. aU ig) iple parabola as summit curve is found to give good riding comfort ta inadequate DESIGN OF VERTICAL ALIGNMNET iss Length of summit eurv fr stopping sight distance ($50) “Two cases are to be considered in deciding the length (i) When the length of the curve is greater than the sight distance (> $80) (i). When the length of the euveis es than the sight distance (L-< SSD) ( When L> SSD ‘The general equation for length L ofthe parabolic curve is given by : he ns? (ans Sane se Here L = lengthof summit curve, m stopping sight distance, (SSD), m = deviation angle, equal to al; sic difference radians tage ori even anges ee ne i H_ = height of eye level of driver above roadway surface, m ‘h = height of subject above the pavement surface, m ‘The value of H, the height of driver's eye above roadway surface is taken it india s dicusaed i Ar 43.3. The height of objet above the pevemen sae for the purpose of safe stopping distance is taken as 0.15 m as per the IRC standard. Substituting these values in equation 4.29 the length of summit curve is obtained as: nst 44 Zo " ie. L= (430) (i) When L< SSD ‘The general equation forthe length of the parabolic summit curve, when iti less than the sight distance is given by : L = 28- 3p (H+ Vay WN Here the deseription for L, S, N, H and h are the same as in equation 429. By substituting the values of H= 1.2 anh =0.15 m the length of the curve is obtained as A L = 2s- 32) N “Thus to design the length of vertical summit curve providing, for stopping or absolute minimum sight distance, first the safe stopping sight distance is found iter by Calculation as given in Art. 43.2, and equation 42 or ffom the recommended valves fiven in Table 4.5. By using any of the two appropriate formulae by Wil (430 and 4-32) the required length of summit curve is then ealcuated, The minsmum radius of dhe parabolic summit curve may be calculated from the relation R= UN. Length of summit eurve for safe overtaking sight distance (OSD) or Intermediate Sight Distance (ISD) “Two cases to be considered in deciding the length ae + Lao. HIGHWAY GEOMETRIC DESIGN agit of evye is gteator thaw the overtaking OF Iniermedkatg DESIGN OF yj SIGN OF VERTICAL AL L ALIONMNET Fable 4.14 Minimum ten (When the te alistance (tS) aint the length of the curve is less than the overtaking or lntermediate 9) uy ith of Vertical Curves (i) When distance (LSS) re 3 ; i rade cage Gps iy When >S te _Hol equting averse pe _ Tyee genera B32 8 apliale in his eave alt in ths a 0 1 og Hand "he Both are taken equal fo 1.2 m, Substituting b= TY inthe Eq, 4.29 ee [a ut a te hes 29 a 2 2 en - 3S ae 06 sit os ‘The highest point on the summit listance the first grade nj, is is oes ice amet aes on simple parabola uation : y= ax? and by differenti equating (0 zero for the Noone height y with respect to a ee As discussed in Art, 4.3.3 the height of the eye level of | the driver as well as the h of the approaching object are taken as 1.2 m, Substituting the value of Hl, the he eye level of driver above the pavement surface equal to 1.2m, 2 finding the ordinat summit ver ns? i fe value Ys, Y3, Ys... for dit ical curve may be ploted ij io, ee tangent point, by taking measurements alone sh different lengths X,, X. plotted by ment alo te Xap Ky oa 26 Valley eurves ng the tangent lengths. heal ine Here. L. = the length of parabolic summit curve, N = deviati VAY CUPS oF Sag curves are formed d wviation angle, radians or tangent of the deviation angle, Fig: 4.37. In all the cass the maximum in any one of the cases i lescendi ; possible devi cases illustrated ie S = overtaking or intermediate sight distance, (OSD/ASD) ing gradient meets with an ascending gradient. angle is obtained when a ti) When L ‘ ate L283 , a Here again substituting the value of H as 1.2 m, the equation reduces to: (b> 96 L=/28- Here L, Sand N are as in Eq. 4.33. ‘Thus to design the length of vertical st overtaking, the value of the overtaking or interme the available data as given in Art. 4.3.3 and Eq. given in Table 4.7 or twice the SSD value as given in ISD._ By using any of the two appropriate formula 4.3 sammit curve is then calculated. The minimum radius of the curve the relation R= L/N as before. When the deviation angle is small, th summit curve providing for safe of. diate sight distance is calcul 4.6 or from the recomme! Table 4.5, for finding the. 3 and 4.34, the required le Ris calcu ter a Fig. 4.37 Valley Curves There is mo problem of restrictc i ea i 10 problem of restriction to sight distance in valley during iF aay ca during night driving under bead lighs of vehicles, the sight docanse ase ' cin i ‘The most important factors considered in valley curve design ae, 0 krpace es mova of whic San speed or the comfort tothe passengers and (i) leviatio sm : silty of stopping sight distance under head lights of vehicles foe mig diving, The jess than the sight distance. In very smal lowest point in the valley curve may be located ftom considerations of ross drainage, times works out as a negative value indicating that At the valley curve, the centrifugal force acts downwards adding to the pressure on the at the summit curve. 6 rings and te upesions ‘of the vehicle in addition to that due to weight ofthe vehicle. ‘The minimum lengths of vertical curve for diferent speeds and for the 8 allowable rate of change of cetioga ekration should Rover ie design aoe ae ee Gn, percent) which do not require vertical ON ire vertion ¢ length of summit curve generally I deviation angles, the length re there is no problem of sight a 148. HIGHWAY GEOMETRIC DESIGN of the valley curves, Obviously the best shape of valley curve is a transit tradually introducing and increasing the centrifugal force acting downwards, parabola is generally preferred in vertical valley curves. As the deviation angle Penal the path traversed by the three types of transition curves, viz. spiral, lenis cubic parabola are almost the same as explained in Art. 4.4.7, ‘During night driving the visibility ahead is dependent on the head light of the when the road lighting is not adequate or has not been provided. There is rest . the sight distance at valley curves as the head light gets intercepted though the_ Tight may be slightly inclined upwards, ‘Therefore the head light sight distance a at valley curves should be atleast equal to the stopping sight distance. However, th er no problem of overtaking sight distance at valley curves during night as ‘other vehi with head lights can be seen from a considerable distance. , Length of valley curve hie ee ‘The length of valley transition curve is designed based on the two <1 allowable rate of change of centrifugal acceleration of 0.06 m/sec’ and (i) the head ig Sight distance, and the higher of the two values is adopted. Usually the sesond e oof head light sight distance is higher and therefore governs the design. ‘The valley curve is made fully transitiona! by providing two similar transition eu of equal length (without a cirular curve in between). Refer Fig. 4.38, where ASC Is Walley curve of total length L and AB and BC are two equal transition curves ae qeneth L,~ L22, having the minimum radius R at the common point B 4 Lenern oF vaLer conv! Fig. 4.38 Length of Valley Curve (1) The fengih of ransition eure Jr comfort condition is given Ly Value of R (at lengt Ls) = ue ‘by equation 42 DESIGN OF VERTICAL ALIGNMNET 149 re N isthe deviation angle, v is speed in m/sec and C isthe allowable rate of c tiga acceleration which may be taken as 0. mie ne Vikmph = SE mee . ow 16.3.8 L, = 0.19 ¢nv'yi? L = 2L,-038(Nv)!7 (4.36) tence the total length of valley curve is given by enuati 1 aR 1 = a[tzf ono where L = total length of valley curve, N= deviation angle in radius or tangent of the deviation angle or the sleebraie difference in grades. LV = design speed, kph “the mn radius (R mete af he valley curve fer cubic parabola i given bY = ae Bee (a3) NaN, jined for the (a) The length of valley carve or hed igh sight stone may be detent two conditions : (i) when the total length of valley curve L is greater than the stopping ‘sight distance SSD and (ii) when L is less than SSD, as given below. f) L> SSD “The length of valley curve L is assumed to be than the head light sight distance which oo atleast equal to SSD, as ‘shown in Fig, 439. Let the height of the head light be hy and the focused portion of the ‘beam of light be inclined at an angle upwards. ‘The sight distance available will be ‘minimum when the vehicle is at the Jowest point on the sag curve. If the valley curve is assumed t0 be ‘of parabolic shape, with equation y= 8 x*, where a = N2L. - age NS hy +Sune ms Fig, 4.39 Mead Light Sight Distance when L>S Sa 180 HIGHWAY GEOMETRIC DESIGN Ls ns? Gh F2Stana) Af the average height of the head light is taken as by cx 1* by suiting these in the above equation, nn MM bea ns? (3+00355) where, L_= total length of valley curve, m(L> $) S = SSD,m N = deviation angle = (nj +13), with slopes ~ ny and +n; ay L SSD (Eq. 4.30), ue = 2979m( 128m) “Therefore length of summit curve =298 m ‘This value is higher than the minimum specified length of $0 m at 80 kmph speed as per Table 4.14. Example 4.23 ‘An ascending gradient of | in 100 meets a descending gradient of 1 in 120. A summit ‘curve js to be designed for a speed of 80 kmph so es to have an overtaking sight distance of 470m. Solution IfL>Osp_ From Eq. 4.33 length of summit curve, a eee br 96 0x96 ‘As this Value is ess than OSD of 470 my assume L estan OSD. 152 HIGHWAY GEOMETRIC DESIGN DESIGN OF VERTICAL ALIGNMNET IfL>OSD 153 From equation 4.34 length of summit curve, fL> sD b= NS? 1323008 96 400596 ‘As this value is greater than the SD of 360 m, the assumption is correct. Its possible to provide the ISD of 439 m to allow limited overtaking operations and the Summit curve inthis ease is less than the maximum available length of 00m =e" =439m 9.6600 = 2«470- = 416.4 m, say 417m This value is less than 440m. te 4.25 ‘Therefore, the length of summit curve = 417 m. mee ‘A valley curve is formed by a descending grade of 1 in 25 meeting an ascendin Example 4.24 ot SSE” Design the lng ovale cure to full buh confor conion eS : light sight distance requirements for a design speed of 80 kmph. Assume allowable rate ‘A vertical summit curve i to be designed when two grades, + 1/50 and ~ 1/80 meet on of change of centrifgal acceleration C= 0.6 msec a highway. The stopping sight distance and overtaking sight distance required are 180 Solution and 640 m respectively. But due to site conditions the length of vertical curve has to be restricted to a maximum value of 500 m if possible. Calculate the length of summit curve needed to fulfil the requirements of (a) Stopping sight distance (b) Overtaking sight distance or atleast Intermediate sight distance and discuss the results. 23°30 «150 V = 80 kmph, v= 80/3.6= 22.2 m/sec (i) Comfort Condition From Eq. 4.34, Solution 1 (a) Requirements of stopping sight distance (eae P =23.1m 150° 06 SSD = 180m 1 iP eae] 6 (i) Head Light Sight Distance Condition Neglecting the ascending and descending gradients at the valley curve wsing Eq. 4.1 and assuming t = 2.5 secs. ‘and f= 0.35, SSD =vt+\"/2gf= 22.2 x 25 + (22.272 9.8 x 0.35)= 127.3 m IFL> SSD, using Eq. 4.38 Assume L> SSD NS? _ 13x180 =239.3m 44 400x4.4 ‘As this length is greater than SSD the assumption is correct. ‘The length of summit curve required is 240 m which [s les than the prescribe maximum limit of 500 m. i (b) Requirement of overtaking sight distance osD = 400m Ns? 1127. bee = (S¥0.0358) _ 150(1.5+0.035%127.3) As this value is higher than the SSD of 127.3 m, the assumption is comes, The way carve length based on headlight sight distance being higher than that based on comfort Assume L> OSD condition, the design length of valley curve is 199.5 or sty, 200 m- 2 NS? 13640" 2 1387 96 400%9.6 curve obtained is higher than the sight distance, lengtt REFERENC Es 1 Matson, TM, Smith, WS., Hurd, H.W. Traffic Engineering, McGraw Hill Book Co. Inc., New York. 2 Dhir, MP. and Bhatt, AK I National Seminar 20 years Design Ministry of Transport and Shipping ‘As the length of summit required is 1387 m. ‘As suggested in probl ir iit lity of Our Pavement, Improving te RS on and Bridges, Vol. 1 lem if the length of the summit curve is restricted to a value and C (Roads Wing), 1968. than 500m, it is not possible to provide the required 0-S.D. of 640 m. 4 ‘Therefore, to provide limited ‘opportunities for overtaking, Intermediate Si (4S) equal to twice the SSD of 180 x 2 = 360 m may be provided if possible: 154 HIGHWAY GEOMETRIC DESIGN PROBLEMS 3. Specification and Standard Committee, Widths of Highway Pavement : Iss Indian Roads Congress, Vol. X-1 and 4. 1946-47. s+ Journ, 6 Write short notes on : 4 Specifications and Standard Commitee, Policy Concerning, qTatrorsis Dimensions, Weights and Speeds of Motor Vehicles in India, Journal, Indian R (b) Kerbs Congress, Vol. XIII-2 and 4, 1949, ads {c) Roads margins 5. Specifications and Standard Committee, “Dimension and Weights of Road Design _ (4) Pavement unevenness Vehicle”, Journal Indian Roads Congress, Vol. XVII-2, 1953-54. (e) Shoulders 6 Geometric of Roads, Indian Roads Congress, 1966 (Reprint from Tr i tic Communication Monthly Review, May 1966). casei os von ae oo ee (g) Right of way. 7 Specifications and Standards Committee, “Standards for Sight Distances § Highways”, Joumal Indian Roads Congress Vol. XN-1 and 4, 1950-51. 8 AASHO, A policy on Geometric Design of Rural Highway, American Associtio of State Highway Officials; Washington, 9 Specifications and Standards Committee. Horizontal and Transition Curves for Highways Journal, Indian Roads Congress, Vol. XI-3 and 4, 1946-47. 4 10 Design Tables for Horizontal Curves for Highways, Indian Roads Congress, IRC 38-1970, Now 1970, 11 Ives, H.C., Highway Curves, John Wiley and Sons, Inc., New York. 12 Specifications and Standards Committee, Vertical Cures for highways Journal Indian Roads Congress, Vol. XVI-1 and 4, 1951-52. 13. Indian Roads Congress, Recommended Practice for Sight Distance on Rural 7 Draw the typical cross sections of the fllowing roads indicating the wi pavement, roadway and land ge aicaing erie (a) MD. in embankment (b) OD.R. in cutting nal highway in embankment in rural area (d) National highway in cutting (@) Acity road (® A divided highway in urban area 8 Explain sight distance and factors causing restrictions to sight distance. Explain the significance of stopping, intermediate and overtaking sight distances. 9 What are the factors on which the stopping sight distance depends 7 Explain Highways, IRC : 66-1976. brietty 2 14 Indian Roads Congress, Recommendations about the Alignment, Survey and 10 Explain vot revetion tine’ iver and he rs on which lade Geometric Design of Hill Roads, IRC : 52-1981. *PIEV" theory. 15 Indian Roads Congress, “Geometric Design Standards for Rural (Non Urban) Highways”, IRC : 73-1980 16 Institute of Traffic Engineers, “Tronsportation and Traffic Engineering Hand Book”, Prentice Hall. 17 Indian Roads Congress, “Geometric Design Standards for Urban Roads in Plains", IRC : 86-1983. 11. Derive an expression for finding the stopping sight distance at level and at grades. 12 Calculate the stopping sight distance for a design speed of 100 kmph. Take the total reaction time 2.5 second and the coefficient of retion = 0.35. 13. Find the stopping sight distance for a design speed of 65 kmph. Assume suitable data, What are sight distance requirements at gradient of! in 40. (im for Sotucion : ‘Assume total reaction time as 2.5 second and design coefficient Of friction as 0.36: find the stoping sish distances for level, ascending and descending grades). 11 State factors on which the overtaking sight distance depends. Explain briefly. 15 Derive an expression for caleulating the veraking sight stance on ighv2y 16. Why are overtaking zones provided ?. What ihe bss of deg © neh? Draw a neat sketch and show the signs tobe installed and their Pos! jcle are 80 and 60 kmph respectively The speeds of overtaking and overtaken vehicle 25 cond, calculate the IF the acceleration of the oversaking viele is 2S PROBLEMS 1 What are the objects of highway geometric design ? List the various elements to be considered in highway design. Explain the role of pavement surface characteristics in highway geome desi Stare the factors affecting friction between pavements and tyres of vehicles ? a i y je iscuss the factors on which @ 3. Explain camber. What are the objects of camber ? Discuss 1 f expel cr camber to be provided depends. Specify the recommended Tans st ne camber for different types of pavement surfaces. Fe passing sight distance for a i ross one-way traffic 4. Discuss the effects of shape of camber and the effects of providing st°¢P = on ae ‘ he & Enumerate the factors governing the width of carriage way. Sie OTs specifiations for width of carriage way for various clases of roads 156 HIGHWAY GEOMETRIC DESIGN : . 18 In problem 17, shat should be the Fength of overtaking zone, Show the pj ae signs by means of a sketch, ton 19 Find the safe overtaking sight distance 100 kmph, Assume all data suitably, [Uline for Solution : Assume speed of overtaken vehicle «(100 ~ 16) kmph; mac Actelion af overtaking vehicle = L6 kmphsee, as given in‘Table 46) "8 20. Discuss the factors to be considered in deciding the sight distance at intersections, 21 (a) Enumerate the design elements to be considered at the horizontal alignment. oa (b) What are the effects of speed on horizont design speeds for: alignment design? What erent classes of roads specified by the IRC ? r 22 (a) Explain superelevation superelevation depends ? What are the factors on which the desi (b) Explain maximum and minimum superelevations. 23 Derive an equation for finding the superelevation required if the design coefficient of lateral friction is *P be Pe 24 Enumerate the steps for practical design of superelevation ? 25 Design the superelevation required at a horizontal curve of radius 300 m for speed oF 60 kmph, Assume suitable data r 26 Calculate the maximum allowable speed on a horizontal curve of radius 350 m if) the maximum allowable values of lateral coefficient of friction is 0.15 and rate of superelevation is 0.07. 27 A radius of 250 m has to be provided at a locality due to site restrictic National Highway with design speed 100 kmph. Design the superele\ there be restriction in speed ? 28 Explain with the aid of neat sketches the methods of eliminating camber introduction of superelevation. 29 What is the basis of calculating absolute minimum radius of horizontal curve 2. What do you understand by ruling minimum radius of curve ? 30 Calculate the absolute minimum and ruling minimum radius of horizontal for a design speed of 80 kmph. 31. State the objects of widening pavement on horizontal curves ? What are the factor on which the design of widening depends ? Explain. 32. Derive an expression for finding the extra widening requited on horizontal curve 33. (a) Why should the psychological widening be added to the mechanical widening? (b) How is the i (c) What is offtracking ? Explain with sketches. (d) Determine the offaracking of a vehicle with wheel base 7.0 ™ negotiating a horizontal curve of radius 100 m. 34 Calculating the extra width of pavement required on a horizontal curve otras ‘on atwo lane highway, the design speed being 80 kmph. Assume whee! base /=" lening of pavements introduced in the field ? PROBLEMS, e 6 in a mountainous tern, a ilar curve of rads transition of 20:m on both end, Cake extra widening de eee knph. Sueyest suitable method of providing the widening on atmo lane pavement 4 What are the objects of providing transit f 3 3 Oe ee Eagle ing transition curves on the forizontal alignment 37 List the various types of transition curves used in highways, What is an i transition curve? Explain, Sm inet 438 Discuss the factors to be considered while designing the length of wansition curve. 39 Derive an expression for finding length of transition curve on horizontal alignment ‘of highways. 140 ‘The radius of a horizontal curve is 400 m, the total pavement width at curve is 7.6 mand the superelevation is 0.07. Design the transition curve length for a speed ‘of 100 kmph, Assume pavement to be rotated about the inner edge. 441, Calculate the length of transition curve fora design speed of 80 kmph at horizontal curve of radius 300 m in arural area. Assume suitable data, (lint for Solution : Solve the problem as in Example 4.16), 42. (a) Explain curve resistance and compensation in gradient on horizontal curves, (b) There is a horizontal curve of radius 60 m on a stetch of bill road with 2 gradient of 5.0%, Determine the grade compensation 43. Explain with sketches how sight distance is restricted on horizontal curves and hhow the desired sight distance could be obtained 144 ‘The stopping sight distance required for a highway is 80 m. Find the required set back distance from centre line of a circular curve of radius 300 m assuming the length of the curve is greater than the sight distance, ie i i |. Find the required 45 The overtaking'sight distance required on a highway is 250 m. Find clearance of obstruction from centre line ofa circular curve of radius 350 m and length 180 m. Assume two lane highway with d = 1.9 m. “6 A national highway passing through 2 ft terrain bas arizona arse ot cia fequal to the ruling minimum radius. If the design speed ed se eaghet absolute minimum sight distance, superelevation, extra widening transition curve. Assume necessary data suitably. 47 Enumerate the various design factors controlling the vere highways, ; 48 (a) Explain ruling, maximum and exceptions! grediens- Specify the valves recommended by IRC for plains and hill. : nae (b) State the various considerations in deciding he ee sean 49. The ruling gradient of all oad is 1 in 20. What Ss am afer sradient and compensated gradient on # oro allowing for curve resistance ? i 50. Exptain summit and valley curves andthe 2 While two different gradients meet | alignment of rus cases when these are formed 138 st 32 33 37 58 HIGHWAY GEOMETRIC DESIGN Sate the factors that govern the length of summit curve, How is it decided 9 Discuss the requirement of summit curves and its shape. Explain how the vertical curves on a hump formed due tothe presence of a slightly above the profile may be designed. Explain the factors based on which the length of valley curve is designed, |A vertical summit curve is formed when an ascending gradient of 1 in 25 4 ‘another ascending gradient of 1 in 100. Find the length of the summit curve. provide the required stopping sight distance for a design speed of 80 kmph. The deviation angle at a summit curve is 0.05 and the overtaking sight distance 300m. Find the length of summit curve required. = : ‘An ascending gradient of 1 in SO meets a descending gradient of 1 in Determine the length of summit curve to provide (a) ISD (b) OSD, for design s ‘of 80 kmph. Assume all other data. A valley curve is formed by a descending gr ascending gradient of 1 in 30. i of 1 in 40 which meets (i) Design the total length of valley curve if the design speed is 100 kmph so as to fulfil both comfort condition and head light sight distance for night driving, after calculating the SSD required. (ii) Find the position of the lowest point of the valley curve to locate a culvert. Chapter 5 Traffic Engineering 5,1 INTRODUCTION 5.1.1 General “Traffic engineering {s that brarich of engineering which deals with the improvement of traffic performance of road networks and terminals. This is echieved by systematic taffie studies, Scientific analysis and engineering applications. The method includes planning and geometric design on one hand and regulation and eontrl on the other. Traffic Engineering therefore deals with the application of scientific principles, tools, techniques and findings for safe, rapid, convenient and economic movement of people and goods. ‘The road traffic is composed of various categories of vehicular traffic and the pedestrian traffic. Each eategory of vehicular traffic has two components, the human Tlement as the driver and his machine as vehicles. Traffic engineering has also to be by social and physical science. The profession of traffic “engineering as known today has evolved with the advent of motor vehicle. During the ‘advances have been made in many phases of the profession divanced study and training facilities have been made available at several universities and institution notably in the U.S.A. Traffic engineering has now been recognized es an sential tool in the improvement of traffic operations in metropolitan etes like Bombay, Delhi, Calcutta and Madras. Definition Institute of Traffic Engineers, U.S.A. defines, engineering which deals with planning’ and abutting lands, and with traffic operation thereon, as thei i convenient and economic transportation of persons and g00%s"- < versity has proposed a modified definition, wt ‘and travel, the study of the basic ‘praffic engineering is that phase of ic design of streets, highways, sr use is related to the safe, ete Ress Blunden of California Univ affic engineering is the science of meastring repens a wun rand gneaon ar EPO eve ale professional practice of planning designing and oPe=8"6 and efficient movement of persons and 159 160 ‘TRAFFIC ENGINEERING 5.1.2 Scope of Traffic Engineering ‘The basic object of trafic engineering is to achieve efficient free and rapia traffic, with least number of traffic accidents. Factual studies of traffic eh provide the foundation for developing methods for improvement in general a solving specific problems. and ‘The study of traffic engineering may be divided into six major sections, viz.: (i) Traffic characteristics (ii) Traffic studies and analysis (ii) Traffic operation-control and regulation (iv) Planning and analysis (v) Geometric design (vi) Administration and management study of traffic characteristics is the most essential prerequisite for any improv of traffic facilities, The traffic characteristics are quite complex with various types road users in the roads moving with different motives. The human psychology is to b given particular attention. The study of vehicular characteristics is an essential p ‘Apart from these the various studies to be carried out on the actual traffic include speed, Siume, capacity, travel patterns, origin and destination, traffic flow characteristics parking and accident studies. Various aspects that are covered under traffic operations are regulations, contol and the aasvuns for application of controls. Regulations may be in the form of laws and aoe aera se other traffic regulatory measures such as speed limit ete. Installation of, srere wontvol devices like signs, signals and islands are most common means 0 regulate ey control the trafic. Actual adoption of traffic management measures, such as traffic regulations and contro! need adequate attention. d “Trafic planning is a separate phase for major highways like express wi axterial ro mass transit facilities, and parking facilites. "All the aspects such as cross section and esc aght distance requirement, horizontal and vertical alignment, MME TEE areas ‘Meections and parking facilities ae to be suitably designed for beter performance, ‘The various phases of traffic engineering are implemented with the help Engineering, Enforcement and Education ot °3-ES" Enforcement is usually, mad through traffic laws, regulations and control. Education Oy be possible by sufi publicity and through schools and television. ‘aims at improving the human fast Peffie performance. Engineering phase isthe one which js constructive. It deals wi improvement of road geometries, providing ‘additional road facilities and installat suitably designed traffic control devices. 5.2 TRAFFIC CHARACTERISTICS 5.2.1 Road User Characteristics The human element is involved in all actions of the road users either 1s pte cyclist, cart driver or motorist. The physial, mental and emotional characters¢ TRAFFIC CHARACTERISTICS ey uma beings affect their ability t0 operate motor vebile sae . icly on to service as a uma an Hence iis important to the wali Fes sations of the road users Uaffic engineer to study the characteris aad ‘The various factors which affect road under four heads : user characteristics may broadly be classified Physical Mental Psychological and Environmental Physical characteristics ‘The physical characteristics ofthe road users may be either permanent or temporary ‘The pavement characteristics are the vision, hearing, strength and the general reaction to ions, the most important role of all these, These include the acuity of vision. jon and eye movement; glare vision glare recovery and depth judgement. it are often laid down by licensing authorities. Field of clearest and acute vision is within a cone whose angle is only 3 degrees, though the vision is fairly satisfactory up to 10° in general and even upto 20° in the horizontal plane. However in the vertical plan the fed of clear vision may be sbout two thirds ot That in the horizontal plane. These factors are particularly taken care of while designing nd installing control devices, As the field of clear vision is limited, the road users have to often shift their eyes within the peripheral field to obtain clear vision. The total time taken for the eye movement depends on some ofthe physical characteristis including the Tesponse to stimuli. The effects of glare, adopablity to changes of light ic. darkness 19 Tiant and bright light to darkness, should also be studied. The depth judgemens & important for a driver in judging distance and speed of vehicles and other objects ahead Hearing helps drivers in a way, though it is more iampenant for pedestrians and cyclists Though strength is not an important facior in general, lack of strengih mip make parking, maneouvers dificult, particularly for heavy vehicles, Ths weer co Matte aieatone depends on the time required t perceive and understand the ‘raffic aa nee te take the appropriate action. This depends on many facors such a4 penmanent and temporary physical fects menial and psychological SH oP speed and Fae a ian the time required to ake an appropriate aon Sepa oo the type of the problem and the familiarity, The PIEV theory explaining the ‘total reaction time has already been discussed in Art 4.3.2, “The temporary physical characteristics ofthe road wsers acing efficiency are fatigue, ‘alcohol pass ‘and illness. All these reduce alertness ‘and increase the reaction time and also affect the quality ofjudgerentinsorme sitions Mental characteristics Knowledge, skill, intelligence experience and ee a rvng pectic. characteristics, Knowledge of vehicle chart 5 ‘of roads and psychology of oad = be qui actions to certain trafic. situations Toe platen depends of Understanding the traffic regulation and spec! instruction intelligence and literacy. eg rer NT 162 TRAFFIC ENGINEERING Psychological factors These affect reaction to trafic situations of road users to a great extent emotional factors such as attentiveness, fear anger, superstition impatience, Sititude towards traffic and regulations and maturity also come under this. Distiact by non-traffic events and worries reduce attentiveness to traffic situations. a tetions are likely due to impatience. Some road users do not pay due regard - a to regulations and do not have the right attitude towards the traffic. oie Environmental factors The various environmental conditions affecting the behaviour of road user are stream characteristics, facilities to the traffic, atmospheric conditions and the locality. ‘The traffic stream may consist of mixed trafic or heavy trafic whereas the facilites to vera r to different traf characteristics depends on the driver's characteristics as well as the ten purpose of entering the traffic stream can be social, recreational, business, routine vrovement or an emergency dash. ‘The time, place and route are fundamentally chosen b/d the road user based on ihe needs. Whatever be the motive of movement, ones the individual enters the traffic stream, the road user, is usually motivated by the desire for 4 time-distanee economy on one hand, and comfort and safety on the other. Together with modifying factors of motivation, there isa grest variation among road users and thei ; behaviour in every traffic stream. The locality may be a shopping centre or a place with other distractions ‘to the road users, thus affecting their behaviour. The other environmental factors of importance are the weather visibility and other atmospheric conditions. 7 ‘The total reaction time or the “PIEV” time of the drivers vary considerably from driver to driver based on the above road users characteristics. But the reaction time of a particular driver may vary depending on the type of the problem and also environmental and modifying factors. Driver characteristics such as the simple, reaction time, depth judgement, field of Vision, visual acuity, glare recovery etc. may be measured inthe laboratory using Driver Testing Equipment. 4.2.2 Vehicular Characteristics It is quite important to study the various vehicular characteristics which affect the design and traffic performance, because it is possible to design a road for any vehicle but. ft for an indefinite vehicle. The basic criterion of highway engineering is to cater foF the needs of existing and anticipated traffic. It will not be economically feasible te keep tn increasing the geometric standards and thickness of pavements from time to time 1 on rahe needs ofa few vehicles whose dimensions and weight are increased. Hence th hile standards should be uniform at least within a country, Keeping in view the Mare = percentage of existing vehicles and those likely to be manufactured inthe near Mie The standards for the dimensions and weights of vehicles should be consistent with road facilities now available or could be made available in the near future. The vate a cic eraetrsties affecting the road design may be classified as static and dynam characteristic of the vehicles. ‘Static characteristics of vehicles affecting road design are the dimensions, weight am maximum tuning angle. The height of vehicle affects the clearance of the over'et ‘inuctures, The height of driver seat affects the visibility distance and the height of be the oi ‘TRAFFIC CHARACTERISTICS 163 affects the head light sight distance at val i Tego depends onthe design of wind shld unde nt porn fhe see ir te clearance below the chasis approach, depanure and ramp pipe vote affects the design of vericl profil of drive ways, humps and dips. The length of verjeles affects the capacity, overtaking distance and maneuverability of vehicles, The vein turing, radius depends on the length of wheel base and the fatares of the imring system and this affects design of sharp curves forthe manoeusre of vehicles a seep apeeds. ross weight, axle and wheel loads of vehicle govem the suc! design Srpavements and cross drainage structures, ‘Dynamic characteristics of vehicles affecting road design are spee, acceleration and raking characteristics and some aspects of vehicle body design. The speed and byeleation depends upon the power ofthe engine and the resistances to be overcome ath are important in all the geometric design elements. ‘The deceleration and braking ataracteristies guide safe vehicle operation, The stability of vehicle and is safe crevement on horizontal curves are affected by the width of wheel base end the height of rove of gravity. The riding comfort on vertical curves depends on the design of SEspension system ofthe vehicle, The impact characteristics on collision andthe injuries spe occupants depends on the design of the bumper and body of vehicle. Some of the Vohiele characteristics have been discussed below in detal light Vehicle dimensions “The dimensions to be mainly considered are the overall width, height, and length of different vehicles, particularly of the largest ones. The width of the vehicle affects the ‘vidth of the traffic lanes, shoulders and parking facilites. If the width of the lanes are ot adequate in view of the widest vehicles using the roads, the capacity of road wil xperwace Height of the vehicle affects the clearance to be provided under structures such or gverbridges underbridges, electric and other service lines. Length ofthe vehicle i 2 fipontant factor in the design of horizontal alignment as it effets the exra width of pavement and minimum turning radius. Lenglh affects the safe overaking disses rivectty of a road and parking faites. The length should also be considered i the find dips. The maximum allowable width, height and length of Vehicles have been standardized by the Indian Roads Congress and are as given 10 Tule 3.1 G@),. The configurations of different pes of transport veicles and their axle ‘arrangement are given in Fig, 5.1. ‘Table 51 (a) Maximum Dimensions of Road Vehicles Viaxima dm. Dan 5 = ; és (exuding fon Wehicte = Wii [Ai weietes ti Weight [(a) Single-decked vehicle for normal application asd (6) Double-dec 73) i ‘Length [(a) Single-unit truck. ith two of more axles ( 12.00 See ant baw nor mores O92 Demy] tao | ¢} Semitrailer tracor combinations (Types 251.75 1—500—] Se ae eombiatons (Types 22.32.29. + 3 ‘units, and no such combination, No combination {s allowed t be of Oe PT Te ex 13 m. The other laden or unladen is allowed to have . the minimum turning dimensions to te considered tre the wheel base (ohh affects the mi 164 ‘TRAFFIC ENGI TE -2 TYPE-3 TYPE 2-S1— TYPE 2 ~2 TYPE 3-51 TYPE 3-2 TYPE 3-2 TYPE 2-2 TYPE 2-3 TYPE 3-3 Fig. 5.1 Types of Road Transport Vehicles radius and extra widening required) the front, rear and ti centre clearances and | approach, departure and ramp angles (which affect the design of vertical profile f highways and driveways). Weight of loaded vehicle The maximum weight of loaded vehicle affects the design of pavement thickness gradients. In fact the limiting gradients are governed by both the weight and power of t heavy vehicles. The maximum permissible gross weights and axle weights have beet | standardized by IRC as per Table 5.1 (b). ‘Table 5.1 (b) Maximum Permissible Gross Weight and Axle Weight of Transport Vehicles AI Lit) i) ‘Maximum Maximum axle weight, tonnes Gross weight, [Truck/Tractor | Trailer Neti tonnes" [FAW] RAW [FAW [Type 2 (Both axles single tyre)| 12.0 6 | 60 Type2 16.0 6 | 102 |(FA-Single tyre RA-Dual tyre) [Type 3 24.0 6_|18 (TA) Type 2-S1 264 6 | 102 [Type 2-S2 342 6 | 102 18 (TAY [Type 3-S1 34.2 9 [18 (TA) [Type 3-S2 42.0 6 _|18(TA) 18 (TA) | [Type 2-2 36.6 6 | 102 [102] 10. [Type 3-2 444 6 | 18 (TA) | 10.2 [Type 2-3 444 6 | 102 [102] 18 [Type 3-3 52.2 6 _|18(TA)] 102 | 18 FAW = Weight on Front Axle; RAW = Weight on Rear Axle TA= Tandem axle, fitted with 8 tyres. TRAFFIC CHARACTERISTICS 165 powor of vehicle ie power of the Heaviest vehicles and their louded weights gove an tng values of gradient on roads. In this regard och oain an ating oF inertia, rolling resistance, ar resistance nd gad resistance ne consiered. From the total hauling capacity and the power required to overcome the total tractive Fiance, it is possible to determine the speed and acceleration ofthe vehicle which in resistin traffic regulation, planning, and design sspecd of vehicle ‘The vehicle speed affects, (i) sight distances (ii) superelevation, length of transition curve and limiting radius on horizontal curves (ii) length of transition curves on vertical Salley curves and on humps (iv) width of pavement and shoulders on straight end on Jorizontal curves (v) design gradient (vi) capacity of traffic Jane (vi) design and control ‘measures on intersections. “Thus the design speed controls most of the geometric features of highways Braking characteristics “The deceleration and braking characteristics of vehicles depend on the design and type of braking system (such as mechanical, fluid or air brake) and its efficiency. The safety Of vehicle operation, stopping distance and the spacing between the two consecutive Vehicles in a traffic stream are affected by the braking capacity. Thus the highway ‘capacity and overtaking sight distance requifements also indirectly get affected. Braking Test It is possible to measure the skid resistance of pavement surface under the prevailing conditions by conducting braking tests on the road at the desired running speed Ifthe brakes are applied tll the vehicle comes to stop it may be assumed tht wheels are folly locked and the brake efficiency is 100 percent. Atleast two of the following three tneasurements are needed during the braking tests in order to determine the skid resistance of the pavement: (Braking distance, L metre (ii) Initial speed, u msec. Gil) Actual duration of brake application, t second od Sages ‘The method of calculating the average skid resistance of the pavement using two of the above thee values has been illustrated wih the help of Examples 5.1, 5:2 and 53- om ipped by applying: aged Ina braking test, a vehicle traveling at a speed of 30 kmph was n brakes fully and the skid marks were 5.8 m in length ‘Determine the average skid resistance of the pavement surface. Solution 30 ‘i Initial speedu = 5 7833" 2 = (see Ba A) Broking distance = 58™™ 3,7 166 TRAFFIC ENGINEERING 3.332 A kid f = fect oom 2x9.8x5.8 =061 Example 5.2 A vehicle travelling at 40 kmph was stopped within 1.8 sec: of the brakes. Determine the average skid resistance, Solution onds after the appt; 5; 40 sc Initial speed u = <= = 11.11 msec; Braking time t= 1.8 sec. Using the relation of motion for uniform acceleration/retardation, mat v=u+tat,v=0,retardationa = "= pT ONT sec? ton From the relation, force F = ma, wf= ¥2 g Average skid resistance f = 2 = £17 9.65 e988 Example 5.3 A vehicle was stopped in 1.4 second by fully jamming the brakes and the skid measured 7.0 m, Determine the average skid resistance, Solution Using the fundamental relations of motion for uniform acceleration/retardation @ v=u +at, as the final velocity v= 0, u=—at Gi) Vv -w=2as 2 gry? s = - 2 = 2 and therefore 2a 2a Given braking distance L = 7.0m=s and braking timet = 14sec a _ 2x70 Average skid resistance f = — = =0.729 B 98xi4? Example 5.4 'A vehicle moving at 40 kmph speed was stopped by applying the brake andthe of skid mark was 12.2 m. If the average skid resistance of the pavement is known 0.70, determine the brake efficiency of the test vehicle. Solution L.A msec, L = 12.2 m, f= 0.70 ‘TRAFFIC CHARACTERISTICS Average skid resistance developed 167 Yin, 2gL 2x9.8x122 = 0.516 ie = OOF _ 100x0.516 Brake efficiency, % = 1008 _ 100x0-516 1, 15, f 0.70 os off Tracking When a four or six wheeled vehicle, such as car or bus (vehicle other than two and three wheelers) negotiates a horizontal curve at relatively slow speed, the rear wheels do not race the same path asthe corresponding front wheels, as explained in Ar. 44.6 under "Mechanical Widening’. At relatively slow speeds when the centrifugal force developed is lesser than the counteracting forces due to the superelevation and transverse friction, the rear wheels follow paths on the inner side of the horizontal curve as compared with the path followed by the corresponding set of front wheels. This difference in distance between the curved wheel paths ofa particular set of front and rear wheels (i. either the set of front and rear wheels on the outer side of the horizontal curve or the set on the inmer side) is called offtracking or the mechanical widening for a vehicle which is equal to PAR (see Eq. 4.15). Thus the off tracking depends on two factors : (i) the Length of wheel base or the distance between the front and real axles of the vehicle and (ii) the turning angle or the mean radius ofthe horizontal curve traversed. Example 5.8 ‘A vehicle has a wheel base of 6.5 m. What is the off tracking while negotiating a curved path with a mean radius 32m, Solution [= 65m,R=32m Po 6s ing - = SS 066m Ofttredting "aR 2x38 5.2.3 Traffic Studies ‘Traffic studies or surveys are carried ‘out to ‘analyse the traffic prey aerae | studies help in deciding the geometric design feature and Ene rena cfficient traffic movements. ‘The trafic surveys for collecting traffic census. ‘The vi traffic studies generally carried out are + (a) Traffic volume study (b) Speed studies (i) spot speed study (ii) speed and delay study los. TRAFFIC ENGINEERING (©) Origin ad destination (0 & D) study “TRAFFIC CHARACTERISTICS tap (4) Traffic Now characteristics ‘on a pneumatic hose placed movements mn Placed across the roadway or by using any other (©) Traffic capacity studs [insor. ‘Traffic count is recorded by electrically ee ney a mop cording the impulses. The impulses caused by vehicles of cent oth pis ty er a i Stuesrian traffic by this method, Other methods of working the by photo-elecrie cells, magnetic detector and radar detectors i rechanical counter is that it can work throughout the day and night for the desired fecording the total hourly volume, which may not be practicable in marual cane te fpain drawback of the mechanical counter is that i is not possible to get the traffic volumes ‘f various classes of traffic in the stream and the details of turing movements Manual counts weight may not be ible to easily record wechanical detectors are (Po Sing study (g) Accident studies or the traffic lop Traffic volume study Traffic volume is the number of vehicles crossing a sectio it wan nt of whe ig ion op wh used units are vehicles per day and vehicles per hour. A complete traffic volume od, may include the classified volume study by recording the volume of various ae classes of traffic, the distribution by direction and tuming movements and the deel on different lanes per unit time. The obj 7 on dite P ime. ‘The objects and uses of traffic volume studies are given (a) Traffic volume is generally accepted as a true measure of the relative i ‘ lume is a relative importance of roads and in deciding the priority for improvement and expansion. (b) Traffic volume study is used in planning, traffic operation and control of existing facilities and also for planning and designing the new facilites. (©) This study is used in the analysis of traffic pattems and trends. (@) Classified volume study is useful in structural design of pavements, in geometric design and in computing roadway capacity. (©) Volume distribution study is used in planning one-way streets and other regulatory measures. ‘This method employs a field team to record traffic volume on the prescribed record sheets. By this method it is possible to obtain data which can not be collected by mechanical counters, such as vehicle classification, turning movements and counts where the loading conditions or number of occupants are required. However it isnot practicable to have manual count for all the 24 hours of the day and on all days round the year. Hence it is necessary to resort to statistical sampling techniques in order to cut dowa the ‘manual hours involved in taking complete counts, First the fluctuations of traffic volume during the hours of the day and the daily variations are observed. Then by selecting typical short count periods, the traffic volume study is made by manual counting. Then bby statistical analysis the peak hourly traffic volumes as well asthe average daily waffic volumes are calculated. This method is very commonly adopted due to the specific advantages over other methods. Presentation of traffic volume data ‘The data colleted during the traffic volume studies are sorted out and are presented in any ofthe following forms depending upon the requirements, (0), Annual average daily raffic (ADT ot ADT) of the total traffic as well as classified traffic are calculated. This helps in deciding the relative importance ofa route and in phasing the road development programme, In order to conver the different ‘hice classes to one class such as passenger car, conversion fctrs known as Passenger Car Units (PCU) are used. (see Tables 52 and 5.3) (&) Trend chars showing volume trends over period of years are prepared. These data are useful for planning future expansion, design and regulation. (©) Variation charts showing hourly, daly and seasonal variations are also prepared These help in deciding te facilities and regulation needed during pak traffic periods, the (@) Traffic low maps along the routes, (ihe thickness of the fines representing traffi volume to any desired scale), are drawn, These help to find the trafic volume 65 a = (from N~$ strips) = 272 22410 65476 16-19 755 min 19.36 we Sade 35x60 =278 7.55 Pe = 19.36 velvmin Joumey speed = Average stopped delay = 1,8 min Average running time = 7.55 -1,80 5.75 min 35x60 S75 Average running speed = = 36.5 kmph Origin and Destination Studies ‘The origin and destination (© & D) study is caried out mainly to (i) plan the road network and other facilities for vehicular trafic and (i) plan the schedule of different modes of transportation forthe trip demand of commuters. ‘The O & D studies of vehicular traffic determines their number, their origin and destination in each zone under study. The data may also be supplemented by the number of passengers in each vehicle, purpose of each trip, intermediate stops made and reasons ete, Origin and destination study gives informations like the actual direction of travel, selection of routes and length of the trip. Thee suis are a ee ja pnang new highway facilities and in improving some of the existing syst ‘exampl there can be a high percentage orth traffic which may be diverted by eoving a by-pass and thus considerable saving in distance and time can be wate © é ‘et Provides the basic data for determining the desired directions of flow is “ ae ae This is considered to be one of the imporant affic sues needed 10 soe Ditty problems in a zone and the most important study to plan the highways) mass transit facilities in cities should Scientific planning of transportation system and it be based on © & D data of passenger trips. Al fae tal ee extrapolating the data from © & D study, together W! Att 5.2.5 for estimation of future traffic). 180, TRAFFIC ENGINEERING ‘The various applications of © & D studies may be summed up as follows () 1 judge the adequacy of existing routes and to use in planning new network = : : Gi) toplan transportation system and mass transit facilites in cities including routes schedules of operation. (ii) t0 locate expressway of major routes along the desir lines. (iv) to establish preferential routes for various categories of vehicle including by-pass, (¥) to locate terminals and to plan terminal facilities. (vi) to locate new bridge as per traffic demands. (vii) to locate intermediate stops of public transport. (viii) to establish design standards for the road, bridges and culverts along the route, ‘There are a number of methods for collecting the © & D data. Some of the methods, commonly adopted are : 5 Road-side interview method, License plate method Return post card method, Tag-on-car method and Home interview method. The choice of the method is made ju Road side interview method The vehicles are stopped at previously decided interview stations, by a group persons and the answers to prescribed questionnaire are collected on the spot. The information collected include the place and time of origin and destination, route, locations of stoppages, the purpose of the trip, type of ‘and numbers of passengers in each vehicle, The traffic may be filtered through a prescribed lane by previots ‘warming signs and with the help of police so that each driver of the selected sample of vehicles is interviewed. The percentage of sample interviewed out of the total traffic in. each selected period should also be noted from appropriate traffic volume study taken simultaneously : In this method the data is collected quickly in short duration and the field organisation is simple and the team can be trained quickly. The main drawback of the method is that the vehicles are stopped for interview, and there is delay to the vehicular movement ‘Also resentment is likely from the road users. Further, unless there is enough spac: undue congestion may result due to stopped vehicles. a License plate method fl ‘The entire area under study is cordoned out and the observers are simultaneously stationed at all points of entry and exit on all the routes leading to and out of the afe® Each party at the observation station is given synchronized time pisces and they note the license plate numbers (registration numbers) of the vehicles entering and leaving the cordoned area and the time. Separate recording sheets are maintained for each direction of movement for a specified time interval. After collecting the field data major work remains of the office computations and analysis, by tracking each vehicle number and it8 \¢ of entering and leaving the cordoned area. TRAFFIC CHARACTERSTi¢g ‘The questionnaire to be filled in by the road user is pri Me eae ae Fant tg in tray be selected where vehicles have to stop as incase of tall nts “The method is suitable where the traffic is heavy. The personnel ned i cr trained just for distributing the cards. Only apart of the ond wer ey toon a in the desired details properly and correctly. If conclusions likely that these may not gives ne picture, Tag on car method In this method a pre-coded card is stuck on the vebicle as it eters the area under study, When the car leaves the cordon area the other observations are recorded on the tag. This method is useful where the traffic is heavy and moves continously. But the method gives only information regarding the points of entry and exit andthe time taken to traverse the area. Home interview method A random sample of 0.5 to 10 percent ofthe population it selected andthe residences ae visited by the trained personal wo cll the tne a fom ach ember of house hold. Detailed information regarding the trips made by the member is obtained on the spot. The data collected may be usefil either for planning the road net work and ther roadway facilites for the vehicular traffic or for planing the mass wansporstion requirements of the passenger. The problem of sopping veile and conequert difficulties are avoided altogether. The present travel needs a lat tno ‘analysis is also simple. Additional das including socio-economic and other deals ny be collected so as to be useful for forecasting trafic tnd transportation Bo have complete coverage ofthe entire cross section ofthe population i ery tedious, While planning for © and D studies ata place it it = Study. The selection of the method is dependent on the ODETTE F te influence of year and dates of study on the type and nora oft ne ‘oun. Care is needed in seleting the method of sm888 Sean Sample size should be decided keeping in view | Soe ‘Work spot interview method seed byoecing be 0D aH The transportati ritrips cn be pli peso neve. portation needs of work trips ons, ee Y ‘Work spots like the offices, factories, edveationl insist Presentation of © and D data “The data are presented inthe following forms * 182 @ a TRAFFIC ED (GINEERING 7 origin and destination tables are prepared showing, number of tips between diferent zones. Desire lines are plated which isa graphical representation prepared in almost all esi soneys. Desire lines are straight fines connecting the origin points with aeianations, summarized into different area groups (See Fig. 5.7), The width of sesh desire lines is drawn proportional to the mumber of tips in both directions, The desire line density map easily enable to decide the actual desire ofthe road, aie and thus helps to find the necessity of a new road link, a diversion, a by-pass ‘ora new bridge. These desire lines may be compared withthe e CENTRE LINE OF c Existine ROAD —— esine tines Fig. 5.7 Desire Lines TRAFFIC CHARACTERIStics ji The eeltive magnitude ofthe gener Ci) THE involved may Be repesencd by aad diameter being proportional tothe number of gy Contour lines may be plate snl io SSrours would indicate general wate neo ats TH ne of he {Weomerrica relationships of the par in which ck we dawn the traffic flow characteristics and studies ‘prafic stream generally has flow and counter flow aln umn is separated nto pa of one-way Qowsby proper dy enepaion ‘phe basic traffic manoeuvres are diverging, mer : Fig. $8. Of all these, diverging onthe left isthe easiest mo and crossing 2s shown in ry, 8, fate, espe eM eee at cn from the left side also does not cause much of conflict. But dives ie: Marge, from Merging ffom the right creste conflicts and hazard to Lon oe igh path. Transfer ofa vehicle fom one tfc lan othe nex a ee seated Zane change and this involves diverging and merging. rest wae es vert er oivercme ee vert ‘ant menses a tery crt cRossiné Sf wert wow weve ) alse Fig. 58 Traffic ManoenT® ae poli erossing traffic is the grees dion in cae of sing oven of eS bests vehicles on one road have 0 s6P Pt at cay Tt Locross thir pth, Thus the wae capacity oft creas When Ne Sf any ane ofthe crossing roads or ve ee sree shisucy aca the path of nate! vel ovine Angle of crossing, the action is termed #6 goes al sa No be considered to consist of merging m= TRAFFIC ENGINEERING isa tn twoay moventents there may be crossing and over taking manoeuyres, Thy crafty: aroun characteristics are attested by the Wide range of vehicles and road useee as SFr of the ron! aa intersections ann other regulatory measures. Funes hs enviromental connitions also attest the traffic stream How, the The poins to be poticulasty stale’ in trafic Now’ are the transverse and longituding, isenbathon of vehicles on the various routes. The gaps ahead of each vehicle determine the longitudinal distribution of Vehicles in one-way stream, See Fig. 5.9, The time interyal bpermeen the passage of sascessive vehicles moving it the same lane and measured fom bread to head as they passa point on the road is known as the tine headway, The benven successive vehicles moving in the same line measured from head at any instance i the sna’ Avan’ ofthe spacing ofthe vehicles in the stream. The variations in he show the longitudinal distribution of the stream. The number of headways per unit time ig Sepengent on the rate of trafic flow and is therefore a direet measure of traffic volume, With increase in speed of the traffic stream, the minimum space headway increases where as the minimum time headway first decreases and after reaching a minimum value at ‘optimum speed on the stream, inereases as shown in Fig. 5.9. Maximum flow or capacity flow is attained at this speed when the time headway is minimygn, 2 “| Frwonane wicewar | ‘ Tt i : pr sam ha . [sacins z 3 he z z | oa ar USERVED SPEED OF PAIRS OF VEHICLES Kmpn Fig. 5.9 Variation of Minimum Space and Time Headways with Speed Another important factor to be studied in traffic low characteristics is lane change inthe traffic stream. When the headway of the lane changing vehicle rapidly decreases to almost zero, the lane change is forced; in all other cases the lane change may be optional. The frequency of demand for lane change will be high when the speed range of vehicles in the waffic steam is high. The lane change manoeuvres and characteristies would very much depend on the number of lanes and whether itis one-way or two-way movernent. The merging, diverging, weaving and overtaking operations, all come under lane changes. Study of traffic flow characteristics includes both transverse and longitudinal distribution of vehicles in the trafic stream and this is useful in geometric design features such as traffie capacity, volume, number of lanes and width of carriageway. The study is also very much needed to decide traffic regulatory measure like one-way movements and for the design of traffic control methods. Traffic flow study is particularly essential for large intersections, ‘Traffic capacity studies Before studying details of traffic capacity, it may be worth while to define some of the related terms which are often used. TRAFFIC CHARACY ERISTICs, ratfe volume is the number dune oF roadway CE pass gly Fratic volume 1s expressed as yh OF Vehicles moy i ing ina 3 po Yee direction hls pr hoe ton dr pete wt ar ; Pet ou or cies pa A te Ir ea eR Pre igo given instant, ‘Pressed a5 vehicle me of lane of road faoduct of the tac density and atic peg” PNM Trafe volume ae ‘The highest traffic density will occur when cna sven route, and inthis ease traffic volume pte capaci ees as the maximum number of veicle ina toe aye te vole. I font in unt time, usually an hou, ie, whites per her {volume ae mesturesoftalfic Now and fave he are cer ant offload ponds tts ee Ame ln ra capability oF maximum rat of flow witha cena level of seine chase ea pe carried by the roadway. The capacity ofa a roadway and traffic conditions, roadway depends ona nunber of prevaing jhicles are practically ata I apposch e tea H Hand tn Basic capacity is the maximum number of oi eee mim if ng st on conditions which can possibly be attained. Two roads having the same physical features will have the same basic capacity irespective of traffic conditions, as they ae assumed to be ideal. Thus basic capacity is the theoretical capacity. Possifle capacity is the maximum numberof vehicles that can passa given point on a lane or roadway during one hour under prevailing roadway and traffic conditions. The possible capacity of a road is generally much lower than the basic capacity as the prevailing roadway and traffic conditions are seldom ideal. In a worst case when the prevailing traffic condition is so bad thet due to traffic congestion, the taffic mey come toa stand still, the possible capacity ofthe road may approach zero, ‘When the prevailing roadway and trafic conditions approach the ies! conditions. the Possible capacity would also approach the basic capacity, Thus the value of possible ‘spacity varies from zero to basic capacity. For the pupose of design, neither base ‘2pacity nor possible eapacity ean be adopted as they represent two extreme cass of ‘radway and traffic conditions. Practical capacity is the maximum numberof vehicle that can pas » sien pnt on 3 Jane or roadway during one hour, without traffic density being so great Sie Unreasonable delay, hazard or restriction to the driver’ freedom to mapost% © Prevailing roadway and traffic conditions. It isthe practical capacity bea Dal es als Interest to the designers who strive to provide adequate highway facilites ‘also called design capacity. Petermination of theoretical maximum capaci, , tees wwe fo imate of theoretical maximum or si cpio 8 'm the relation : may be made 63 1000 $s i , “®. © = capacity ofa single lane, veicle pet ot V = speed, kmph TRAFFIC ENGINEERING 186 , centre spacing of vehicles, when they fol erage centre 10 = average ey a queue or space headway. m the other asa quel OW one beg | ie e speed V and spacing S. The gS capacity depends upan the spec aiemg! stan Thus the ee eer vehicles is equal to the average length of venice berween oe paweet the vehicles in the stream. The minimum clear spacing tags clea spacing ee or sae stopping of the rear vehicle in case the venep seh ae ae aye found that rivers follow the vehicle ahead ata ely suddenly 0p nd the clear spacing i inerexsed instinctively at higher sponge 2 low traffic stream (Fig. 5.9). a ‘Thus the space gap allowed by the driver of a followed vehicle depend on factors such as (i) speeds of leading and following vehicles (ii) type and characteristics of the two vehicles (ii) driver characteristics ofthe following vehicle hi (iv) traffic volume to capacity ratio of the road section at the instant or the level of service (8) The proportion of vehicle classes in the stream (oi) road geometries and (il) environmental factors The assumption that space gap increases in direct proportion with the speed of ie vehicle or that ofthe trafic stream is therefore a very much simplified one and gives of an approximate average value of the space gap between vehicles in the traffic stream The space gap allowed by the following vehicle in a traffic stream is some time assumed to be equal to the distance travelled during the reaction time of the driver, assuming | the braking distances of the lead and the following vehicles are approximately equal. the reaction time is tsec., the minimum space gap Se is given by : S @ 8 =v 1) 278 Vt,m ‘where v and V are average speeds in m/sec and kmph a The minimum space headway S ina traffic steam is therefore equal tothe mining ‘space gap plus average length of vehicle L in the stream a S = Sp+L=0278Vt+L mn @ stream flow, as the driver of the following vehicle is quite alert, the over reaction time is found to be low; this value is often assumed to be 0.70 to 0.75 ae this analysis of overtaking sight distance (Art. 4.3.3) the value of reaction time ‘ssumed a5 07sec. in the empirical relation for spacing, i.e. ae (iy a= O.7v+L)=O2V+L),m Thus a suitable value of $ may be adopted in Eq, 5.3 to estimate the theore! ‘capacity ofa traffic lane with homogeneous traffic flow. ic 1 hes been observed (as explained earlier) that with increase in speed of the tal team the time headway decreases and after reaching a minimum value at i ae trans ineteasing (See Fig, 5.9). The maximum theoretical capacity of 8 Imay therefore be obtained if the minimum time headway Hi, is known. {ivy -¢ C is the capacity, nwa in second. ‘The relationship between speed Figg, As the speed is increased mae deereasing due to increase in headway at the speeq range, “PY Of the lane stars TRAFFIC, CHARACTERIS Tics sarin 7 a Vehicles per hour ¢ (3600 second), and } 2nd His the minimum time capacity fa traffic lane et is shown futher, the mani che optim oa [TWo= LAM Hera eeuimum capacity oF oe cane e erage : zo a Fig. 5.10 Speed and Capacity Factor affecting practical capacity Some of the important factors that affect the practical capacity of a trafic lane are listed below : Lane width : As the lane width decreases, the capacity also decreases. The practical capacity of 3.0 m wide lane in a two-lane rural road may decrease to 76 percent of the capacity ofa 3.5 m lane. Lateral clearance : Vertical obstructions suchas retaining walls, r parked vehicles near the traffic lane reduce the effective width of a lane and thus rest in reduction in the capacity of lane. Further, restricted lateral clearance effects driving comfort nd increases rates. A minimum clearance of 1.85 m fom te pavement edge tothe Sbstruction is considered desirable so that capacity isnot affected adversely. When the distance from pavement edge to an abstucton deren to 0.75 m on one side Only, the capacity decreases to 96% and when this obstrectin is on both Percentage further decreases to 80% of the standard design capacity {idth of shoulders: Narrow shoulders reduce he efetve wih ofa Tes Vehicles travel towards the centre of the ae ies 4) as to park 00 the emergency (like that of a tyre other trea oid resulting Shoulder of insufficient width, there is reduction in efetie “reat reduction in the capacity ofthe lane. : ruck ad buses CUP Commercial vehicles : Large commercial vibes Ie WE he vehicles tr space and influence the other aici Be SM Ey my wave at none the adjoining lanes, Also these heavy com "Wet speeds especially on grades. ‘TRAFFIC ENGINEERING 188. nent and geometrics are not upto the desireg ae Ie uhe align iclarly, restrictions to sight distance nt ty ‘steep and long grades affect the capaciy. Wena stand OSD, the capacity decreases to gg lignment 0 ane cause reduction in apacil of the road Jength has sut ‘standard design capacity cp imersections at grade : tersections restrict free Now of trie (vi Presents iy afet the capacity, The capacity of an intersection ote ts ow at grade wll be slightly less than the road with lower capacity of hey cree rnalized intersections asthe vehicles have t0 stop altemately to allow ‘talc the capacity ofthe intersection willbe further decreased. In order tp Consistent traffic flow and maximum capacity on important highways, is ) ( time for eross raed plus an initial interval when pede init 2 2055 minus amber period. hs ‘minus amber period for the cross road. (©) with pedestrian signal the i 4essthan seven seconde equal to red time for jan clearance time bs itil interval is the WALK peri (b) where no pedestrian sions): is inital merece” SBMA i used, a minimum period of five seconds ' 18) js cents Road a = ib iss Needed is then increased based on the ratio of 22 = 0secott traffic volume per hour per lane, THe oe Road B= 73. the next higher Scent | os TRAFFIC ENGINEERING sing 7 secs. for initial walk-period, minimum red time UH) ARES Tat foe rad Bs (10 # 7) ses. i») Minimum green times based on pedestrian criterion : ‘ 8 RoaB = 1S+7-3=19 secs, for road Aig TRAFFIC OPERATIONS a ey RoadA = 10+7-4=13 secs, (9) Based on approach volume, the green time calculated is increased fy Roa higher taffe volume. i G. ny Use relation =A. = TA Gang 35 Gx and Ga are green times and na and ng are approach volume per lane oe Gg is taken as 19 seconds as in iv) above. . Green time for Road A, Fig. 531 Traffic and Pedestrian Signal Settings Table 5.9 Details of Design Timings (Ex. 5.13) x aime Fae mamta eran | VEN dean era i / Pern | Wa Pein eal Tale | (i) Tota cyte length ~ OA+AR+RA=Gy+Aq+Gn 4+ Ap wis 3 fas 2s) for Bslto as] = 23244+19+3=492 secs, 119.00 REHOME) Tow nars rahe | Hence adopt cycle length of $0 secs, 08 ’ - ‘ | The adiltional period of 50 — 492 = 0.8 i wu) is wa HS | = 0.8 secs. is distributed to green 4 120) Icgat9s}Ry~225 Propestion to approach tat i936 low.-225 Pr ic volume +s lezaae{w_2236|!925| cristo] 26 isaac Ga = 232+0.44=23.64 secs, 6 306 ‘ a a Ga = 19.00.36 = 19.36 secs, we oat — Trae signa Pedestrian sigs = Gp+Ag=19.56-+ 3,0 = 22,36 secs, Fer enane) Be eer neni Bet a Re = Ga+ Aq=23.04+ 4027.64 secs Sateen gor DWdo not wa 04 + 4.0 = 27.64 secs. a eet a Pesign of pedestrian signal discedponde Note Amber period of shor itera however be provi Byes i eos forgetting ready to ross. This eid par oft ed pdf signal ap, (OW) Petiod of pedestrian signal at road A (PSq) is red period of! G) Webster's method feast total delay 0 the For PS,, In this method, the optimum signal eycle Co co tae a is a tational approach i DWa = Ra=27.64 soos Tylcles at the signalized intersection has been worked ittime on each approach ‘or PS, ‘ i ree DWa = Ra=22.36 sees, E B for eosang BE intervals (Cl) ae of Is sing from (ii) above? The walk time (" W + qySt ae ; io yt = grSi an 72° SE nd 10 sees, respectively, forroads AM Sed on the higher value of normal flow, the Taio 31 = 4 TT cecessary to fined aie , mr : 1 and 2, Inthe ease of mixed act) For, W) is calculated rom ttl el Sone a te ee rt fv For PSp, Wa 50-0764 15)=7.36 secs. ; “Neh shoud be determined separately sat ik sues ty nig na =n Wa = so- Theater ed from cate! pnd the correspon Details of design timin 5922.36 + 10) = 17.64 secs OF yehigettration flow is to be obtain pase ‘may be drawn, a ate the ing les inthe stream of compact flow during shown inti butted in Table 5.9, Alternatively a phas® 4 we ‘TRAFFIC ENGINEERING x isely. tn the absence of data the approximate valug dime ie reing 160 pou pet 03 mete widh ofthe approach, Tyg the wraffic is als determined onthe approach roads from the field studeg period (during the peak or ofF peak hours asthe case may be). et “The optimum signal cyle is given by : _ LSL+S 1-¥ where L = total lost time per cycle, secs. = 2n + R (n is the number of py allred-time) Y= nity = 1@- = ‘Then, a ‘y (Co~ L) and Ga, y (o-L) Example 5.15 ‘The average normal flow of traffic on cross roads A and B during design 400 and 250 peu per hour; the saturations flow values on these roads are os 1250 and 1000 peu per hour respectively. The all-red time required for crossing is 12 secs. Design two phase traffic signal by Webster's method. Solution a> Yat Yo =0.3240.25 = 0,57 2n+R=2x2+12= 16 secs. c= SLES _1sxt6es Y "1-057 29 day *97A Sy, 675 sees. Ya YF o-L= 2 (675 ~ 16)=29 secs. Y% FY Co-Ey= 925 (675 _ 16) = 20.5 secs ke. : 037 ed time for pedestrian i Providing Amber a times. WeWSH IDeA mG7 sect 2 SECS. each for clearance, total ‘rossing = 12 secs, TRAFFIC OPERATIONS + Sketch a phase diagram as shown in pent Tass Fit 529 07531 or prepare a able to show {q Desig Method as per TRC Guideline The pedestrian green time required forthe 4 Fased on walking speed of 12 misc, and the minimum green time required forthe roads respectively. ji) The green time required forthe vehicular traffic on the major road is increased (© roportion tothe traffic onthe two approach rds r (Gi). The eye time is calculated afer allowing amber ime of 2.0 secs. cach ‘Note: The steps mentioned above are sinilar to the Approximate Method explained under Method (2) and Example 5.4, i (iv) The minimum green time requited for clearing vehicles aiving during a cycle is determined for each lane of the approach road assuming thatthe frst vehicle will take 6.0 secs. And the subsequent vehicles (PCU) ofthe queue will be cleared ata rate of 2.0 sees. The minimum green time required forthe vehicular traffic on ny of the approaches is limited to 16 ses. (0) The optimum signal cycle time is calculated using Webster's formula (explained in method 3). The saturation flow values may be essumed as 1850, 1850, 1950, 2250, 2550 and 2999 PCU per hour forthe approach roadway widths (kerb to median or ‘entre line) of 3.0, 3.5, 40, 4:5, 5.0 and 5.5 m; for widths above 5.5 m, the saturation flow may be assumed as 525 PCU per hour per mete width. ‘The lost time is calculated from the amber time, inter-green time and the inal delay of 4.0 secs. forthe first vehicle, on each leg. (vi) The signal eycle time and the phases may be revised keeping in view the preen time required for clearing the vehicles and the optimum cycle length determined it steps (iv) and (v) above. ‘The design method is illustrated in Example 5.16. ‘major and minor roads are calculated ‘nkal waking time of 7.0 secs. These are Vehicular traffic om the minor and major Example 5.16 ‘Ata right angled intersection of two roads, Road 1 120 m and Road 2 has two lanes witha ttl wit approaching the intersection during design proaches of Road | and 278 and 180 PCUour on the wo Design the signal timings as per IRC guidelines. Solution Design traffic on Road 1 = higher of he 0 aS *450 PUM. Design traffic on road 2 =278 PCUMr volume per lane = 90072 5 120, pom tT es. (0) Pedestrian green time for Road ! 12 = 88 4709125 8 Pedestrian green time for Road? 12 ‘TRAFFIC ENGINEERING 230 Road 2, Gz © 17.0 sees, TRAFFIC OPERATIONS Green time for vehicles om : ual marking Road oF traffic markings are made of | ihe pavement, Kerb, sides of islands or on the fixed ob) Mie abd (or tefiectors on rae markings may be called special sign inane St the roadway he aie. The markings are made usig pi onto, the pavement or ‘ihe back round. Light reflecting pains are trafic marking. In order to ensure that the mar Tongtudinal lines shouldbe atleast 10cm tick ad the tannin cn in such a way that they are visible at sufficient distance in advance te san sen ae adequate time to respond. ‘ive road users ‘The various types of markings may be classified as, (a) Pavement markings (b) Kerb markings (©) Object markings and (4) Reflector unit markings 450 Green time f aye 17x = 27.5 sees, (ii) Green time for Road 1, Gr * 17 35g ees, hae ss (ity) Adding 2.0 sees, each towards clearance amber and 2.0 secs, inter cash phase total eee time required = @+17+2)+ 42754 qr time = $5 secs. The extra 2.5 sees. per cycle may be apportioned to the green times of a5 1.$ and 1,0 secs. and so Gy =27.5 + 1.5 = 29.0 secs. and Gz = 17.0+ 19 tiv) Ve arrivals per lane cycle on Road 1 450/83 = 8.2PCU Minimum green time for clearing vehicles on Road 1 6+ (82 —1.0)2= 20.4 secs. Vehicle arrivals per cycle on Road 2 > 278/5S=5.1 PCU Minimum green time for clearing vehicles on Road 2 = 6465.1 1.0)2= 14.2 secs. As the green time provided for the two roads by i i te >y pedestrian crossing eriteri above are higher than these values, the above design values are alright. tS (©) Kost time per eycle = (amber time + inter-green time + time lst for in rst vehicle) fortwo phases = (2 +244) x 2 = 16 sec. Saturation low for Road 1 = 525 x 6= 3150 PCU/hr Pavement Markings Pavement or carriageway markings may generally be of white paint. Yellow colour rmarkings are used to indicate parking restrictions and for the continuous centre line end barter line markings. Longitudinal solid lines are used as guiding or regulating lines and are not meant to be crossed by the driver. Transverse soli lines indicat the postion of stop lines for vehicular traffic. Some of the common types of pavement markings are given below : (a) Centre Lines : These are meant to separate the opposing streams of trafic oa Undivided owo-way roads. On rural highway with two or three lanes, single broken lines of width 0.1 m and length 4.5 segments and 7.5 m gaps may be painted on straight stretches of NH and SH, these may be decreased 10 3.0 and 6.0 mat horizontal curves and approaches to intersection. On other roads at straights the sezments are 3.0m in lengt® and gaps 6.0 m (which are reduced to 3.0 m at curves and approaches fo inersecto0), On four or six lane undivided roads two solid continuous parallel lines of 0.1m width with 0.05 to 0.10 m space in between are painted. es On urban roads with less than four trafic lanes the cen ine consists of white bok lines of width 0.10 to 0.15 m, length of segment 3.0 m and a et Ae Teduced to 3.0 m at curves and approaches to intersections, On undiviind DSN" aleast ie trafic lanes for each direction of ale flo he cen ine makin ‘onsist oF two solid continuous lines. sed to guide the (©) Lane Line : Lines are drawn odesanae trafic es: TH 2 Saturation Now for Road2 = 1850+ 49%3 ~ 1974 pcume 3 900 Y= F5p 70.286 and yo Y = 0.286 +0.148 = 0,434 Optimum eyete time Cg = ESL+S _ 15x1645 iT = 51,2 sees. “affic and to properly utilize the carriagew3Y- ‘ ing is not ‘Therefore the eyele time of eS (©) No Passing Zone Markings : These ae marked to nt overtaking signal timings are given pete oe Ssigned earlier is acceptable. THE Pemmited Iateral 355m Fig. 29 and'S.31 or ina tape ee pea) either be shown in the frm of G al nea intersectnn 1 deste PPS! . la formas rena pil), Tirm Markings: These are we ons, . Tong Green Aaa | Re ; Passat vices bore ua 9 sje it coon Toe ss 2 ie (©) Sto : jean for vehicles pand pce Re 2 +2 °p Lines : These are mé top oad 2 18 2 os “etalned intersection etc. wher the veils BNE ‘TRAFFIC ENGINEERING sacs Wolk Lines : The particular places where pedesty ae oe “parked by the pavement markings. ‘The atte Peveer ay be between 20 and 401m depending on local requiremen, (@)appraahto Oberon: These may be indiated by appropriate py (a) Parking Space Lins: For proper utilization of parking facility, may (i) Border or edge lines indicate carriageway edges of rural roads which stones along the edges. ; h @ Route aie arrows are marked by one or more arrows to guide ef traffic into correct lanes. (8) Parking space limits on urban roads are marked to promote effi parking spaces in a systematic manner. Y (0) Bus Stops : The length of kerb which is reserved for buses to stop ae mar : ‘continuous yellow line on the kerb indicating ‘parking prohibited”. ‘Th ‘meant for bus stop is also marked by the word ‘BUS’. Kerb Markings These may indicate certain regulations like parking regulations, Also the the kerb and edges of islands with altemate black ine i ae cde te black and white line increase Object Markings Physical obstruction on or Properly marked. ‘Typical obs ‘rossing gates, traffic islands, Reflector Unit Markings Reflector markers are used as during night. Hazard markers rel *¢ of about 150 m, Road Delineaiors wo i de est" ae devices or treatment to outline the roadway ora portion vpassistance to drivers about the alignment of a road ahead, near the roadway are hazardous and hence tructions are supports for bridge, signs and sir narrow bridges, culver head walls etc, hazard markers and guide markers for sft flecting yellow light should be visible from: night. Three it ms Makers and Objos age =O" that may be used are Roadway Indico" tors are in the for i ‘ black and white se i¢ form of guide posts, 0.8 to 1.0 m high and paint, ofthe roadway so at ay oF Without reflectors and are intended to delineate ‘Traffic islands Traffic islands channels through whichis’ *€25 Constructed within the roadway to establish mor than one eh te Vehicular trac non be guided. Trai islands O88 TRAFFIC OPERATIONS, 23 ‘re taffic islands may be classified based onthe forton at {i Divisional islands (iy Channelizing islands iy Pedestian loading islands (iv) Rotary, Divisional islands ate intended to separate opposing flow of wafic on a highway with four or more lanes. By thus dividing the highway into two one-sy rousys the bend- Gn eolisions ae eliminated and in general cher accident are also reduced. The mist of {he divisional islands should be large ifthe head light glare is to be reduce during night thiving, The kerb should be high enough to prevent vehicles from ertering ito tee islands. ‘Channelizing #Stands are used to guide the traffic into proper chancel through the imerseetion area. Channelizing islands are very useful as trafic control devices for intersection at grade, particularly when the area is large. The size and shape of the ‘Mennelizing, islands will very much depend upon the layout and dimensions of the fmersections. Considerable professional experience and skill is required for tbe ccessful design of channelizing islands. Ifthe islands are not properly designed and placed, there is a possibilty of violation of rues be the wai resulting ia greater fazards. The various uses of properly designed channelzing islands ae listed below = i i This is ilstrated ji) ‘The arca of possible conflicts between trafic stream is reduced. © ih Fig, $32. By introducing chanslzing islands bot the major and minor cont areas are reduced. (ji) They establish the desired angles of crossing and merging of wali seams (ii) They are useful when the direction ofthe flow is tobe changed ‘rai contol devices. (Gv) They serve as convenient locations for other () They serve as refuge islands for pedestrians. (©) Area of conflict without channelei"® Fig. 5.32 Com! anes TRAFFIC ENGINEERING 234 ‘The design and factions of rotary islands, has been discussed in dey « under traffic rotary 4 Pdesrian loading islands ate provided at regular bus stops and simigg protection of passengers. A pedestrian island ator near a eross walk tof gedestran crosing the caiazeway may be termed as pedestrian refuge ig crossing mukilane highways, pedestrian refuge island after two or three desirable. ‘The area in the roadway adjacent to the kerb which is ke pt rese stopped bus may be called a bus kerb loading zone. ved Rotary island isthe large central island ofa rotary intersection; this san larger than the central island of channelized intersection, The crossing re converted to weaving by providing sufficient weaving length. Further details under Rotary Intersection in Art. 5.4.2. 833 Control of Access on Highways Ifeffective access control is not affected along a highway facility, ribbon develo and encroachments follow, resulting in increase in the number of acciden considerable reduction in level of service for vehicle operation, ‘The control of aeee either be full or partial Full conttol of access on highways means that the aut conta the access is exercised to give preference to through traffic by providing Sonnections with selected public roads only and by prohibiting crossings at g direct private drive way connection. When there may be some private connections and some crossings at grade, this is called partial control of access: 4 arterial highvvays for motor trafic with full or partial OF access and zenerally provided with grade separation at intersection. Arterial hig ae nee meant for through traffic, usually on a continuous route and have es access Express ways are divided Major corridors of inter-cit traffic are increasing in importance and afe t protected from unregulated road rome np Side development by exercising limited access: Grade separation across highways ms ic i ed te " i fv highaye i the AADT of fist veces ony ta eee nee e icles only on the cross road within next five exceeds S000, Grade separation should be monided nee cutie vale crete ae Product of ADT of fast road vehicles and the number of trains Toon a within the next five years; in the case of new constuction IKE owes even IF this figure exceeds 25,000 the grade separation may be justified. ‘54 DESIGN OF INTERSECTION SALI General AA the intersection the moveimenis may be handled grain oueh taming and crossing trafic and these sec IS teak st ferent ways depending on the type of intersect eirlly imersection problems are wravoidsbre except i? @ Such problems are avoided by provid, peta ccess. The “9. 5 Nena etersetion procedure increased produce hazards and ed into 40 broad groups : DESIGN OF INTERSECTION Rae jp Imersection at grade : These include all Co eee The tafe ménceuvres te nan the intersections at grade, ii) Grade separated intersection : The inerseting roads are separated by difference in (i) eel. thus eliminating the eressing manoearne Sealy a | roads which meet t more of less the same ‘merging, diverging and crossing ace involved in some of the traffic factors 1 be considered in intersection design are relative speed and maneuver areas. Relative speed is an important fector in traffic low at grade. Relative speed is the vector difference in the velocities of two vehicles in the same flow and is the sum of the Speeds of approaching vehicles from opposite direction, Ie the speed of convergence of Nthicles in separate traffic flows as they approach a point of potential collision, Reative Speed is dependent on the absolute speeds of the imersecting vehicles and the angles erween them. Whein the angle of merging is small, the relative speed will aso be low If there is a collision between two vehicles at small angle at about the same speed or a iow relative speed, the impact would be much less than when vehicle collide at high relative speed. AS the relative speed increases, the judgsment of drivers regarding time ‘nd distance is likely to be more inaccurate, thus increasing the possibilty and severity of accidents. Thus in intersection design care has o be taken to keep the relative speed low. ‘areas are those areas where, in actual manoeuvre, there is a potential calsion nd ae tee hes of och an epee we Be mince influenced. Elemental manowore areas are those fomed by ly 1 single one-way lanes of flows when they diverge, merge orcs, hee being the simples of th manoeuvres, But in multiple manoeore areas where more than two onelans oa ows are present, afc operatins are much more complex and ene re be ee section design. The poi wherethe pose path of two vices nt points andthe area containing all possible confit pons ste cont area. In intersection design the confit area, expecially the mor const ‘more than one vehicle is subjected to conflict simultaneously ho SA.2 Intersection at Grade Al road interseetions which meet like merging, diverging, crossing, and intersections may be further classified 1 lowing vafic manoeuvres a shout ec neonate. TR Serre anckannlized, chamselized and rotary intersections, ‘The basic requirements of intersection Fee mca + a jet shoul (i) Atthe intersec te fn oth be aml (i) The relative speed and parsiulary 1 mene iit) Adequate visibility should be availble for vehi (iv) Sudden change of path should be avoided. a (©) Geometric features like fring rads a ® 2 Provided. (i) Proper signs should be provid drivers aching te intersection. (ii) Good tighting at night is desir — nl TRAFFIC ENGINEERING, 26 sans ant eyelsts ae Lage, Separate provision gpa (sytem of Fee ee intersections wih High volume or m te, made for their sale : trai Paty ous forms of intersections are shown in Fig. 5.33. = “The various foems of i Toto i Fig. $33 Forms of Intersections Unchannelized intersections ‘The intersection area is paved and there is absolutely no restriction to vehicles to ‘3y part of intersection area. Hence the unchannelized (all-paved) intersections are the Towest class of imersection, easiest in the design, easiest in the design; but most complex in traffic operations resulting in maximum conflict area and more number of accidents, unless controlled by traffic signals or police. When no additional pavement wit ‘uring movement is provided, itis called plain intersection. But when the pavement ‘widened at the intersection area, by a traffic lane or more, it is known as flared intersection. These have been illustrated in Fig, 5.34 alongwith common types of unchannelized intersections. Th a $s and the area are suitably designed to ccommnial the channelizing islands of proper size and shape, These islands help to channel ic turing traffic, to contol theirs speed and angle of ay decrease the area atthe intersection. Some of the typiel cog each and to deer intersections are considered superior to the vehicles will have 10 Stop while the other DESIGN OF INTERSECTION oe Ve "4 =x = ety ast ty It Jee = a, = a o oy oH sc ene ees Fi $24 cape nln il} 4 NX See SoS QA . ) a I a ig. £35 Chanelze 0 a ‘TRAFFIC ENGINEERING : 238 The advantages of channelized intersections may be summed up as folloys (i) By cnaization vehicles canbe confined to definite paths Gi) Angle of merging steams can be forced tobe at fat angles so as t cause disruption. {iii) Both the major and minor conflict areas within the intersection can eg decreased, as shown in Fig. 5.32. (iv) Angle between intersecting streams of traffic may be kept as desired in af way. (9), Speed conto an be established over vehicles entering the intersection, (i) Refuse islands can be provided for pedestrians within the intersection area. (vii) Points of conficts can be separated. (vii The channelizing islands provide proper place for installation of signs and ¢ ttaffie control devices. Rotary intersection ‘A rotary intersection or taffie rotary is an enlarged road intersection Converging vehicles are foreed to move round a large central island in one (clock wise direction) before they can weave out of traffic flow into their rections radiating from the central island (see Fig. $36). The main objects of pro id 2 Felary are to climinate the necessity of stopping even for crossing streams of vehich and to reduce the area of conflict, The crossing of vehieles is aveided by allowing ‘icles to merge into the streams around the rotary and then to diverge ou tothe d fadiating rosd. Thus the crossing conflict is eliminated and convered into westlig ‘manoeuvre ora merging operation from the right anda diverging operation tothe left. Design Factors of Rotary Various design factors to be consi ‘sland, radius oF rotary roadway, weaving angle, wes roadway, radius of entrance superelevation, grade, lighting and signs. consideration. Though there is no need for stop before allowing eross traffic speed. With these in view the desi mph for rotaries in rural area when one allother cases and for rotaries in urban areas, a speed 30. mph is adopted for design: (il) Shape of cemrat island : ‘The shape of the central island depends on the Si! rae or eau of the intersecting roads. The outline of te alee rene ara onbet cunes of large ra ‘The various shapes considered to suit dif serene ar, cca, epic turbine and tangent sae ea Ce advantages and limitations (see Fi When two equally important roads 6708S roughly right angles ie. all the four radiating rosde placed symmetrically, a circult without comers DESIGN OF INTERSECTION 29, ceuyoar x ace Jer Da Islands ! of RoIaTy Fig. £37 Shapes - ‘TRAFFIC ENGINEERING. / 240 a t often elongated to accommodate j ot er much elongation and tangent shape are also not desirable 3 sett otra inthis direction to move much faster. Turbine shape fons ten peels of vehicles entering the rotary and enables speeding up of Vehicles 4 however at night, the head light glare i a limitation of the design, go i y roadway d round the c: Radius of rotary roadway The one-way rotary roac 1 Central ig sistent adi st diferent points depending on the shape of the central stand Adequate supereleyation cannot be provided on the rotary roads and hence ig to neglect the superelevation and to take friction only into consideration in the and 4.9 to arrive at the allowable radius of the curve, y2 127F ‘The values of the design coefficient of friction ‘P are taken as 0.43 and 0.47 speeds 40 and 30 kmph respectively, after allowing a factor of safety of 1.5. The suggested the radius of entry curve to be 20 to 35 m and 15 to 25 m for rotary: speeds of 40 and 30 kmph. ‘The recommended minimum radii of central island are times the radius of entry curves. Though these radii are for the rotary practic it is convenient to design the central island to conform to the above radi (iv) Weaving angle and weaving distance : The angle between the path of a ‘entering the rotary and that of another vehicle leaving the rotary at adjacent road, crossing the path ofthe former is termed as the weaving angle (See Fig. 5.36). Veh entering the rotary from a road and leaving towards another radiating road have (0 ‘merge into the one-way trafic flow in the rotary roadway around the central islan then weave out to ‘weaving length. For smooth flow of traffic the weaving angle should be small but not less than 15° ‘Pe,iameter of central island required will be too large. For any design speed l freedom of movement on a rotary depends on the size of the weaving area. The wea tenath should be atleast four times the width of weaving section. The recommen ugc weaving length are 45 to 90 m for 40 kmph and 30 to 60 m for 30 kph del speeds. (s) Width of carriageway at entry and exit The ageway width at the entra! and exit ofa rotary is govemed by th tafe euacine hero the 1 or that ving the tay toe road a aM tering the tay om (si) Width of rotary roadway : AM the tattie entering the trafic rotary have short distance. As the outer ketb lt DESIGN OF INTERSECTION a she width of non-weavng tection eof he tary shan : ry tothe rotary and should generally be phate < cobehe ws to the widest single - - : Sih of weaving section W of the rotary shouldbe ne nae Coane ection. The lane wider width ofthe entry and non-weaving section if than the mean we [fe $25] Fe oe (wii) Entrance and exit curves: The curve traced by the inner tear whee! of vehicles deter the radius and shapes to which the ket lin isto be set. A vehicle entering a rotary has to slow down to the design speed of the rotary and therefore the radius of the ‘Entrance curve should be the same as the minisium recommended radius of the central {sland, For the design speed of 40 kph the suggested radius at entry curves is 20 to 35 mand for 30 kmph, 15 to 25 m, It has been seen thatthe buses and truck can ake right Mngled turn easily at these curves at the design speeds, Where practicable three anty curves may be provided instead of simple cisulr curve Vehicles leaving the rotary would accelerate to the speed of the radiating roads end hence the exit curves should be of a larger radius than entry curves one anda haf to two times radius of entry is considered reasonable. should 0 lanes in e normal pavement width at entrance and exit should be equivalent to two lanes an to prevent clustering of mined tfc tte approsches. Ena wien has to be provided at the entrance and exit curve. ‘The pavement width at entrance curve will be the former is less than the larer. i i + The’ practic ofthe rotary is dependent on Capacity of the rotary: The’practical capacity ny iam capacity ofthe individual weaving section. The expacy is called fom he er than a exit curve asthe radius of | 280. (1+ eW)(1-P) ae Ons (1+ W/L) ing sect in pu per hour. Where Qp = practical capacity ofthe weaving section of rotary in peu Per W = width of weaving section (6 to 18m) ¢ = average width of entry and width range e/W = 0.410 1.0 a L. = length of weaving section between ane of channetizin metre for the range of W/L = 0:12 proportion of weaving trafic ve" > cof non-weaving section efor the —b+E inthe range 0410 10 Po Gebeerd 2 left tuming traffic moving slo = right taming irafie moving 008 ght wile eneing the 7927 b = crossing/weaving traffic rains ing te ra) seat tile I A ing © = crossing/weaving trafic tur" ere nec TRAFFIC ENGINEERING m2 odin the calculated cay Some corrections have been suuggestet c capacity vay Abe aan Stand intemal angles an the pedestrian trafic inthe rotary in IRC has recommended the following PCU values for finding the capacity ofthe, Cars light commercial vehicles and three wheelers = 1.9 ‘Buses, medium and heavy commercial vehicles = 2.8 Motor eyeles, scooters = 0.75 Pedal cycles = 0,50 ‘Animal drawn vehicles = 4 t06 (ix), Channelizing islands : Channelizing islands should be = tnd exit ofthe rotary to prevent undesirable weaving, and tuming a ‘alae ond confit, Furher these channelizing islands help in forcing the vehicle non speed to the design speed of the rotary and to serve as convenient pee a traffic signs and as a pedestrian refuge. The shape and size of chatmelci governed by the radius of the rotary the radii of the entrance and exit curve angles and layout ofthe radial road and rotary. The channelizing i provided with kerbs 15 021 cm high + Tie taming aie Sutin eeae ot agua cg? Heme eee (xi). Sight di a posite ‘eal aap ‘rade : The sight distance in the rotary should be a8 Thinimum sight di case less than the safe stopping distance for the design speed. stance should be 45 and 30 m for design speeds of 40 and 30 ki © 2 rotary on level ground. It may also be located on a si male plane, with the slope not exceeding | in 50 with ing requir ish ing each aig tn. (Poms ke ofa cd ot provided near the entrance cure Abies than 60 m diameter. Light *C’ may ab - lestrians are large in numbe (sill) Traffic signs : —— rotary intersection should. be neat alc (warning) signs indicating the presen information to traffic. At night a rad cet, Slt approaching roads to give a above the oad level onthe esr Pelletor or red ight is placed at about one island facing the approaching rode. Geel island and onthe heb of Pie ct in he fs improve visibility. DESIGN OF INTERSECTION m0, from the general traffic utilizing the ‘In India the = otaries are needed in urban areas tine Be number of aoe cat Last Biversead high, making the problem complex. If the number of cyclists are less than $0 per hour they may be permitted to mix up with the other traffic using the rotary : if they are mote. a separate cycle track to segregate cyclists will be desirable. If there ave a large umber of pedestrians, separate foot path with guard rails should be provided around the rotary ‘on the outer side to prohibit them from entering the rotary. However, if they are allowed to cross along the pedestrian crossing near the channelising islands, there wouid be problems of stopping the stream of fast vehicles entering and leaving the retry Provision of crossing facilities to pedestrian by subway or aver bridge is possible atom, bathe popes wold bomererbecouy Conditions when traffic rotary is justified Construction of a traffic rotary needs large area which may be available in rural areas atreasonable cost. But in India generally the volume of fast moving traffic is very low in rural areas. There are various other points to be considered before the construction of & traffic rotary can be justified. ‘The American Association of State Highway Officials, now AASHTO have suggested thatthe lowest limit of traffic volume when a trafic rotary is justified is bout $00 vehicles per hour on all intersecting roads put together and the maximum limit Beyond which rotary may not efficiently Funetion is about S000 vehicles per hour. However, if large proportea of trafic is turing traffic, provision of rotary even outside these limits is justified However the IRC suggests that the maximum volume of traffic that a rotary can cfliciently handle is 3000 vehicles per hour entering from all the legs of the intersection. Keeping in view the mixed traffic conditions, itis recommended by the Indian Roads Congress that traffic rotaries may be provided where the intersecting motor traffic is about 50 percent or more of the total traffic on all imersecting roads or where the fast traffic tuming right is as least as 30 percent ofthe total trafic. Advantages and limitations of traffic rotary Various Advantages of Rotary (Crossing manoeuvre is converted into weaving or merping and diverging operations. Hence there is no necessity of any ofthe vehicles, even those which have to go in cross directions, to stop and proceed within a traffic rotary. Thus the journey is more consistent and comfortable when compared with any other intersection at grade Gi). All traffic including those tuming right or going straight across the equal opportunity as those tuming left. feeulen a (ili) The variable cost of operation of automobile {+ less ata wafie at Tt 2 8 signalized intersection where the vehicles have 10 stOP at eo ae straight distance to be traversed by vehicles which are to tum 10 a Across is higher, still the fuel consumed in the proces ‘of erossing the rotary rotary have. a operation at 8 intersection is likely to be less. This is because ome SEP eT uniform signal is likely to consume fuel requced for traveling speed without stopping, traffic 8 the trafic ; sce ox signa to cota! BE : 9 There is no necessiy of tafe polit oie rsecton andi the simplest lary could funetion by itself as a waco ail, ofall controls. The maintenance cost ishence sino EE. a ‘TRAFFIC ENGINEERING (0) The possible number of accidents and the severity of accidents because of low relative speed. Further weaving, merging and diy are casier and less dangerous operation than crossing. Check on s automatically enforced by proper design. Peed of (si) Rotaries ean be constructed with advantage when the number of j is between four and seven. ne ia (vii) The capacity of the rotary intersection is the highest of all other j grade. The rotary can accommodate a total traffic upto omen bi hour and enable radial streets to carry traffic almost to thei full capacity, Various Limitations of Rotary (i) Rotary requires comparatively a large area of land. costly asin built up areas, the total cost may ten bebe a (i) Where pedestrian traffic is large asin urban areas the rotary by itself cannot (il) In places where there is mixed trafic and large number of eyciss and ped me the design of rotary becom i eee #s too elaborate and operation and control of tr iv) Where the angle of interseti 3 f intersection of two roads than seven imerctng read, ores are usual, oY WHEN there (¥) When the distance between i become troublesome. T=**tions on an important highway is 165, (i) Where there are a large number of cycle and animal drawn rehicles, the extra umber of eyck i i 0 i o be traversed by crossing and right tum traffic is considered tankless id is a tendency to violate the tra i central island, Mic regulation of clock wise movement aroit (vii) When the traffic volume is it a = es in most of the rural areas of. 54.3 Grade Separated Intersections efficient operation, A highway grade separatic intersecting bin ae achieved DESIGN OF INTERSECTION 245 7 occ seo ter owecr Fig. $38 Interchange Ramps “The grade separated intersections have the following advantages and limitations. Advantages of Grade Separation () Moximum facility is given to the crossing trafic. As the roads are separate this avoids necessity of stopping and avoids accidents while crossing, Gil) There is increased safety for tuming trafic and by indirect interchange ramp even right turn movement is made quite easy and safe by converting into diverging ‘o left and merging from left “There is overall increase in comfort and convenience to the motorists and saving in travel time and vehicle operation cost “The capacity ofthe grade operated intersection can practically approach that of the two cross roads. Grade separation is an essential part of controlled access highway like expressway and freeway. Itis possible to adopt grade separation forall likely angles and layout of intersecting, roads. (i) Stage constriction of additional ramps are possible after the grade separation Structure between main roads are constructed. Disadvantages of Grade Seporation (i) This very costly to provide complete grade separation and interchange f of way like built up or urban area or where the onstruction of grade separation is costly, difficult Gi) ww) ” wi) Gi), Where there is limited right topography is not favourable, and undesirable. ~ : Gin ator plain teva, grade separation may introduce undesirable erests and sags the vertical alignment. Grad tion structures Weeds the grades of the 160 “The various types of bridge structures used to separate the pres Of US intersecting highway may be 7 postal prestressed conerete bridges. tlouble decked vehicles are anticipate, the clearance st the bridge structure should be selected depending Upon considerations like site conditions and aesthetics. i rade separated inersttions are classified oO and anders When a od ne ing pile seve We mera the major highway i ‘TRAFFIC ENGINEERING 246 e across ighway, it i called an mn ‘overbridge across another highway, ow ay "the hghvay is taken by depressing i below the ground level ig et rod by means ofan uder-ridge, itis known as under-pass n ice ofthe overpass or underpass depends on topography, vertical iesbagg eer, aesthetic features and preferential aspects for one ofthe we The advantage and dlsadvantages of overpass and underpass are broly Fixed yor Advantages of an Over-pass some drainage problems may be reduced by taking the major hj dia oon fo Pa oo ‘ype ofsiucture when the wider road is taken span of the bridge being smal the cost ofthe bridge structure wil be less, Ins pass of major highway, there isan aesthetic preference tothe main through less feeling of restriction or confinement when compared with the under-pass, Futys expansion or lnteral expansion or eonsiruction of separate bridge structure fay acne highway is possible. s Disadvantages of an Over-pass {n rolling terrain if the major oad is to be taken above, the vertical profile wil have rolling grade line, If the major highssay is to be taken over by constructing hi snbanknrents and by providing steep gradients, the increased grade resistance may {Reet reduction on heavy vehicles. Also there will be restrictions to sight distance long vertical curves are provided. Advantages of an Under-pass There is a wami the turing traffic because the trafic from the erss road can accelerate white Wace to the major highway and the traffic fom the major highway car dec Thuile ascending the ramp to the crossroads. ‘The underpass may be of advantage the main highway is taken along the existing grade without atierston af ie alignment and cross road is deprossed and taken underneath Disadvantages of an Under-pass There may be troublesome drainage problems at ground water level rises high di depressed as much as § m to 7 e the under pass, especially when the ing rainy season and the road at the under-pass is 10 Interchanges ‘ Grade separated intersection with complete imerchange facilities is essential develop a highway with full control of access. When there is intolerable congestion accidents at the intersection of two highways ir carrying very heavy traffic there is no solution than to provide grade separated intersection ©” "@ Some of the types of interchanges are shown in Fig. 5.39, Of all these complete lOve" Jeaf fulfils all the requirements of tuming trate involving the simplest traffic a viz. diverging tothe le and merging from the lef by, Providing four indirect ramps. DESIGN OF INTERSECTION ate Fig, 539 a) (b) & (©) Types of cuoven Lear srchanges (Contd) 247 en en ‘TRAFFIC ENGINEERING 248 arr cuvee eae Fig, 5.39 (d) Types of Interchanges 5.5 DESIGN OF PARKING FACILITY In cities the problem of parking vehicles is becom day. When vehicles are parked on the road side, to other vehicles pass public places and loc: design of parking facil ing more and more acute day by” even for a short while there is restriction by, resulting in congestion and accidents, In shopping centers, ies with offices there is a shortage of parking facilities ies is essential in cities and large tovms. Parking facilites may be broadly divided into two types : (, On-street or kerb parking, (i) OfF street parking 8.5.1 On-Street or Kerb Parking In this type of par king, Angle parking or parallel parking may be allowed in th . See Fig. 5.40. Angle pulang thay be a gh oe nO llawed inthe kerb parking. See Fi. egrees. Angle parking accommodates DESIGN OF PARKING FACILITY 49 AS ANGLE PaRING ANGLE PARKING ¢ Leume vener Ts amocvtone SE ing terns of Kerb Par i coarsest can be ake i ian oeuvre More vehicles per unit length of kerb and Er parking and wpa Oa ‘angle of 90 degree. The width-of road require® Ne king angle uP id of road requis tn paral {88 more with angle parking and in fr aoe cesaking in mre ing is cough al : say AOE en ea ecient than te parallel pking, Out of YAO Ese ahve Aeqree ange is considered the est fom all es \AFFIC ENGINEERING 250 7 ing is generally preferred when the width of kerb parking s or eevar are Tinted. But the parking and tunparking operations Uifcult needing a few forward and reverse movements before parking proper taking ot Paral parking may be wit equal spacing, facing the same dre he two eats placed closely with open intervals between (wo-car units, ag 9h Fig, 5.40. 5.8.2 Off-Street Parking ‘When parking facility is provided at a separate place away from the kerb, it jy as off-stree! parking. The main advantage of this method is that there is np congestion and delay on the road as in kerb parking. But the main draw back i the owners will have to walk a greater distance after parking the vehicle, 7 possible to provide the offsreet parking facility at very close intervals especially business centers of a city, Two basic types of off-street parking facilities are 4 parking lots and multisfloor parking garages, Parking fois may be convenient where sulfcient space is available at compar low cost, ‘The parking of vehicles may be done by owners or drivers of the cars and th this is called self purking system, IF the vehicle is left by the driver atthe entrance sp and xan collected from there, the parking and delivering operations belng carried oul attendants, it is called atvendant parking system, Most important advantage of attend Parking is less space required to store and manocuvre the same number of cars Multtstoreved purking garages are restored to when the floor space available parking garage is tes and is very costly. It iy possible to construct multlesto karaes (0 atk a large number of curs at a time, It is necessary to provide the Int travel facillly for the vehicles, which may be cither by elevators or by ramp Hrechanized garages the elevator may be desiyned to move both in verte! and horizontal directions to carry and place the vehicle in the approprlate parking stall and 0 | deliver it back, If tampy are provided for driving the vehicles to and from the a stall, the space requirement will he Increased considerably. On the other hand, If th a mechanical Leak down oF a power falta, the fettoning ofthe elevator system ‘conte to san st Hoth in the parking tots nnd the gornyes, the basic trafic operitions consist of eps, namely, entrance acceptance, sturnge delivery nnd exit, Hence some definite is required in font of the parking lot oF waroye for vehicles during entrance weed and exit operations, ‘This space provided is called sexervoie urea, the size of Wile Alepemls on the average rate of aitival of vehlclen tw te parked during. peak nye average tine required to llpose ofl one ear and the number oF attendants employe storayge-oprato ‘The tate OF highway aecldenty nnd fiat times higher in terme oF velicle-kilomette, than tnt durlng day driving, O08: Yarlous couses of increased aceldent rate diving night amy bo atrlbuted 1 pode MBL Visibility. Wikhway lighting Is particularly imere impottant ut intersections, Hi level crossings anid In places where thee We extitlon ot tafe ne rrooegetl ‘on rural vals fi sot yet become canny m, evilenily di Hees umber of pedestrians anid other sow trate ual te ally at nigh Of * that ocenir during wight delving 1s HIGHWAY LIGHTING roads where the density of population is also high, road lighting has other advantages like feeling of security and protection, Thus even though head lights of vehicles may be suiicient for safe night driving, stil rad lighting may tothe road users. During night driving the manner in which objects are visible vaties with beth the absolute level of brightness and the relative brightness of the road surface and the abject ‘When the brightness of the object js less than that of the backgyound, that is when the object appears darker than the road surface, discernment is principally by silhouette If | the brightness of the pavement is uniformly increased, discernment by silhovete is enhanced. Hence itis obvious that night visibility on concrete and other light coloured pavements are better than on black top surfaces. A light coloured, rough textured pavement surface that can reflect light back is considered most desirable. Surface that becomes mirror like or shiny when wet (such as smoathened black top road surface) should be avoided as practically no light reflects back from them, When the brightness of an object is more than that of the immediate backyround, discernment is by. reverse silhouette, ‘The objects adjacent to the roadway, projections tbove the pavement surface such as island or vehicles may be seen by this process of reverse silhouette. When the pavement surfice is very dark like black top surface, the ‘object which are relatively brighter in colour ae seen by this process. ‘Thus the varlous factors that influence night visibility are: (a) amount and distribution of light lux fom the lamps, (b) size of object, (6). brightness of object. (4) brightness of the background, (6) reflecting characteristics ofthe pavement surface, 251 be considered as an added facility (1D) glare on the eyes of the driver, and (x) the available to see an object. Devign factors of highway lighting Vatlous fetors to be considered In the design of rod lighting are () Lamps: (1) Luminalte distribution of tight (Hi) Spacing of fighting unite ((8) Height and over hang of mounting (VY) Lateral placement (1 Ling Iayouts Lamp som several comieratons in ‘ho cholo of the Inmp, Hs type, siz ad colo vical te use the pce, 1 €e000 ‘lia db ftp Hux the eT rg my of pvemer lost mp size tna Iuinaee whlch wl TRAFFIC ENGINEERING 252 shuness; but this depends on the spacing ofthe lamps also, ‘The various types lamps. The cheapest amongst these, is the filament lamp. Sodium-vapour limp preferred at large intersections. " Luminaire Distribution of Light To have the best uty’ ofthe luminaire or source of light, iis necessary 0 haye py distribution of light. The distribution should be downward so that high percentage off light is utilized for illuminating the pavement and adjacent area, The light distribu selected should be the one which would produce maximum uniformity of pay brightness, The distribution from the luminaire should cover the pavement between the kerbs and provide adequate lighting on adjacent area i.e. 3 m to 5 m beyond the pa edge. The illumination is necessary for traffic signs and other objects on the road, There are five typical luminaire distributions (see Fig. 5.41) which meet most of th highway lighting requirements. Itis suggested that the average level of illumination oi road side may be 20 to 30 fon important urban roads carying fast traffic and about 15 lux for other main carrying mixed traffic and in arterial roads. In secondary road it may be 4 to 8 lix ‘depending on traffic. However the actual intensity of illumination in most of the exis roads may be lower than the above values. The Indian Standards Institution recommends an average level of illumination of: {ux on important roads carrying fast traffic and 15 lux on other main roads, the ratio of minimum to average illumination being 0.4, Spacing of Lighting Units ‘The spacing of lighting units is often influenced by the electrical distribution Property lines, road layout and type of side features and their illumination. Large lamps with high mountings and wide spacings should be preferred from economy point of view, Height and Overhang of Mounting ‘The distribution of light, shadow and the glare effect from street lamps depend also the mounting height. The glare on eyes from the mounted lights increases with the of the lamp directed towards the eye and ; decreases with increase in height of mo Usual mounting heights range from 6 m to 10 m, possible at least for important urban roads. The mini electric power lines upto 650 volts has been specifi by the Indian Roads Congress. ‘Over hangs on the lighting poles would keep ‘but still allow the lamp to be held above the ker better distribution of light on the pavement effect of mounting height and over hang on is desirable to have higher mounting Lateral Placement The street lighting poles should not be installed close to the pavement edge. If OF are too close to the carriageway, free mover . v "ment of traffic is obstructed, dect capacity of the roadway. Indian Roads Congress has specified the horizontal ¢ required for lighting poles as given below ied as 6 m above the pavement the poles away from the pavement ei ie rb or towards the pavement. This enables — and less glare on eyes of road users. TP the length of shadow is shown in Fig. 542+ is and necessary overhang projections. The clearance specified apply 10 poles ines also, 1 m <2 HIGHWAY LIGHTING 283 narrow roads) ‘Twosway lateral distribation (or for narrow roads) Narow asymm he! ta ‘clam wih async men dan Me ET pnt ‘Wie aye nea itn . younting at centre of ighways and ae 4 Distribation ‘ of Luminai peste aad Tera Om ome se oT [Minimum 0 254 TRAFFIC ENGINEERING =r TENG OF SHADOW Fig. 5.42 Effect of Mounting Height of Overhang on Length of Shadow 4 Lighting Layouts (On straight roads the lighting layout may be of the following types : (a) Single side (b) Staggered (both sides) (c) Central Single side lighting is economical to install; but it Der oer entities ane te ca ae a oor For wider roads with three or more lanes the staggered system or the central lil system may be adopted, These systems of lighting have illustrated in Fig. 5.43. spacing of the lights in each of these systems is decided based on various consid including location, lamp size, height of mounting and lighting requirements. Fig. $43 Lighting Layouts Special care should be taken while locating the lights on curves, Lights are ‘at closer spacings on curves than on straights. The i ieamte me heim iis ar leat on he on 3 Figure SA vertical summit curve lights should be installed at closer intervals ™ HIGHWAY LIGHTING 258 Fig. S44 Lighting Layout on Horizontal Curves ‘At intersections, due to potential conflicts of vehicular and pedestrian traffic, more itumination is required. For simple intersections, in urban area, the illumination should te alleast equal to the stm of illumination values for two roads which form the intersection, See Figure 5.45, A detailed traffic volume and flow study should be made in the cases of compound intersections before deciding the layout of lights. The liging nit shouldbe located near the pedestrian crossing, channelizing islands and signs. The lighting layout fr traffic otaies has been shown in Fig. $46, q A it Fig. S45 Lighting Layout for Intersections Design of highway lighting systems shee of luminaire distribution, the utilization coeficiet aga Pr a verge a of enya een es lamp lumen, mounting height, width ‘of paved arca and spacing brwern lighting cists se ilizaioncoeticint charts ven 8 FE 46. ‘The flowing relatos used fr compares i ‘TRAFFIC ENGINEERING a 256 ‘stacey Sie ca 02 House si0 o cotrriciest oF uniuizaTioN =F wiotn oF anea 88110 guMTING WEIGHT — Fig. 546 Coefficient of Utilization Spacings = er ‘Average Lux Width of road ‘The coefficient of utilization is obtained from the appropriate chart, as in Fi ‘The maintenance factor takes into account the decrease in efficiency of and an average value of about 80% may be assumed. : Example 3.16 Design a strect lighting system for the following conditions ~ Street width — 15m Mounting height — 7.5%'m Lamp size" 6000 lumen Luminaire type — 11 Calculate the spaci ig between lighting units to produce average Lux = 6.0 Solution The rata Povementwidth 1 Mounting height ~ 75 From Fig. 5.46, coefficient of uilization = 0,44 Assume maintenance factor = 0.8 Spacing = Lémp lumen Coefficient of utilization x Mainten Average Lux Width of road = [.8000%0.44 [eset] =232 ‘TRAFFIC AND TRANSPORTATION PLANNING 257 5.7 TRAFFIC AID TRANSPORTATION PLANNING 7.1 Traffic Planning, ing trends in growth of population and traffic around cities andthe steady growth fof national productivity ereate a continuing. demand for improvements highway ‘The problem of traffic accidents and congestion in urban roads is being viewed with grave concem in the recent years. The main causes for this problem are improper planning of road net-work and other roadway facilities and poor wai planning. Hence Fraffie functions nos occupy a good position in Corporation and Municipalities. The functions and duties of traffic engineering units were intially limited to traffic surveys and control devices. But now this branch of engineering has developed considerably and include mony other activities like design, regulation, planning and administrative functions. In municipal organizations a fll fledged traffic engineering unit can be entrusted to look after public safety. Such a trafic engineering unit may have several division such as: (@) Field studies (0) Accident analysis (©) Traffic control deviees (@)_ Design and planning {6) Special investigations (0) Economic analysis and decision theory in engineering design, and (@) Administration “Trafic engineering units should have a proper place in highway deparments or Public Works Department of the States. The financing for trafic engineering activities is ‘another problem. Obviously, the travelling public is more concemed about their safe and Gqick movements and hence a provision can be made to divert part of the income Sbrained from the road users in the form of axes, parking charges, tolls ete towards hese activities. 5.1.2 Urban Transportation Planning Process “The transportation planning process is developed in a series of stages : Anventories (i) Trip generation (iit) Trip distribution Gv) Model spit (0) Traffic assignment (i). Plan preparation and evaluation Inventories i i ti vel istics: in eh ea wl gens Sh a em Fee tne low ules are normally followed for zoning : 258 ‘TRAFFIC ENGINEERING (i) Zones should be homogeneous in land use, Ga) Zones shouldbe of homogeneous traffic generating characteristics, (ii) Zones should conform to enumeration district, natural and physical bari, wy) Zee should not be large enough to produce errors resulting from the as all activities occur at zonal centroid, (0) Zones should preferably have a geometrical sh i centroids . ape for easy determial Detailed surveys are than organised to assess the existing activity’ levy gy transportation facilities. Normally Home interviews surveys, Populatic uwends. Socio-economic characteristics of the population, and wean 2 activites, Traffic volume census; Travel time studies and a Physical | Highway net work are carried out. Some ofthe surveys are explained in Arle §, ‘The information collected are analysed with respect a experatons foring te basis for frberenvel Qemand nays. Trip Generation This is the first stage of the travel de: i mand i Sones wih th tmnt of uber ofp pone es anced oe is defined as the “One-way movement having single purpos ee between a point of or i ace two popular Berean Plt of vgn an a pint of desinion’. Two popular meds i Multiple Regression Analysis Category Analysis In Multiple Regression Analysis a functional relations ttips rer zone and vari i eeshh wis er zone and various socioeconomic activity levels in that is expressed bet YT DOF DIRE + on vos oe HDG KK i” dependent variable-trips produced or attracted in a zone. XI, X2ees oe Kk independent variables that i ne i vat cause generation of trips. where y by = regression coeffici n coefficients which are ccna whic ae tobe eairatd from z ¥ = 119732 +0,0957 x ere x} = number of workers in the zone andy = number of trips produced for work purposes in a zone On the assumption that thi likely to be generated ean pe ale te stable overtime, the fature numbet + fk the independent vatiables in the coal ing the future estimates of th * n. i TRAFFIC AND TRANSPORTATION PLANNING 299 Inthe eategory analysis, the household trip making is considered rater than 20m sp anakiog, The procedure is to divide the households into » st uf catepris and ‘Maer the base year trp rates to each category. In England 10H dite: eispeets seerymally used based on 6 income clases, 3 car ownership levels, and @ bans hold a aor pedi the future rip generation the expected numberof hoch in Such category 2t the design year are to be mul ied by the corresponding trip rate and ‘summed up over the zone. Trip Distribution “Trp distribution isthe stage where the tips generated 094 artracted from cach zone age distributed to any other zone. "Tine most important method for this procedure isthe a arr This model fx based on the principle that the tips between 22 oe {andj are directly proportional tothe number of trips inthe 2one i the number aa tracted vo zone j and are inversely proportional fo some faneion of distance or Separation between the Zones. “The model is as follows = GiA\F y= 2A (530) San I Here Ty number of ips from zone io zone} Gj = Trips generated in zone i ‘Aj = Trips attracted to zane j Fy ~ Empirically derived “ition factor’ caleuated on area wise basis ‘n= number of zones inthe urban area. Existing data is used inially to calbrate the mode parameters Fy through « Su ‘mmation of the variable over all the observations REFERENCES 261 Its convenient to prepare a tabular form for computations Zone x y xy 1 i 26. 12 32, 676, 1a] 2 [38 1 308 184 ni 31 i S27 961 2s 433 1s 95 1089 Ds] 3 | 12_| 264 aes ae 6 [30 15 350 300 Bs 7 | 20 180, 400 _t a [25 3 335 625 11 qa 8] Sx= 25 [Dy 104 [Dy = 2861] Sx = 5919 |= 398 Substituting these values inthe formulae given earlier 1=215 104 8xS919-(215)° bo = (104 ~ 0.469 x 2159/8 = 0.396 by = 0.469 ‘Therefore, the trip generation model is y = 0396+0.469% ‘The correlation coefficient for this model is 8«5919-215))] += oo (tama a “The linear regression model is given by : y = 0396 +0469 x,r= 0.82 (b) The future population of a zone = 40,000 x = 40 for use in model ‘The total trips generated, y = 0.396-+ 0.469 x 40 = 19.16 in hundreds = 1936 REFERENCE 1. Matson, TM, Smith W. S. ond Hurd Fo W., Traffic Engineering. McGraw-Hill Book Co,,Ine,, New York, 2. Evans, H. K Traffic Engineering Hand Book, institute of Trafic Engineers, Connecticut USA. 3, Specifications ond Standard Comite, Traffic Rotries, Jour, Indian Reads Congress, Vol. XIX-3, 1954-55. 4. Copelle, D. G, Cleveland D. E and Rankin, W. Wan Itoduction te Highway ‘Transportation Engineering, Insitute of Traffic Engineers, USB. 5, AASHO, A policy on Geometric Design of Rural Highways. “American Association estate Highways Officials’ Washington. (Now 4451170) a. 2. 3B. 24 28. 26. . Burrage, RH and Morgen, E G., Parking, The Eno Foundation for Hi . Indian Roads Congress, TRAFFIC ENGINEERING sata & Pond Aggarwal, A. L, A Study of Tramey {pets Journal, Indian Roads Congress, Vol. XX1V.5, 19) Bolan HL Walled Cif ot Sehgal TR. A Prelim Rawk Congress, Vol. XXD Gunnar N S and Gupta, J D, Parking Demand of Different Land | SEESSp. tourna dian Roads Congress, Vol. XXXIV, 1971, Aahonsraman § and Schgal TR, Methods for Estimating F a Joumal Indian Roads Congress, Vol. NXX-3, 1968, ven Svinnatan NS and Suryonaravana, Y., Forecasting Methods i Planning, National Seminar, 20-years Design and Consreciar Ah Badges, Miniscy of Tran:port, Govt. of India, 1968. Kenneds, N. Kell JH and Homburger W. S., Fundamentals Engineering. institute of Transportation and Traffic Engg., University of C Wokl, M. and Martin B. 1", Traffic System Anal neers and NcGraw-Hil Book Co. New York. ne of ae HRB, Highway Capacity Manual Speci i ea ay Capac Special Report 87 Highway Research Bas ‘HRB, Channelization, The Design of Hi Intersectic Grade, c: Mtighen Raweth Bowe Wassngee Beene ‘Traffic Comrol, Connecticut, USA, Khanna, SK. Jitender Kumar and Agarwal, G. K., Analysis of Vehicle for High Si ice See ‘Symposium, Modern Trends in Civil Engg. U Carmichael T J, Motor Vehicle ighway Safety, mi 7 cle Performance and Hi / Highway Research Board, Washington Vol. 33, 1953. . How to Pl 3 low to Plan Modem Street Lighting, General Electric New York (USA). = Specfcatons ond Staderd C vase “ommittee, Dimensi id Weig f Road D ‘le, Journal, Indian Roads ‘Congress, wee ee 5 Indian Roads Congress, ignment, Roads fecommendations about the Ali Geometric Design of Hill oad, IRC 52-1981 (Fist Revision). Highoays RC 73q9n0, Ceomete Design Standard for Rural Indian Roads Cor IRC : 86-1983, aa Justa, CE G., and Tuladher, Sura oft nian Reads Congres Vane oat Vast Indian Roads 3 cee Forms A-1 and IRC : 53. !adion Roads Congress, Code of Practice for Road Signs, RC : 67-1977. Geometric Design Standard for Urban Roads PROBLEMS. a 27. Walls, GR. Traffic Enginecring: An Icboction, Crit Lone. 2 tndian Rows Comress,Tesetve Guntline om Capasso Raa 2 Ral Aa. IRC: 64-1976 29, Indian Ross Congress, Reconmansed Practice for Rotates IRE 30. tndin Standart Inston, Laminates foe Street Lgheing, 6 3146078 31. Indian Sanders Intiatons 1S. Code of Practice for Lapaiog of Peas ne, Fare, 1S. 1944 Part 1 11,1970, PROBLEMS eit 1, Whatare the objects and scope of traffic engineering ? Faplasntiefy +2, What is the significance of road user characteristics in aff 7 Daca Tibi the various factors which affect the road wer characteristics a hie eee in traffic performance. 3. What are diferent vehicular characteristics which affect the road design ety explaia. “4 tndiate the maximum dimensions and weight of vehicles alowed stn # Specified by LR.C. Discus the effect of wider vehicles on the reas. . Discus the various trafic sti and their importance. 6 Explain te term uaffic volume, Wht ae the objects of carrying ou talfic wane studies ? +. Enumerate the diferent methods of carrying out traffic volume states, ciate Se principle of each, 4 Indicate how the trafic volume data are presented and the resus ea Tae cngineering 4. Explain spot speed, running speed. space-mean speed time-tnean speet art aweras speed. How are spot speed studies cartied out? 10. What are the various objects and applications of spotspecd stasies 7 11 Explain how the speed and delay stalin are caried out What are Shr earins mee ‘of speed and delay studies? 12, Explain origin and destination study, What are the various wen of O 8 D sie. 13. Explain the traffie manoeuvres and their applications, 14, Explain traffic capacity, base capacity, possible capacity and praca! 70 . Show the conflict points atthe intersection of the following types : ‘The relationship between the hourly traffic volume as percentage of ADT tater of hours in @ year when the traffic volume exceeds is as given below: road. % No. of hs. exceeding | ' | 5 | 10 | 20 | 30 | 60 | 90 | 150 wAADT [24 | 20 [18 [16 [sé | 1s [| 4 | 3 If the AADT projected for the design period is 8,500 estimate the design ‘volume justifying your results. {(@) Cross-roads, both two-way (b) Cross-roads, one-way (©) T-imtersection both two-way (d) Y-imtersection, one one-way B. The width of road A is timings. (a) By trial method (b) By simple method with pedestrian signals, shpat note dines ated er of weight 0.8 and bth the veil speed of the van sa Pefore coming to stop. Calculate the ° ibs not ‘ tod skids through a distance fz newer alison Gi) it applies PROBLEMS. 267 57, Two trucks A and B of gross weight 7 T and 12 7 approaching from right angles applied brakes and skid through distances 3.2 m and 1.9 m respectively before collision. After collision truck A was thrown back making an angle of 50° with ts original direction and skid through a distance of 2.6 m. Truck B skid along a stance of 3.8 m deviating at an angle of 60° from the original path. Estimate the initial speeds of approach of the two trucks. 158, The following Table provides the land use and tip data. Develop @ mode! for trip attraction : Compute the correlation coefficient Zone No._| Total Employment | Trips Atiracied | i 9482 9428 2 2010 2192 3 374 330 4 127 133 3 3836 3948 6 953 1188. 7 2 240 s 36 35 9 2223 2064 10 22 280 59, Develop a model for the following data: Education Tips we | 6 Te 20 ‘School Population | 20 | 24 | 28 [32 | 36 [39 (Y) Chapter 6 “ y Highway Materials 6.1 SUBGRADE SOIL. 6.1.1 Significance of Subgrade Soil Subsrade soil is an integral part of the road pavement structure as it provides ti support to the pavement from beneath. The subgrade soil and its properties are impo in the design of pavement structure. The main function of the subgrade is to, gf adequate support to the pavement and for this the subgrade should possess suff stability under adverse climate and loading conditions. ‘The formation of waves, corrugations, ruttins jing it : s. Tuting end shoving in black top pavements the phenomena of pumping, blowing and consequent cating Seca ‘conered pavements are generally attributed due to the poor subgrade conditions. } When soil is used in embankment cx i ic soil is onstruction, in addition to aurea lity is also important as differential settlement may cause ‘ompacted soil and stabilized ‘Soil are often used in sub-base or base course of highw pavements. The sol is therefore considered as one of the principal highway materials. 6.1.2 Characteristics of Soil ete pat of mineral matter formed by the disintegration of rocks, by indieidoal et fost temperature, pressure or by plant or animal life, Based on ‘peividual grain size of soil particles, soils have been classified as gravel, sand, sit sy. The characteristics of soil grains depend on the size, shape, surface & chemical composition and electr ity i teenginering bear oa sity 7 MESS Meister and dy dey it 6.1.3 Desirable Properties The desirable properties of soil as hi as a highway mi () Stabitiy _— Gi) Incompressibilty (Gil) Permaneney of strength 268 SUBGRADE SOIL, 269 (iv) Minimum changes in volume and stability under adverve cooditions of weather and ground water (¥) Good drainage, and (vi) Ease of compaction. ‘The soil should possess adequate stability or resistance to pertnanent deformation tunder Toads, and should possess resistance to weathering. thus retaining the deveed subgrade support. Minimum variation in volume will ensure rinimum variation in differential expansion and differential strength values. Good drainage is essential to avoid excessive moisture retention and to reduce the potential frst action Ease of compaction ensures higher dry density and strength under particular type and amount of compaction. 6.1.4 Index Properties of Soil ‘The wide range of soil types available as hi ‘obligatory on the part of the highway engineer to identify and classify the ciff ‘The soil properties on which their identification and classification are based are known 35 indee properties. The index properties which are usually used are grain sie distitution. Tiquid limit and plasticity index. Further the properties which are some times used are ‘shrinkage limit, led moisture equivalent cenrifuge moisture equivalent and compacted dry density. Grain Size Analysis ‘The grain size distribution is found by mechanical analysis. The components of sous which are coarse grained may be analysed by sieve analysis and the soil fines by Sedimentation analysis. The grain size analysis ot the mechanical analysis is hence arried out to determine the percentage of individual grain size present ina soil sample The sieve analysis is 0 simple test consisting of sieving a measured quantity of the material thraagh successively smaller sieves, the weight retained on each sieve is expressed ara percentage ofthe total weight of sample. The sedimentation principe. hat the larger grain a suspension settle faster, is used for finding the grain sire distribution of fine soil fraction passing 75 micron sieve, Two methods of test viz. : Hydrometer ‘method and Pipette method are used based on sedimentation principle. ‘The details of the {Dal size anatys tests as well a all fests on highway materials have been given bythe Authors in their book. Highway Material Testing. Consistency Limits and Indices ‘The physical properties of fine grained sols, especially of clays difer very must at different water contents, A clay may’ be almost in aliquid state. or it may show plastic tbehaviour or may be stiff depending on the moisture content. Plasticity is 3 property of futstanding importance for clayey solls, which may be explained as ability to undergo Changes of shape without rupture. Auerberg in 1911 proposed a series of tests mostly empirical, forthe determitation ofthe conssteney and plastic propertcs of fin soils ‘These are known as Aiterberg limits and indices. i ‘which the soit wil low ‘Liquid limit may be defined as the minimum water content at which the soit will dn ay sal shearing ce Te gut iis ly termined in the laboratory using a mechanical device. co Wr te nim dg st : Jastic sate. This Jower limit is arbitinely te rence between the tiqu ‘ovisture Content Over hie at whic further reduction fi owest water content that e the plast th values of liquid limit and plasticity’ index are poor engineering materials Both ligui limit and plastic limit depend on the type and a fof ciay in soils, The plasticity index generally depends only on the amount of present: giving an indication of clay content in soil. In soil having same values of lig limit, but with different values of plasticity index; it iS generally found that rate Volume change and dry strength increases and permeability decreases with incre plasticity index. In soils having same values of plasticity liquid limit, it is seen that compressibility and permeal wrease and dry str déecreases with increase in liquid limit. Thus the values of liquid limit and plas index help in classifying the cohesive soils. {In addition to the above tests certain other properties have also been some time us idemtifying and classifying soils. These include shrinkage limit, field m ‘equivalent, centrifuge moisture equivalent and compaction characteristics of the soils. Field moisture equivalent of a so absorbed water aie fully satisfied. The centrifuge moisture equivalent of a soil ‘moisture content retained against a force of 1000 times gravity for one hour. These are seldom carried out now-a-days. In most of the soil classifications systems that commonly in use, the classifications are based on the grain size distribution (by sh analysis), liquid limit and plasticity index of the soils. 6.1.5 Soil Classification Based on Grain Size There are several classification systems for soil grains based on grain size of according to which soils have been classified as gravel, sand, silt and clay. The limits of grain size for each of these components are not same in all these classifi ‘The most widely accepted grain size classification system is the M.LT. classificall system. The Indian Standards Institution (1.S.1.) has also adopted the same limits M.LT. system for the Indian Standard Classification ‘System of soil grains. The Jimi the grain size for each component as per this system are shown below: ‘Sand Sit |Coarse| Medium) Fine | Coarse | Medium Clay Fine | Coarse | Medium Gravel} i 06 = 02 002 9.006 2.0" *Values are in mm. 0.06 ‘oxtufil Classification ihe textural classification system i ot heipfia in classifying a suit which cont dipsat ood iy. Atypical textural classification chart for 7 trea Gsnggested by C28) Barca of Pu % of Pees me sur Fig. 6.1 Textural Classification Chart 6.1.6 Soil Classification Syste “The various soil classification systems in use inthe field of highway engineering a= (i Burmister descriptive classification Gi) Casagrande soil classification ii) Unified soil classification of Revised Casagrande soil classification and |. S. sat classification systems. : (iv) U.S, Publi Roads Administration (PRA) classification igi ¥) Highway Research Board (HRB) or American Association of «eet (AASHO) classification o Revised PRA classification. (i). Federal Aviation Agency (FAA) classification, (vii) Civil Aeronautic Administration (CAA) classification (vii) Compaction classification. Of these systems, the Unified sol classification system has been very wiely acereed in general for the classification of soils for civil engineering and the HRB, raccteation or the revised PRA system is adopted forthe classification of subgrade sols ification systems are given here rway Engineering. Hence these two classification systems ae EP Unified soit classification system in 1942 to classify soi. “The Casagrande classification systera which was developed in 1942 wo : th the U. S. Crops of Engineers and the lute revised, modified and adopted by both the U. S. Crops of EASInec™, Te ae aa Rcelematon and was renamed a Unie Sol Clas pL ee a A eee an HIGHWAY MATERIALS ‘The soils ae divide into wo board groups, coarse grained and fine graingg, rain size. The coarse grained soils include gravels (groups symbol G) and sare’ symbol S). Each of these component have been subdivided as Well graded on Well raded wit clay binder symbol ©) poorly graded (symbol ym containing considerable proportion of silt symbol M). ‘Thus a well graded ei Well graded sand SW, poorly graded gravel GP and so on, ‘The fine grained soil with more than half passing 200-mesh 1. §, si 5 size) have been divide into two groups : pa (Soils with liquid limit less than SO percent or soils with compressibility (group symbol L). pat Gi). Soils with liquid limit greater than SO percent or soils with hi (group symbol H). Symbols M,C and O have been assigned to inorganic silts incl inorgani igh comp luding J i ML, inorganic sit with igh plasticity is MH and so on. Unified soil classification groups, symbols and limits of test propert r , roperties are gi Table 6.1. This sll clsifeaon sytem makes use ofthe role of on od id Timit and plastic limit tests. The classification group of the fine grated" found by making use of plasticity chart given in Fig. 62. ~ Table 6.1 Unified Soil Classification aes ia na RTT FE | omesewuae| ou [Mindionsomapcin mere itan | ile orm foes” ne coarse kel ‘maternal lager | sim seve Fines? ce [Clavey em ‘than No 200 {more than 12%) foil aera eee ear SS ers a (0074 eren) | mote than | title or no finest | SW [ing © alin ‘half of | (Less than $85) i pee sie oe Par ara pe arco fraction, ‘Sands wath Sk ite sae —Har iter parm a | eee | ae bea a ma ca a om [ant | acy sais sre than | 481 Mit ess han 80=L= [can clays rally andy a 0% materials js eciate oe Wurman) | Stesetclyswiiqud [MY Iricrcoueuuy “SS *4| amt greater than $0" | CHT [inmeyarue fut cays *Fises are shove: fe reine sails i Feat and iter Tagly ogame aol ere LS. soil classification ‘The Indian Standard Institute (ISI) has als e ty ne So Coton Sac The Ei age Shc ace i “fa ‘ymbols. The particulars of 1S soil classification Raven li Toble 62 SUBGRADE Som. 2 « 3 ow g > Zao | wy E ec | 220) . BL ec coe & IF pectoris 30 —a0 20 8000 t1auID LIMIT Fig. 6.2 Plasticity Chart Table 6.2 1S. Soil Classification ‘Seb Dirision_ “Soi Group Description ig eee | mayen, fatgon faa parieiioeaae sea To] face = eee Se ee —— te i Pooh gad paar pls arr wih eer eer 1 ea — area ee = ea = ies 2 ee See = \Ssewsso Petras ace para sation oa 7 Se a eae Eee eo ee" eee aera To ee es q ee ea eet . aa ee eae = SS a ae ore z = ee ‘i Highway Research Board (HRB) classification of soils This is also called American Association of State Highway Officials (AASHO) classification of Revised Public Roads Administration (PRA) soil classification system. The original soil classification was developed by Bureau of Public Roads in 1928. After about 15 years of experience, certain revisions and modifications were made and the HRB classification system was developed. In fact by these modifications it is now possible to have subgroups for properly classifying different soil types and the number of Classification tests were decreased from six to three, thus enabling classification of soils by three simple laboratory tests namely, sieve analysis liquid limit and plastic limit. Soils are divided into seven groups A-I to A-7. A-1, A-2 and A-3 sols are granular soils, en i ie Gt fen eve bang kw tun 3. A AS, AS an Av, soba Fine gyained or silvelay soils, passing 0.074 mm sieve being greater than 35 percent 274 HIGHWAY MATERIALS ‘AcL soils are well graded mixture of stone fragments, gravel coarse and nonplstie or slightly past soil binder. The soils of this group ae sup two subgroups, A-I-a, consisting predominantly of stone fragments or gray) consisting predominantly of coarse sand. ad) ‘A2 group of soils include a wide range of granular soils rangi groups, consisting of granular soils and upto 35% fines of A-d, A ee Ala Based on the fines content, the soils of A-2 groups are subdivided imo sub 23 A-2-5, A-2-6 and A-2-7. "groups Ay 3 soils consist mainly, uniformly graded medium oF fne sand similar toby or desert blown sand. Stream-deposted mixtures of poorly graded fine sata coarse sand and gravel ae also included in this group. Bh 4 soils are generally silty soils, non-plastic or moderately plastic in liquid limit and plasticity index values less than 40 and 10 respectively, ‘A-5 soils are also silty soils with plasticity index less than 10%, but with te values exceeding 40%. These include highly elastic or compressible, soils, us diatomaceous of micaceous character. A-6 group of soils are plastic clays, having high values of plasticity index ex 10% and low values of liquid limit below 40%; they have high fim change with variation in moisture content, AT soils are also clayey soils as A~6 soils, but with high values of both lig and plasticity index, (LL greater than 40% and PI greater than 10%)... These so ow permeability and high volume change properties with changes in moisture Groups Index of Soil Fine grained soils of each clas subgrade material. _ln order to classify tho fine grained soils within one group and for jud suitability as subgrade material, an indexing system tae ee ae lassification which is termed as Group Index, Soils are thus assigned umerical numbers known as group index (Gl). Group index is function of ‘material passing 200 mesh sieve (0.074 mm), liquid limit and plas dex of is given by the equation : GI = 02a+0005ac+o001bd ication group exhibit a wide range of pro Here, a = that portion of material passit ing 0.074 mm sieve, greater than 3: exceeding 75 percent (expressed as 4 whole number from 0 t0 40) 'b = that portion of material passing 0,074 mm sieve grester than 15 and! exceeding 35 percent (expressed as a whole number from 0t0 40): © = that value of liquid limit in ex Qf (expt whole number fam O20) of 40am ess than 69 XPS a= ? that value of plasticity index exceedi 30(68 28a whole number Rom 01920) ne 7 amd net mone than 200 possible value of group index is 2€° lues of soil fraction passing 0.074 ‘tively higher than 75, 60 and 30 Poorer isthe soil as subgrade material ns “The sub-groups A-2-6 and A-2-7 soils of A-2 group have Gl values 0 10 4, A-4 group of soi! have GI values up to 8; A-S soil up to 12, A-6 soll up to 16 and A-7 soil upto 20, ‘The group index value i indicated as sufi to the soil proup within brackets, such as A-6 (4) or A-6 (16). In this example an A-6 soil with group index value 4 is considered as superior subgrade material than the A-6 soil with group index value 16. ‘The soil groups showing the classification limits of various properties is given in Table 63. In order to classify a soil, the values of test results are attempted to be feted in from Jeft column towards right side of Table 6.3 and the correct group is found by the process of climination. The first group ffom the left to which the test data fits in gives the ‘lassification group. Figure 6.3 gives the chart for finding the group index value form the values of percent passing 0.074 mm sieve, liquid limit and plasticity index, instead of using the Eq, 6.1 for group index. Here the group index value isthe sum ofthe values obtained fon vertical axes from both the charts based on the values of percent passing 0.074 mam Sieve, LL and PL Fig. 6.4 gives the chart for clasifying fine grained soll from the liquid ‘and plasticity index values. GI values have been made use of in the design of flexible pavement thickness in ene ofthe empirical design methods as given in Art. 7.3.2. PERCENT PASSING NO 200 SIEVE or tess or more 5, L ,,. 35 PERCENT PASSING NO. 200 SIEVE Fig. 6.3 Group Index Chart 2 ise Characteristics of Soil Classification Groups tic i int in subsequent years, as the ‘The soil classification systems have gained importance ct yar soils belonging to the different classifi soup pale base ah te canes eng ee nt ees aren spc, cnet me ke ped oi fing aoe ceaion oe “AAs sil classification is useful to predict some of the soit characteristics from simple physical tests. 6 HIGHWAY MATERIALS PLASTICITY INDEX- PERCENT LIQUID LIMIT - PERCENT Fig. 6.4 Chart for Classifying Fine Grained Soil (H.RLB. system) ‘Table 63 Highway Research Board or AASHO or Revised P.RLA. Soil j 3 | _Saois bas Oe = | Seaatessterm | eee, || emer I | Sowa, Soares a 2 FAAS |S cae cartenen (etaikia| | APE TRTE| 5 aoe ie ‘e cea maT ‘te Teme a a eae 2/a) a 3 (Az sl az joa. | me : Ne. 20 sere 5 : ata =(2( 2) = (S| ST] & (Darecontes ofan : aseq ee 0 seve s ina eit a bear — : I eee x | io i ee Example 6.1 ‘The results of sieve analysis of a soil are given below : Sieve size, mm | Percent passing ; 476 60 q 2.00 30 3 0.60 10. a 0.40 A 3 0:20 0 (a) Classify the soil by (i) Unified i and (ii) HRB soil classification systems (bY) Discuss the sulatitny ofc cca cc. eieice 5 SUBORADE Son. Fad Solution (a) Soil classification (@) By Unified System Refer Table 6.1. As more than half (6/f%) is finer than 4.76 mem sieve and there are 00 fines the soil is sand or SW or SP groups. Uniformity coefficient Cy = D# = 278 79 Gradation coefficient Cz = ‘As Cy is greater than 4 and Cy is between ! and 3, the classification group of the sand issW. (i) By HRB System From Table 63, it is found that the classification group is A-l-2. (0). Suitability as Subgrade Material Refer Tables 64 and 6.5 Drainage excellent j Volume change almost none | Potential frost action {none to very sight Suability Thigh ‘Values as subgrade material | very good } Example 6.2, “The properties of subgrade soil ae given below : "passing 0.074 mmsieve = 55% Liquid limit = 50% Plastic limit = 41% (a) Classify the soil by revised PRAVHRB system (b)_ Discuss the suitability ofthe soil as 2 subgrade material, Solution (0) Soil Classification From Table 6 3, as percentage passing 0.074 mm sieve is more than 35, the soil fine grained. LL = 50% Pl = LL~PL=50-41=9 Using the char (Fig 64) the classification group is A-5. Group Index, GI = 0.2.2 +0.005 ¢ + 0.01 bd HIGHWAY MATERIALS 2 SUBGRADE SOIL 28 ine Characteristics of unified Soll Classification Grog 3 Table 6.4 So i ‘ Oa A Subgrte | Volune edglz a\e 2 # & Yaa) weit OS othe | ce | ane geslcia| ila |S 2 tees t| “han | CBR charactaisies esl) * | 313 @ sane |g | api ‘wer? Pees eu zi os Te | W-80| > 830 | Gecient — Ainaaraaa x ons | le | z. |s Plas) t [isl isl 9 re br rae RS | S| Le eelglelaiHl Eley at 2 eo) ce de a i i B\E si it =@0 [585-833] Poor Sight 2 £ GE] tet [THe] see wlil,ls 4 iH jp 2 | Sr eat aa | [SE 7m | Re self if ijejaya Tan 3 £ sit | parr TT WE | SE] — Be —| Row En igi : akg a 216 | VO=40 [585-859 | Fae to poor | Very sag | Sige gil a Bla = ee ee i iby afel ej Flee to | High Oils © Fea pined| WL | FovtaPew | 10-200] S-20 [O1O~558| Far to poor = 3 a: i aby Ta a ee bh ae awa Mhalslal 2 (2 lzlal: an ree Tae oo] ea 45 Fil al A Spt =z 2 = 2 2 Faaganet | WH] Pew | 128-180) 3-8 [276-555] Poor _ eE le fale] 2 i govlecdo seed Wpeme: |G] awen | OST SESE | epee gale |glal ¢ 2 r gti es aa ee Wiealel s le lel: om fe a\48 ; lala a= 55-35=20 Se leae a| 3 g e | 8 ze [82 8 b = 55-15=40 aE (FS iE © = 50-40=10 ae a 2 gee RE. & oS 5 be galas i d= 9-100 (nin) eo re iH Re ni fall GEE 5 AGI = 0.2%20+0,005 x20 x 10 +0.01 a a E Hell Al SEgEE a = ajuideowan VENEDHE bias 4.0+1.0+0 re a telaa lagb Es aepaed Alternatively GI may also be found using charts (Figure 6.3) an He is a ses leas GI=5.0+0=50, Sle le le > lS Soil classification is A ~$ (5), a2 |i i |e |e | (©) Suitabitity as subgrade material 7 ln |e I From Table 6.5, laf |= [2 = ~ 280 6.1.7 Subgrade Soil Strength “The factor on which the strength characteristics of soll depend are (@ soil ype (ii) moisture content (iil) dey density (is) internal structural of the soil, and (¥) the type and mode of stress application ‘The problem of predicting the stress-strain relationship of ‘soil the diversity in the soil types and the non-homogeneous nature foundations. Generally the highway engineer is interested in resistance to deformation of the soil under the stress applications, is difficu, beg of they the stability gy In a soil mass, the deformation is largely due to sli Hence the shearing resistance in soil represents the strength ie aa soils is complicated as the shear deformation cause reorientation of. pat eae changes in volume, valence bond between particles, thickness and ove’ adsorbed layer of water. anette Though many theories of failures of materials heory t are known, 's is useful one for soils. The basis of his theory is that a rate fs ver ss on the ue ple is definite function of the normal stress acting on Meh ilure occurs by slippage only. Shearing resistance in a soil ° attributed to internal friction and cohesi fb wit Forty Fob sipcbemabet a sion parameters of the soil. For mi tance js made up of both friction and cohesior resistance on any plane is given by Coulomb's empirical law : ee S, = C+otang ‘ He is r ‘ ee Ss cohesion pr unit area, @ is the angle of intemal friction anda is ena auld woe of } depends on the dry density of the soil, grain size dist stars and texture of soil strength hence depends on the value of ¢ and the % by brad deniona pees Cohesion C is the resistance of soil grains to displ intermoleculer avd then he ace OF contact by very fine grained soils as rst inainple sunant eeockemical forces of attraction. Cohesion may be said to it cron cats: both the true cohesion which is due to intermolecular atraction C depends onthe toe of an urtee tension effets ofthe held water. THe clay and the water cemtemt mineral, its size, the surface charges, the proportion leprae eet sa oF its strength property is often determined from is oe intensity and rate of ris depends very much on the type of stress applet °F application. In case of static stresses the period of stress and the intensi . Viscoelastic Beheviou. a Pblication have significant effect on soils loading cycle, the magnitude gfoee seh ated application of stresses, the tude of stress and the number-of repetitions have inf 8 Evaluation of Soil Strength The tests used three groups : ‘SUBGRADE SOIL. 2a1 (i) Shear tests, Bearing tests, end (iii), Penetration tests ‘There are numberof test methods in each group. ‘Shear tests are usually carried out on relatively small soil samples in the laboratory In order to find the strength properties ofa soil, a numberof representative samples from diferent locations are tested. Some of the commonly known shear tests are direct shear test, triaxial compression test and unconfined compression test. Vane shear tests may be” carried out either on a soil sample or in-situ soil inthe field Bearing tests arc loading tests carried out on subgrade soils in situ with a load bearing area. The results of the bearing tests are influenced by the variations inthe soil properties ‘within the stressed soil mass underneath and hence the overall stability ofthe part of the soil mass stressed could be studied. Penetration tests may be considered as small scale bearing tests in which the size of the loaded area is relatively much smaller and ratio of the penetration to size of loaded area is much greater than the ratios in bearing tests. The penetration tests are carried out in the field or in the laboratory. The California Bearing Ratio test and cone penetration tests are commonly known penetration tests. ‘There are number of factors which affect the results ofthe strength tests as mentioned below: ; (i) Factors which are primarily associated with thé actual tests such as size and shape of the specimen, method of loading, rate of loading and drainage conditions. Gi) Factors which are associated withthe soil such as soll type dey density, moisture ‘content, permeability structure and other properties of the soil ‘Some of the commonly adopted tests for evaluating soil sirength characteristics are briefly discussed here. Direct shear test ‘This is one of the oldest of shear tests. The direct shear apparatus consists essentially ofa box divided horizontally into two halves. One half is kept fixed and the other half is free to move horizontally. A vertical load is applied and the horizontal pull is caused to produve a certain rate of horizontal displacement, The vertical and horizontal movements are measured by dial gauges and the horizontal force is noted fom the proving ring dial ‘The maximum horizontal force is measured for different values of normal load. The Valles of maximum shear stress applied for the different vertical stresses are computed hd plowed as shown in Figure 6, The values of cohesion (C) and angle of ital friction (4) are found using Coulomb's equation for shear stress (Equation 62) “There are a numberof limitations of this tet. The file plane being predetermine horizontal plane, need not necessarily represent the imminent plone oF falur The Nearing tess and strain long this horizontal failure plane is seldom wnform. The shearing Stes mor apecimen decreases with displacement. The Mow of water too Hom the soll specimen can nat be easily controlled or measured It 80 0 practicable 10 measure pore water pressure during the test. Carrying out an ‘undrained (quick) test on a Sandy soll rather impracticable inthe shear box. HIGHWAY MATERIALS SUBGRADE Som, 283 Fig. 6.7. A common line tangential to the circles is drawn, representing the Mafr rupture Fe tone. The imercet of thi ine wity Vans repesens te ccbewen Cs the srelination with X-axis represents the angle of internat friction 6 ofthe si Fig. 6.5 Direct Shear Test Results Triaxial compression test “The abject of triaxial compression test isto determine the shear strength of si Ioreral confinement. An altempt is made to simulate the confining pressure Jonded soil mass. In the test set up, it is possible to provide uniform fuid-o pressure only. Cylindrical specimen of height to diameter ratio 2 is inserted in a membrane, placed in a triaxial cell and the required lateral pressure is applied, normal load is applied through vertical piston by means of a strain controlled: ‘and the maximum vertical load at failure is recorded. The specimens: are subjected to a constant strain rate of 1.25 mm per minute. Usually'the lateral pressre, is maintained constant and the vertical pressure; c} is increased until the specirien fl In some studies the volume of the specimen is maintained constant by adjusting by shearing oF in the case of saturated clayey soils by bulging. The deviator under which the specimen fails is (oy - 03). (See Fig. 6.6). The various values of no stress, o} and hence deviator stress, og corresponding to the different values of lat pressure 03 are obtained from the triaxial tests. Fig. 6.6 Stresses in Triaxial Test ‘The i it Inet pga Sared out at diferent lateral pressures preferably wih Rs eel ms mre ot the pavement design, lateral pressure of 0, 0.75 and 1-5 ke failure are plotted and vomit ae sie pra ol OS ing through these points are drawn Fig. 6.7 Triaxial Test Results Correction for Area of Crass Section In is necessary to correct the stress value forthe increased area of eross section due te fonding. The corrected value of deviaior stress is found with the assumption thatthe ‘Volume of the specimen remains constant and the are'ofeross section of the specimen i uniformly inereaged ftom the original value of Ag t0°A1 “due to load Py. Volume ‘Aglo= At y= Ai (lo~ A). In that case the deviator stress eg is given by : ogo Be Ah Hof hd Ay Aol “Ae (bo a= Shan G ie. a a-8 63) Here A = total deformation of the specimen = (lo ~ ft) 8 = unitstrain = d/o ‘Ag = otiginal area of ross section P, = applied load og = deviator stress. ‘After calculating the corrected value of oa, 01 #8 calculated from the relation, 91 = 04+93 Basic Types of Triatlal Tests “The behaviour of the soil specimen during testing is influenced by the condition of drainage allowed during the test. rained tests or slow tests are those in which the drainage of specimen is permitted uring the application of both stresses, 03 and oa: ie specimen is allowed 10 ty consolidate under the confining pressure, 0) and then the deviator stress. oa is *P®! ae raed so slowty that no signiiant pore pressure is bullt up while de specimen is tunder the test. 0 HIGHWAY MATERIALS, th conmitednsined test oF conslidaed-qick test, complete gy aoe under sh contin esse ete applying evita yey, yo pert whe the deviator stress is applied tn undraived or quick test, no drainage is allowed at any stage, prevented before applying the confining pressure, oy and. during th {vitor stress until the specimen fails, EXeess pore pressure commonly, the test In pavements the load applied are mostly transient and- durin, deimage cannot take place in the subgrade soil, In Orde 0 simulate sarees yee subgrade under the pavement, it may be desirable to consolidate the ao oft equivalent confining pressure before the application ofthe load and thus the ee 128 Auk testis justified forthe pavement design. If the pavement is designed for loading conditions as forthe parked or stationary vehicle, then the possi in the subgrade during the loaded period may be considered. The da © pi exis Interpretation of Results of Triaxial Test Besides values of C and > the behaviour of the soil tinder the loading conditions Judged from the stress-strain relationship of the soil as shown in Figure 6.8, These pig seldom indicate & straight ponion even at the intial stages of stressing. Hence it possible to interpret the strength of soil in terms of modulus of elasticity as done case of various structural materials. In flexible pavement design the modi deformation of the soil is used. The modulus of deformation is the ratio of stress to at an arbitrary point on the stress-strain curve. This point is decided based on th anticipated stress in the subgrade under the pavement or an allowable deformation. As an example, if the anticipated stress under a flexible pave p ky/em’, then the modulus of deformation is equal to p/S where 6 is deforms corresponding to a stress of p kg/cm”, in the triaxial test. Now the value of the mod of deformation at any stress level will depend-on the confining pressure 03 -Hence it becomes necessary to decide the value of confining pressure for thet However the term modulus of elasticity is also used instead of modulus of deform for usual computations. ane Fig. 6.8 Stress-Strain Relationship of soils Unconfined compression test The unconfined compression test compression test when the confinis may be considered as a special case of f only in " ing pressure is d | compress applied to the cylindrical specimen, The as ay be applied SUMIKADE SOM, as formation and fond readings are nated until the specimen (ails The ate of exose section of specimen for various strains may be corrected wv at the shure A te Specimen remains constant and thatthe specimen retains eylindeical crtaa section areas fs explained under ttiaxial test. ‘The raximum axial strxin it noned. tn clays when plastic failure takes place, no definite value of maximurn or peak stress cent he niced, In suich cases the stress at 20 percent strain is some time sdoqted The Mohr circle of rupture for an unconfined compression test pres threagh the origin. 11 is not possible to draw the Mohr rupture envelope from a single cwcle, and so the values of C and 4 cannot be determined at such, In case of quick or undrained text carried out on saturated clays, the value of @ may be assumed as zero and bere the Moh envelope will be a horizontal line touching the circle with the unconfined cormpreserve strength qy a8 diameter as shown in Fig. 6.2). Hence cohesion is half the unconfined compressive strengtivin this case i. C = qu. ee (a) For saturated clay {b) For cohesive soil where ¢ = 0 Fig. 69 Unconfined Compression Test Results ‘The failure plane of an unconfined compression or triaxial specimen (when a definite failure plane exists) makes an angle O;= (45 + 4/2) with major principal plane. Hence if ‘the angle made by the failure plane in the specimen with the horizontal, 0, is noted. may be calculated. The value of C also can be calculated from the geometrical relationship of the Mohr diagram, The values of C and 4 may be very easily found graphically as shown in Fig. 6.9 (b). ‘The unconfined compression or the simple compression is suitable to test cohesive soils and all materials having high values of cohesion or bond strength such as soii- cement ete. Plate bearing test % it is ing power of subgrade for use in ‘The plate bearing tests is used to evaluate the supporting power pavement design by using relatively large diameter plates. The plate bearing te We Driginally devised to find the modulus of subgrade reaction i the Westergaand's anys for whee! load stress in cement concrete pavements “The test setup consists of a set of plates of diameter 78, 60, 5 and 30 Ore 8 ice seviee consisting of jack and proving Cn amagemet an recon fa which the jack ean give a thrust 10 loaded brea and dial gauges from this frame are used to mensare the cetement of Vie loaded plate, The loading arrangement is shown in Fig. 6.10 2. ig. 6.10 Plate Bearing Test Set Up Modulus of Subgrade Reaction Modulus of subgrade reaction K may be defined as the pressure sustained » deformation of subgrade ot specified deformation or pressure level, using specin Size, The standard plate size for finding K-valve is 75 em diameter. But in some} smaller plate of 30 cm diameter is also used. The test site is levelled and the plate is properly seated on the prepared surf stifening plates of decreasing diameters are placed and the jack and pro assembly are fitted to provide reaction against the frame. Three or four dial pa fixed on the periphery of the plate, from the independent datum frame for settlements, A seating load of 0.07 kg/cm” (320 kg for 75 cm diameter plate) is and released afier a few seconds. A load sufficient to cause approximately 0,25 settlement is applied and when there is no perceptible increase in settlement or rate of settlement is less than 0.025 mm per minute (in the case of elayey soils soils), the readings of the settlement dial gauges are noted and the average settlements found, and the load is noted from the proving ring dial reading. The load is t eased tll settlement increases to a further amount of about 0.25 mm and the av settlement and load are found, The procedure is repeated till the settlement reaches cm. A graph is plotted with mean settlement versus mean bearing pressure as sh K-value is calculated by the reaction, “ Pp 2, 3) K= Diag Kelem"/em (or kg/em’) F ean serrcnen, oo Fig: 6.11 Load-Deformation Curve from Plate Bearing Test Allowance for Worst Subgrade Moisture Leen sient at the time of carrying out plate bearing tess “a "St moisture condition likely to occur at the test site Tt SUBGRADE Son. 287 practicable always to carry out the test at such a condition. In such eases the value modulus of subgrade reaction K is found for the prevailing moisture content and ‘or value so obtained may be modified for soaked condition. ‘After the plate bearing test, two consolidation test specimens ate prepared One specimen is tested as such without soaking by applying increments of preswate and the pressure deformation curve is plotted as shown in Fig. 6.12, The deformation & of a sample corresponding to a pressure p kg/cm” required in the plate bearing test to cause a deformation of 0.125 em is noted. Then the other specimen is soaked and consul:dation testi carried out; the pressure py required to produce the same deformation 6 noted, in the soaked test., (Refer Fig. 6.12). The modulus of subgrade reaction K, for the soaked condition is then calculated from the relation 65) treaties OF itcminan Fig. 6.12 Correction for Soaking in Plate Bearing Test Correction for Small Plate Size In some cases the load capacity may not be adequate to cause 75 cm diameter plate to seitle 0.175 cm, In such case, a plate of smaller diameter (say 30 cm) may be used. Then K-value should be found by applying a suitable correction for plate size ‘Assuming the subgrade to be an elastic medium with modulus of elasticity E (kgicm*) the theoretical relationship of deformation, A (cm) under a rigid plate of radius a (em) is, ven by : ae ee (66) E Ke a If the value of E is taken as constant for a soil then K is inversely proportional to w or constant ie., Ka = Kay or K = Kyay/a. Hence if the test is carried out with 2 smaller plate of radius ay and the modulus of subgrade reaction K, is found. then the corrected value of modulus of subgrade reaction K for standard plate of radius a, iy obtained from the relationship K+ ki (67 HIGHWAY MATERIALS 288 Example 6.3 A plate load tes plate diameter of readings are given st was conducted on a soaked subgrade during mons a 30 cm, The load values corresponding to the meat $69 plate. {Sean settlement valves, cum [0.0 [0.24[0.3210.76 [1,02 [Tay [Load values kg {0.0 [460 [900 {1180/1360} 1480} Solution The load-settlement curve is ploted on a graph paper (Similar 10 the ong Fig 6.11) andthe laud value pcoresponding mean settlement vale of «30 determined = 1490 kg. 1490 Unit load py = SPS kgfem? niIS* Modulus of subgrade reaction Ky for 30 cm diameter plate BL 1490 A wis? x 0.125 Modulus of subgrade reaction K for standard plate of dia. 70 em. Kya _ 16.86%30 a 5 (Note : As the plate load test was conducted under soaked condition during mon season, there is no need to apply correction for subsequent soaking). California Bearing Ratio (CBR) test = 16.86 kg/em? 5.75 kgem? This is a penetration test developed by the California Division of High method for evaluating the stability of soil subgrade and other flexible pai materials, The test results have been correlated with flexible pavement thick requirements for highways and air fields. The CBR test may be conducted laboratory on a prepared specimen ina mould or in-situ in the field. ‘The leboratory CBR apparatus consists of a mould 150 mm diameter with a bai and a collar, a loading frame with the cylindrical plunger of 50 mm diameter gouges for measuring the expansion on soaking and the penetration values, Briefly the penetration test consists of causing a cylindrical plunger of 50 diameter to penetrate a pavement component material at 1.25 mnv/minute. THE ales to cause 2.5 mm and 5:0 mm penetration are recorded. These loads ar as percentages of standard load values at respective deformation levels to bal value. The standard load values obtained ftom the average of a large mumble? crushed stones are 1370 and 2055 at 2.5 penetration. kg (70 and 105 kg/cm?) respectively The specimen in the mould is subj ae . : pjected to four days soaking and the ‘ter absorption values are noted, The surcharge a is placed of te below. Determine the modulus of subgrade reaction fore i SUBGRADE Som, 29 ER OU METER ‘oencutD 150 PEDESTAL OF TESTING MACHINE Fig. 6.13 CBR. Test Set up specimen in the mould and the assembly is placed under the plunger ofthe loading frame 3 shown in Fig, 6.13. The load values are noted corresponding to penetration values of 00, 0, 1.0, 15,20, 25,30, 40, $0, 75, 100 and 12.5 mm. The load penetration raph is plotted as shown in Fig. 6,14. Alternatively the load values may be converted to pressure values and plotted against the penetration values ‘Two typical types of curves may be obtained as shown in Fig, 6.14. The normal curve it e ing to 2.8 and is with convexity upwards as for Specimen no.! and the loads correspond ” 50'mm penetration values are noted. Some ines a curve wit nial upward eanewity corrected origin C are noted. HIGHWAY MATERIALS, PENETRATION, Mo —— Fig. 6.14 Load-Penetration Curves in C.B.R. Test (Example 64) The passed through the appropriate gauge ofthe length gauge wit the longest idem oder eparate the elongated particles. The gauge length would be 1.8 times the mean se of aR aggregate. The portion of the elongated aggregate having length greater than the Specified gauge from each range is weighed and the total weight of the elongated sc sPexpressed as a percentage ofthe otal weight of the sample, to et the elongation index ongated and flaky aggregates are less workable; they are also likely to break ender smaller loads than the aggregate which ate spherical or cubical, Flakiness index sad Stongation index values in excess of 15 percent are generally considered undesirable owever no recognised limits have been laid down for elongation index. Angularity Number Based on the shape of the aggregate particles, they may be classified as rounded, ieepuler or pay rounded, angular and aky. Angular pale possess well deft wees formed atthe intersection of roughly plane faces and are commonly fund sear tes prepared by crushing oF rocks, Since weaker ageenes may Ne SAS srarrenpacton, the anguaymumber docs not apply t ay arent cs BT) Sere core tis est Anglaiy or absenee of rounding ofthe parce of n age so ag itch is of mpotance beaut iat the xe of nding As spree) inde. The determination of angulriy number of a Ses ‘essentially 2 laboratory method intended for comparing the properties of fageregates for mix design purposes “The deere of packing of particles of single sand asec, TENS the shape and angulority of the aggregate. Hane the angularity of the ageree2" propenies of voids in a. sample of ageresste ee ropes of ois pect Sl von. The lel YOM oo er aor ee eel in especies amet, Ne PES ngolunty mabe, re) of xt rounds gravel in a well ane gt repent the volume of sods (im SEM SL ary ompacted sate which would then have 39 Boe Se aes ensures the voids in escess of 33 perents The hg te te oscil geregate. The range of angular rab or see constr 300 HIGHWAY MATERIALS fhe apparatus for testing the angularity number consists of 5 caput ftv, ramping rod and a metal scoop. The test sample ig sieves tS cerranges of the aggregate, such as 16~20:mm, 12.5 ~ 16 mm, ete are seta ‘scoop full ofthis single size aggregate is placed in the cylinder and ta by the rod. Second and third layers are placed and tamped similarly aggregate is stuck off level to the top surface ofthe cylinder. ‘The weight ory the eylinder i found to be We. Then the cylinder is found = Cg. ‘The specin® of the aggregate is also determined. The angularity number is found from hee ‘Angularity number = 67 - pa 3 This value is expressed as the nearest whole number. n tests a Specific gravity and water absorpt ‘The specific gravity of an aggregate is considered to a measure of the strength of the material. ‘Stones having low specific gravity values are generally than those having higher values. The specific gravity test also helps identifying th specimen. Stones having higher water absorption value are porous and thus weak. q are generally unsuitable unless found acceptable based on crushing and hardness tex About 2 kg of dry aggregate sample is placed in wire basket and immersed in for 24 hours. ‘The sample is weighed in water and the buoyant weight is found aggregates are then taken out weighed after drying the surface. ‘Then the ag dried in an oven for 24 hours at a temperature 100 - 110°C, and then the dry determined, The specific gravity is calculated by dividing the dry weight of aggregatet ‘weight of equal volume of water. The water absorption is expressed as the percents absorbed in terms of over dried weight of the aggregates. ‘The specific gravity of rocks very from 2.6 to 2.9. Rock specimens having mored 0.6 percent water absorption are considered unsatisfactory unless found acceptable bi on strength tests, However slightly higher value of porosity may be acceptable aggregates used in bituminous pavement construction, if the aggregates are otherwise suitable. Bitumen adhesion test Bitumen and tar adhere well to all normal types of road aggregates provided dry and are free from dust. The process oF inal binding Pe eiroled lores viscosity of the binder. In the absence of water there is practically no adhesion pr in bituminous construction, The problems are observed due to the presence of, First if aggregate is wet and cold, it is normally not possible to coat with a bit binder. This problem can be dealt-with by removing the water film on the agers drying, and by increasing the mixing temperature. Second problem is siripping of from coated aggregate due to presence of water. This problem of stripping is 8 experienced only with bituminous mixtures which are permeable to water. is due to the fact that some aggregates have greater affinity towards water bituminous binders and this displace z i ae onthe system, Placement depends on the physico-chemi Most road stones have surfaces that are electri arte my re electrically charged. As an © 2 Common constituent of igneous rocks possess a weak negative charge and het have greater attraction with the polar liquid water than with bituminous binde’® STONE AGGREGATES mat linle polar activity. These sgereyats which are elecuon ate electioneyative ate waterlinking and are called Aydrophillic. Basic aggregates like lime-stones have dislike for water ae greater attraction to bitumen, as they have positive surface charge, These aggregates ate calied Iydrophobie TS It is important 10 know the type of charge of aggregates used in biummous construction. Now bitumen is also available a8 cationic or positive and anionic or negative and hence a suitable selection may be made depending on aegrepate. avaiable Cationic (+) bitumen may be selected for electronegative aggregate and anionic (-) bitumen for electropositive aggregates, Several laboratory tests have been developed to arbitrarily determine the adhesion of bituminous binder to an aggregate in presence of water. These tests may be ciasified into six types. (Static immersion test Gi) Dynamic immersion test (ii) Chemical immersion test (iv) Immersion mechanical test (¥)_ Immersion trafficking test and (i) Coating test, The static immersion test is very commonly used as it is quite easy and simple. The principle of this type of testis by immersing aggregate fully coated with the binder in Water maintained at specified temperature and by estimating the degree of stripping. The result is reported asthe percentage of stone surface that is stripped off after the specified time periods. IRC has speeified that stripping value of aggregates should not exceed 25 pereent for use in bituminous surface dressing, penetration macadam. bituminous Tmacadam and carpet constructions, when aggregate coated with bitumen is immersed in waterbath ay 40°C for 24 hours. 6.3 BITUMINOUS MATERIALS 6.3.1 Introduction Bituminous binders used in pavement construction works include both bitumen and tar. Bitumen is a petroleum product obtained by the distillation of petroleum crude where-as road tar is obtained by the destructive distillation of coal or wood. Both bitumen and tar have similar appearance, black in colour though they have different characteristics. Both these materials can be used for pavement works. zl genous origin, found in Bitumen is"hydrocarbon material of either natural or pyrogenous origin. found! | ‘gaseous, liquid, semisolid or solid form and is completely soluble im Carbo disulphide tamer ial and occurs ether ind in car ‘chloride, Bitumen is a complex organic material aan con a ria ately rng te dso of pelea, Binds tighway construction beeause of ther binding and their water proofing properties. When the bitumen contains some inert materia asphalt. Asphalt is found as deposits in the form o naterial oF minerals, iis some times called eral mira me i HIGHWAY MATERIALS «of bitumen used for pavement construction work of roads ang gz aust nres ‘grades and those used for water proofing of structures and ings career called industrial grades. The paving bitumen available in In aare clan twvo categories (i) paving bitumen from Assam petroleum, denoted as A-type and designateg ABS. A 90, ete. iy. paving bitumen from other sources denoted as S-type and designated as pra $90, ete. Brades sminous Materials 6.32 Types of Bituminous material used in highway construction may be broadly divided as (i) Bitumen and (i) Tar Bitumen may be further divided as petroleum asphalt onbitumen and native asp ‘There are different forms in which native asphalts are available. Native asp associated with a large proportion of mineral matter are called rock asphalt: viscosity of bitumen is reduced some times by a volatile diluent; this material i ‘cutback. When bitumen is suspended in a finely divided condition in an aqueous, and stabilized with an emvulsifier, the material is known as emulsion, Tar is the liquid obtained when natural organic materials such as wood and coal are carbo destructively distitled in the absence of air. Processing of bitumen and: bitumt products is diagrammatically represented in Fig. 6.19. 6.3.3 Bitumen composition, The portion of bituminous material present in the petroleums ma), Uilfer depending on the source. Almost all the crude petroleums contain cons amounts of water along with erude oil. Hence the petroleum should be dehy before carrying out the distillation. General types of distillation processes are "#8 distillation and destructive distillation. In fractional distillation the various constituents are separated at successively higher temperatures without SUBSE chemical change. The successive fractions obtained yield gasoline, naptha, kerose® lubricating oil: the residue would be petroleum bitumen. In destructive distil ‘material undergoes chemical change under the application of extreme heat an PE ‘The process is usually applied for the manufacture of tar. Steam distillation of Pete is employed to produce steam refine petroleum bitumen in order to remove NBN point constituents such as heavy lubricating oils without causing chemical © When the residue is distilled to a definite consistency without further bitumen obtained as residue is called siraighrun bitumen. Requirements of Bitumen ‘The desirable properties of bitumen depend on the mix type and the consttuel general problems while using bitumen in paving mixes are = G) mixing BITUMINOUS MATERIALS 303 Fig. 6.19 Processing of Bituminous Products Gi). attainment of desired stability of the mix (if) to maintain the stability under adverse weather conditions {iv) to maintain sufficient flexibility and thus avoid cracking of bituminous surface and (¥) to have sufficient adhesion with the aggregates in the mix in presence of water. In view of the above problems, the bitumen should possess the following desirable Properties : (i) The viscosity of the bitumen at the time of mixing and compaction should be adequate, This is achieved by heating the bitumen and aggregate prior to mixing or by use of cutbacks or emulsions of suitable grade. itu i re susceptible. During the (ii) The bituminous material should not be highly temperature suscepti hottest weather of the region the bituminous mix should not Become tou $9 Or unstable. During cold weather the mix should not become 109 : causing cracking of surface. The material should be durable Gi) In presence of water the bitumen shou! ta be adequate affinity and adhesion bet mix strip off from the aggregate. There has i ot the bitumen and aggregate used inthe 208 HIGHWAY MATERIALS 634 Tests on Bitumen Bitumen is avaiable in varity of types and grades. To judge the sj binders various physical tests have been specified by agencies like Insitute, British Standards Institution and the ISI. These tess clue dhtliy tests, softening point tet and viscosity test. For elas studying the performance of bituminous pavements, the penetration ang 8 essential, The other tests like softening point and Mash and fire poi important to guide the paving technologists during field operations, Ig nt 8 been recognized that the above tests are not sufficiem to: define ae suscepibilty of the bituminous materials, ‘The bitumen from dit Possessing same penetration value at a specified temperature mz different viscous characteristics at the application or service "hereore may need intensive correlation with fundamental property like vss ‘The various tests on bituminous materials are : (a) Penetration tests (b) Ductility tests (©) Viscosi (a) Float test (©) Specific gravity test —(f) Softening (g)_ Flash and Fire point test (h) Solubility test (i) Spottest 7 Gi). Loss on heating test (k) Water content test a Penetration test “The penetration test determines the hardness or softness of bitumen by mes epth in tenths of a millimetre to which a standard loaded needle will penetrate ¥ in five seconds, The sample is maintained at a temperature of 25°C. The: Penetration test is shown in Fig. 6.20. Indian Standard Institution has stand ‘equipment and test procedure, ‘ orrumen stam arree seu Fig. 6.20 Penetration Test Concept ‘The penetrometer consists of a needle assembly with a total weight i device for releasing and locking in any postion, There isa gradusted Penetration values to 1/10th ofa millimeter. Refer Figure 6.21. Jit bitumen is softened to a pouring consistency, stirred thoroughly and Containers to a depth at least 15 mm in excess of the expected penetration. Tor anls's are then placed in a temperature controlled water bath at a temp for one hour. The sample with container is taken out and the needle is arate Contact with the surface of the sample. ‘The dial is set to zero or the initial BITUMINOUS MATERIALS. 308 Fig. 6.21 Penetrometer i reading is taken on dal gauge. en and the needle i released for $ seconds. The final reading i Ia least three penetration tests are mae on this sample by texting at distances States Imm sp. Aer each eth eel dsegnged and wiped ith mene and ied. “The depth of peneration is reported in one-tenth millimeter uns. The mean value o thee measurements Is tepored a8 penetration vale, It may be noted tht the penetration value is largely infltenced by any inaccuracy a regards pouring temperatre, Size of needle, weight placed onthe needle andthe test tempers 100 grade bitumen grade is specified in terms of penetration value. 80-100 or 80/ ogi lt ene oe oe Bate Standard test conditions. The pean test apled aims excuse bite. [As road tars ae sft, thepeneatio test canot be carried out on these mat consisteney tests are used for tas, etbacks and emulsions. : in constrcton in Te penermton aes of varius pe often wel pve consion i aaa ee gaconmuctn ie encima ions. tot ines 2 lower penetration grade bitumen like 3040 bitumen it prefered. Ductiity test ee In the flexible pavement constructions where bey peace eel ee . thatthe binders form ductile thin Wims around the agreenc. hs Sees saisfcor binder in proving the ysis! ion of Me SSETPTS on mises, Under tic lads the iuninous pavete IE Aeformation and recoveries. The binder material which fos tt PIS ductility would crack and thus provide pervious Pa HIGHWAY MATERIALS 306 Jed out on bitumen to test this property of the binder. The test peniheshe property of bitumen and its ability to stretch, ‘1 's believed tg peretration slug, bat may fal to satisty the ductility requirements Sl Ensincer would however want that both test requirements are satisfigg ue Penetration and ductility tests cannot in any case replace each other,” "fi ‘The ductility is expressed as the distance in centimeters to w of bitumen can be stretched before the thread breaks. The test at a rate of pull of SO mm per minute. The test set up is shown i section at minimum width of the specimen is 10 mm x 10 mm, — othe ‘Conducted at staat ' scrub. over ao Fig. 622 Ductility Test The ductility machine functions as a constant temperature water bath with a device at a pre-calibrated rate. Two clips are thus pulled apart horizontally ata speed of 50 mm per minute. The bitumen sample is heated and poured in the mould assembly placed on & ‘The samples along with the moulds are cooled in air and then in water bath mains 27°C. The excess bitumen material is cut and the surface is leveled using a et] ‘The mould assembly containing sample is replaced in water bath of the ductility machine for 85 to 95 minute, The sides of the mould are removed, the the machine and the pointer is adjusted to zero. The distance upto the point of b of thread is reported in centimeters as ductility value. The ductility value gets affect:d by factors such as pouring temperature, dimensions of briquette, briqueite in the water bath, presence of air pockets in the modulus briquet temperature and rate of pulling. The ductility values of bitumen vary from 5 to over 100 for different bitumen ‘A minimum ductility value of 75 em has been specified by the ISI for bitumens 45 and above, obtained from sources other than Assam Petroleum (ie. $45,894 the minimum ductility value may be 50 em for bitumens of grades S 35, 0b! these sources. However, as the bitumen produced from Assam Petroleum if I much lower ductility values, the minimum ductility value specified is only 13 6m bitumen grades A 65 t0 200 for use in certain regions. Viscosity test __ Viscosity is defined as inverse of fluidity. Viscosity thus defines the Auld bituminous material. Viscosity isthe general term for consistency and its resistance to flow. Many researchers believe that grading of bitumen SHOU" ‘absolute viscosity units instead of the conventional penetration units. in Fig. 622. They BITUMINOUS MATERIALS out “The degree of fluidity of the binder atthe application temperatore greatly \efluerces tne strength characteristics of the resulting paving mixes High or bom vscanty during snixing oF compaction has been observed to result in lower stability vakuet There w an “ptimum value of viscosity for each aggregate gradation of the mand baurcen grade ‘at low viscosity, the bituminous binder simply lubricates the aggrepse partcies atest of providing a uniform film for binding action Similsely heh visconiny also resists the compactive effort and the resulting mix is heterogeneous in character exiobeing lowe stability values, Orifice type viscometer may be used to indirectly find the viscosity of liquid binders like cutback bitumen, emulsion and liquid tar. According to this method, vxcasrty 1 measured by determining the time taken by 30 ml of the materal to flow from a cup through a specified orifice under standard test conditions and specified temperature Higher the viscosity of the binder, higher will be the time required. Thes ts itwrated = Fig. 6.23. Furol viscosity is a specific test which is used only to measure the viscose of liquid bituminous materials. Its the number of seconds required for $0 mi of mater to flow through an orifice of specified size at specified temperatures, Equipment like sliding plate microviscometer, and Brookfield viscometer are however in use for defining the viscous characteristics of the bitumen of all grades irrespective of testing temperature haan iy sme trick tres moet id wt iy ta ic ei i ea pes enpeme sf ah oe dared te in seconds for $0 mi of the sariple to flow through either 4 0 mm orifice at 25°C en isco 0 of he ee et aaa Float test : e There i a range consistency of the bituminous materials for which neha OY OSE viscometer test nor & penetration test could be used to define he eomesenst mater, The consiteney of mates ofthis group smenresio MAIS The apparatus cnsss ofa float made of lumioum and» ass wit Oe specimen materials to be tested, whichis serewed toe [080 sence inthe collar (mould), cooled 1 a temperature of °C 98 so Te fe Fig. 6.24, The float assembly is floated in waterbath SUES SY need seconds for water 10 force its way through the BAume ah. valle. ‘The higher the float test value, the stir fhe mas

You might also like